You are on page 1of 87

CV EDUCATION SOLUTIONS

NEET/JEE
Date :10/03/2019
Time : 10:31:00 CHEMISTRY
Marks : 2524
3.ELECTROCHEMISTRY

Single Correct Answer Type

1. The standard reduction potentials at 25°C of Li + |Li, Ba2+ |Ba, Na+ |Na and Mg 2+ |Mg are
− 3.05, −2.73, −2.71 and − 2.37 V respectively. Which is strongest reducing agent?
1) Li 2) Ba 3) Na 4) Mg
2. Also the [H + ] for problem 9 using the same data is :
1) 0.00133 M 2) 0.133 M 3) 0.0133 M 4) None of these
3. Four colourless salt solutions are placed in separate test tubes and a strip of copper is placed in each.
Which solution finally turns blue?
1) Pb(NO3 )2 2) Zn(NO3 )2 3) AgNO3 4) Cd(NO3 )2
4. Ionisation depends upon
1) Pressure 2) Volume 3) Dilution 4) None of these
5. When same electric current is passed through the solution of different electrolytes in series the amounts of
the element deposited on the electrode are in the ratio of their:
1) At.no. 2) At.wt. 3) Sp.gravity 4) Eq.wt.
6. A substance that will reduce Ag + to Ag but will not reduce Ni2+ to Ni is :
1) Zn 2) Pb 3) Mg 4) Al
7. Which one is correct relation :
𝜕𝐸
1) ∆𝑆 = ( ) × 𝑛𝐹
𝜕𝑇 𝑃
𝜕𝐸 ∆𝐺 − ∆𝐻
2) ( ) =
𝜕𝑇 𝑃 𝑇
𝜕𝐸 𝜕(∆𝑆)
3) ( ) =
𝜕𝑇 𝑃 𝜕𝑇
𝜕𝐸
4) −∆𝑆 = ( ) × 𝑛𝐹
𝜕𝑇 𝑃
8. For the electrochemical cell, 𝑀 | 𝑀+ || 𝑋 − | 𝑋, 𝐸 ° (𝑀+ | 𝑀) = 0.44 V and𝐸 ° (𝑋 | 𝑋 − ) = 0.33 V. From this data
one can deduce that
1) 𝐸cell = 0.77 V
2) −0.77 V
3) 𝑀+ + 𝑋 − → 𝑀 + 𝑋 is the spontaneous reaction
4) 𝑀 + 𝑋 → 𝑀+ + 𝑋 − is the spontaneous reaction
9. If ϕ denotes standard reduction potential, which is true:
1) 𝐸cell
°
= 𝜙𝑅 − 𝜙𝐿 2) 𝐸cell
°
= 𝜙𝐿 + 𝜙𝑅 3) 𝐸cell
°
= 𝜙𝐿 − 𝜙𝑅 4) 𝐸cell
°
= (𝜙𝐿 + 𝜙𝑅 )
10. In the electrodeposition of Ag, the silver ions are:
1) Reduced at anode 2) Reduced at cathode 3) Oxidised at anode 4) Oxidised at cathode
11. The electrolytic conductance is a direct measure of
1) Resistance 2) Potential 3) Dissociation 4) Concentration
12. The standard reduction potential at 290 K for the following half reactions are,
(i) Zn2+ + 2𝑒 ⟶ Zn(𝑠); 𝐸° = −0.762 𝑉
3+
(ii) Cr + 3𝑒 ⟶ Cr(𝑠); 𝐸° = −0.740 𝑉
+
(iii)2H + 2𝑒 ⟶ H2 (g); 𝐸° = −0.000 𝑉

Page|1
(iv) Fe3+ + 𝑒 ⟶ Fe2+ ; 𝐸° = +0.77 𝑉
Which is the strongest reducing agent?
1) Zn 2) Cr 3) Fe2+ 4) H2
13. What is the value of 𝐸cell ?
Cr | Cr 3+ (0.1 M) || Fe2+ (0.01 M) | Fe
Given,𝐸°Cr3+/Cr = −0.74 𝑉
and𝐸°Fe2+/Fe = −0.44 𝑉
1) +0.2941 V 2) +0.5212 V 3) +0.1308 V 4) -0.2606 V
14. When electric current is passed through a cell having an electrolyte, the positive ions move towards the
cathode and the negative ions towards the anode. If the cathode is pulled out of the solution :
1) The positive and the negative ions both will move towards the anode
2) The positive ions will start moving towards the anode; the negative ions will stop moving
3) The negative ions will continue to move towards the anode; the positive ions will stop moving
4) The positive ions and the negative ions will start moving randomly
15. Which ion has exceptionally higher Λ∞ values?
1) H + 2) K + 3) NH2− 4) OH
16. During electrolysis, the species discharged at cathode are
1) Anion 2) Cation 3) Ions 4) All of these
17. How many atoms of calcium will be deposited from a solution of CaCl2 by a current 0.25 mA flowing for 60
s?
1) 4.68 × 1018 2) 2.34 × 1018 3) 1.24 × 1018 4) 0.46 × 1018
18. What is the potential of the cell containing two hydrogen electrodes as represented ahead,
1
Pt; H2 (g)|H + (10−8 𝑀)||H + (0.001 𝑀)|1/2H2 (g)Pt?
2
1) − 0.295 V 2) − 0.0591 V 3) 0.295 V 4) 0.0591 V
19. Electrolysis rules of Faraday’s states that mass deposited on electrode is proportional to
1) Q 2) Q2 3) 𝐼 2 4) None of these
20. Which will increase the voltage of the cell Sn(𝑠) + 2Ag (𝑎𝑞) ⟶ Sn (𝑎𝑞) + 2Ag(𝑠)?
+ 2+

1) Increase in size of the silver rod


2) Increase in the concentration of Sn2+ ions
3) Increase in the concentration of Ag + ions
4) None of the above
21. e.m.f. of a cell in terms of reduction potential of its left and right electrode is :
1) 𝐸 = 𝐸𝐿 + 𝐸𝑅 2) 𝐸 = 𝐸𝐿 − 𝐸𝑅 3) 𝐸 = 𝐸𝑅 − 𝐸𝐿 4) 𝐸 = −[𝐸𝑅 + 𝐸𝐿 ]
22. The reaction,
1
H (g) + AgCl(𝑠) → H + 𝑎𝑞) + Cl− (𝑎𝑞) + Ag(𝑠)
2 2
Occurs in the galvanic cell
1) Pt/H2 (g) KCl (sol)|| AgCl(𝑠)|Ag 2) Pt/H2 (g) HCl(sol)|| AgNO3 (sol)| Ag
3) Pt/H2 (g) HCl(sol)|| AgCl(𝑠)| Ag 4) Ag/AgCl(𝑠)KCl(sol)||AgNO3 |Ag
23. In the electrolytic cell, flow of electrons is from
1) Cathode to anode in solution 2) Cathode to anode through external supply
3) Cathode to anode through internal supply 4) Anode to cathode through internal supply
24. Use of electrolysis is
1) Electrorefining 2) Electroplating 3) Both (a) and (b) 4) None of these
25. Pure water does not conduct electricity because it is
1) Basic 2) Almost not ionised
3) Decomposed easily 4) Acidic
26. A solution of CuSO4 is electrolysed for 10 min with a current of 1.5 A. What is the mass of copper deposited
at the cathode?
1) 2.096 g 2) 0.296 g 3) 3.029 g 4) 2.906 g
Page|2
27. The hydrogen electrode is dipped in a solution of pH = 3 at 25℃. The potential of the cell would be
2.303𝑅𝑇
(the value of 𝑇
is 0.059 V)
1) 0.059 V 2) 0.088 V 3) 0.178 V 4) −0.177 V
28. An alloy of Pb-Ag weighing 1.08 g was dissolved in dilute HNO3 and the volume made to 100 mL. A silver
electrode was dipped in the solution and the emf of the cell set up
Pt(𝑠), H2 (𝑔) | H + (1 M) || Ag + (𝑎𝑞) | Ag (𝑠)
°
Was 0.62 V. If 𝐸cell = 0.80 V, what is the percentage of Ag in the alloy?
[At 25℃ , RT /F = 0.06]
1) 25 2) 2.50 3) 10 4) 50
29. The conductivity of strong electrolyte
1) Increases on dilution slightly 2) Decreases on dilution
3) Does not change with dilution 4) Depends upon density of electrolyte itself
30. In the concentration cells, the electrical energy is produced due to :
1) Oxidation of fuel
2) Heat energy
3) Chemical reaction
4) Transfer of a substance from one concentration to other
31. Rusting of iron is catalysed by which of the following?
1) Fe 2) Zn 3) O2 4) H +
32. Which modifications are necessary to determine resistance of solution by usual method of measurements
on Wheatstone bridge principle?
1) A.C. should be used
2) A conductivity cell is used
3) Galvanometer is replaced by magic eye or head phone arrangement
4) All of above
33. The standard reduction potentials of
Zn2+ | Zn , Cu2+ | Cu and Ag + | Ag are respectively – 0.76, 0.34 and 0.8 V. The following cells were
constructed
I Zn | Zn2+ || Cu2+ | Cu
II Zn | Zn2+ || Ag + | Ag
III Cu | Cu2+ || Ag + | Ag
°
What is the correct order of 𝐸cell of these cells?
1) II > III > I 2) II > I > III 3) I > II > III 4) III > I > II
34. EMF of hydrogen electrode in term of pH is (at 1 atm pressure)
𝑅𝑇 𝑅𝑇 1
1) 𝐸𝐻2 = × 𝑝𝐻 2) 𝐸𝐻2 = .
𝐹 𝐹 p𝐻
2.303𝑅𝑇
3) 𝐸𝐻2 = . p𝐻 4) 𝐸𝐻2 = - 0.0591 pH
𝐹
35. The molar conductivities of KCl, NaCl and KNO3 are 152, 128 and 111 S cm2 mol−1 respectively. What is
the molar conductivity of NaNO3 ?
1) 101 S cm2 mol−1 2) 87 S cm2 mol−1 3) -101 S cm2 mol−1 4) -391 S cm2 mol−1
36. A correct electrochemical series can be obtained from K, Ca, Na, Al, Mg, Zn. Fe, Pb, H, Cu, Hg, Ag, Au by
interchanging :
1) Al and Mg 2) Zn and Fe 3) Zn and Pb 4) Pb and H
37. The speed of migration of Ag ion and NO3 ion are 0.00057cm sec and 0.00063 cm sec −1 at infinite
+ − −1

dilution. The equivalent conductivity of AgNO3 at infinite dilution is:


1) 140.2 2) 130.1 3) 120.8 4) 115.8
38. Coulomb is the quantity of current defined as :
1) One ampere of current passing for 1 sec
2) One which deposits 0.001118 g of Ag on cathode

Page|3
3) One which deposits electrochemical equivalence of metal
4) All of the above
39. The metal that cannot be produced on reduction of its oxide by aluminium is :
1) K 2) Mn 3) Cr 4) Fe
40. A certain current liberates 0.504 g of hydrogen in 2 hr. How many gram of copper can be liberated by the
same current flowing for the same time in CuSO4solution?
1) 12.7 2) 16 3) 31.8 4) 63.5
41. By how much is the oxidizing power of Cr2 O2− 7 /Cr 3+
couple decreased if the H +
concentration is
−3
decreased from 1 M to 10 M at 25℃?
1) 0.207 V 2) 0.414 V 3) 0.001 V 4) 0.287 V
42. +
What will be the emf for the given cell Pt |H2 (𝑝1 )|H (𝑎𝑞)|| H2 (𝑝2 ) | Pt?
𝑅𝑇 𝑝1 𝑅𝑇 𝑝1 𝑅𝑇 𝑝2 4) None of these
1) log 2) log 3) log
2𝐹 𝑝2 𝐹 𝑝2 𝐹 𝑝1
43. 2+ 2+
The cell, Zn | Zn (1 M) || Cu (1 M) | Cu
°
(𝐸cell = 1.10 V), was allowed to be completely discharged at 298 K. The relative concentration of
[Zn2+ ]
Zn2+ to Cu2+ ([Cu2+ ]) is
1) Antilog (24.08) 2) Antilog (37.3) 3) 1037.3 4) 9.65 × 104
44. For gold plating, the electrolyte used is
1) AuCl3 2) HAuCl4 3) K[Au(CN)2 ] 4) None of these
45. Out of Cu, Al , Fe and Zn, metal which can displace all others from their salt solution is
1) Al 2) Cu 3) Zn 4) Fe
46. The electroplating with chromium is undertaken because :
1) Electrolysis of chromium is easier
2) Chromium can form alloys with other metals
3) Chromium gives a protective and decorative coating to the base metal
4) Of high reactivity of chromium metal
47. The standard redox potentials for the reactions
Mn2+ + 2𝑒 − ⟶ and Mn3+ + 𝑒 − ⟶ M 2+ are -1.18 V and 1.51 V respectively. What is the redoxpotenital
for the reaction
Mn3+ + 3𝑒 − ⟶ Mn ?
1) 0.33 V 2) 1.69 V 3) -0.28 V 4) - 0.85
48. The molar conductivities Λ°NaOAc and Λ°HCl at infinite dilution in water at 25℃ are 91.0 and 426.2 S
cm2 /mol respectively. To calculate Λ°HOAc, the additional value required is
1) Λ°H2 O 2) Λ°KCl 3) Λ°NaOH 4) Λ°NaCl
49. The desired amount of charge for obtaining one mole of Al from Al is 3+

1) 96500 C 96500
2) 2 × 96500 C 3) 3 × 96500 C 4) C
2
50. When a copper wire is immersed in a solution of AgNO3 , the colour of the solution becomes blue because
copper :
1) Forms a soluble complex with AgNO3
2) Is oxidised to Cu2+
3) Is reduced to Cu2−
4) Splits up into atomic form and dissolves
51. Which of the following does not conduct electricity?
1) Fused NaCl 2) Solid NaCl 3) Brine solution 4) Copper
52. The fraction of the total current carried by an ion is known as:
1) Transport number of that ion
2) Conductance of that ion
3) Both(a) and (b)
4) None of the above
Page|4
53. In the problem 15, the limiting mobility of K + ion is :
1) 6.1 × 10−4 2) 6.67 × 10−4 3) 7.1 × 10−4 4) 7.67 × 10−4
54. Which are used as secondary reference electrodes?
1) Calomel electrode
2) Ag/AgCl electrode
3) Hg/Hg 2 Cl2 − KCl electrode
4) All of the above
55. The conductivity of a 0.01 N solution is found to be 0.005 ohm−1 cm−1 .The
equivalent conductivity of the solution will be
1) 5 × 10−2 ohm−1 cm2 equiv −1 2) 5.00 × 10−3 ohm−1 cm2
3) 500 ohm−1 cm−2 equiv −1 4) 0.5 ohm−1 cm2 equiv −1
56. When an aqueous solution of lithium chloride is electrolysed using graphite electrodes :
1) pH of the resulting solution increases
2) pH of the resulting solution decreases
3) As the current flows, pH of the solution around the cathode increases
4) None of the above
57. Deduce from the following 𝐸° values of half cells, what combination of two half cells would result in a cell
with the largest potential?
i) 𝐴3− ⟶ 𝐴2− + 𝑒; 𝐸° = 1.5 V
ii) 𝐵2+ + 𝑒 ⟶ 𝐵+ ; 𝐸° = −2.1 V
2+ +
iii) 𝐶 + 𝑒 ⟶ 𝐶 ; 𝐸° = +0.5 V
2+
iv) 𝐷 ⟶ 𝐷 + 2𝑒; 𝐸° = −1.5 V
1) (i) and (ii) 2) (i) and (iv) 3) (ii) and (iv) 4) (iii) and (iv)
58. The same amount of electricity was passed through two cells containing molten Al2 O3 and molten NaCl. If
1.8 g of Al were liberated in one cell, the amount of Na liberated in the other cell is :
1) 4.6 g 2) 2.3 g 3) 6.4 g 4) 3.2 g
59. The main function of the salt bridge is :
1) To allow ions to go from one cell to another
2) To provide link between two half cells
3) To keep the e.m.f. of the cell positive
4) To maintain electrical neutrality of the solution in two half cells
60. Standard electrode potential data are useful for understanding the suitability of an oxidant in a redox
titration. Some half cell reactions and their standard potentials are given below ;
MnO− +
4 (𝑎𝑞) + 8H (𝑎𝑞) + 5𝑒


2+ (𝑎𝑞)
Mn + 4H2 O; 𝐸° = 1.51 𝑉
2− (𝑎𝑞)
Cr2 O7 + 14H + (𝑎𝑞) + 6𝑒 − ⟶
2Cr 3+ (𝑎𝑞) + 7H2 O; 𝐸° = 1.38 𝑉
Fe3+ (𝑎𝑞) + 𝑒 − ⟶ Fe2+ (𝑎𝑞); 𝐸° = 0.77 𝑉
Cl2 (g) + 2𝑒 − ⟶ 2Cl− (𝑎𝑞); 𝐸° = 1.40 𝑉
Identify the only incorrect statement regarding the quantitative estimation of aqueous Fe(NO3 )2:
1) MnO− 4 can be used in aqueous HCl
2) Cr2 O2−7 can be used in aqueous HCl

3) MnO4 can be used in aqueous H2 SO4
4) Cr2 O2−7 can be used in aqueous H2 SO4
61. The 𝐸°𝑀3+/𝑀2+ values for Cr, Mn, Fe and Co are −0.41, +1.57, +0.77and+1.97 V respectively. For which
one of these metals, the change in oxidation state from +2 to +3 is easiest?
1) Fe 2) Mn 3) Co 4) Cr
62. Resistance of a conductivity cell filled with a solution of an electrolyte of concentration 0.1 M is 100Ω. The
conductivity of this solution is 1.29 S m−1 . Resistance of the same cell when filled with 0.2 M of the same
solution is 520Ω. The molar conductivity of 0.02 M solution of the electrolyte will be

Page|5
1) 124 × 10−4 S m2 mol−1 2) 1240 × 10−4 S m2 mol−1
3) 1.24 × 10 S m mol
−4 2 −1 4) 12.4 × 10−4 S m2 mol−1
63. Which process involves corrosion?
1) Brown deposits on iron articles
2) Green deposits on battery terminals
3) Black deposits on silver coin
4) All of the above
64. If the ∆𝐺 ° of Acell reaction,
AgCl + 𝑒 − → Ag + + Cl− 𝑖𝑠 − 21.20 kJ;
The standard emf of the cell is
1) 0.220 V 2) -0.220 V 3) 0.229 V 4) -0.110 V
65. During electrolysis of H2 O, the molar ratio of H2 and O2 formed is :
1) 2 : 1 2) 1 : 2 3) 1 : 3 4) 1 : 1
66. At 25℃ temperature, the cell potential of a given electrochemical cell is 1.92 V. Find the value of 𝑥.
Mg (𝑠) | Mg 2+ (𝑎𝑞)𝑥M || Fe2+ (𝑎𝑞) 0.01 M | Fe (𝑠)
𝐸 ° Mg /Mg 2+ (𝑎𝑞) = 2.37 𝑉𝐸 ° Fe/ Fe2+ (𝑎𝑞) = 0.45 𝑉
1) 𝑥 = 0.01 𝑀 2) 𝑥 < 0.01 𝑀
3) 𝑥 > 0.01 𝑀 4) 𝑥 cannot be predicted
67. In Ahydrogen-oxygen fuel cell, combustion of hydrogen occurs to
1) Generate heat
2) Create potential difference between the two electrodes
3) Produce high purity water
4) Remove adsorbed oxygen from electrode surfaces.
68. 9.65 C electric current is passed through fused anhydrous MgCl2 . The magnesium metal thus obtained is
completely converted into a Grignard reagent. The number of moles of Grignard reagent obtained is
1) 5 × 10−4 2) 1 × 10−4 3) 5 × 10−5 4) 1 × 10−5
69. The cell reaction for the given cell is :
Pt(H2 ) pH = 2pH = 3 Pt(H2 )
𝑃1 =1 atm 𝑃2 =1 atm
1) Spontaneous 2) Non-spontaneous 3) In equilibrium 4) Either of these
70. How long (in hours) must a current of 5.0 A be maintained to electroplate 60 g of calcium from molten
CaCl2?
1) 27 h 2) 8.3 h 3) 11 h 4) 16 h
71. The standard emf of a galvanic cell involving 2 moles of electrons in its redox reaction is 0.59 V. The
equilibrium constant for the redox reaction of the cell is
1) 1020 2) 105 3) 10 4) 1010
72. An electrolytic cell contains a solution of Ag 2 SO4 and platinum electrodes. A current is passed until 1.6 g of
O2 has been liberated at anode. The amount of Ag deposited at cathode would be:
1) 1.6 g 2) 0.8 g 3) 21.6 g 4) 107.88 g
73. Given the standard reduction potentials
Zn2+ /Zn = −0.74 𝑉, Cl2 / Cl− = 1.36 𝑉
1
H + / H2 = 0 𝑉 and Fe2+ / Fe3+ = 0.77 𝑉
2
The order of increasing strength as reducing agent is
1) Cl− , Zn , H2 , Fe2+ 2) H2 , Zn, Fe2+ , Cl− 3) Cl− , Fe2+ , Zn , H2 4) Cl− , Fe2+ , H2 , Zn
74. The position of some metals in the electrochemical series in decreasing electropositive character is given
as Mg > 𝐴𝑙 > 𝑍𝑛 > 𝐶𝑢 > 𝐴𝑔. What will happen, if a copper spoon is used to stir a solution of aluminium
nitrate?
1) The spoon will get coated with aluminium
2) An alloy of copper and aluminium is formed
3) The solution becomes blue

Page|6
4) There is no reaction
75. Chlorine gas is passed into a solution containing KF, KI, and KBr and CHCl3 is added. The initial colour in
CHCl3layer is :
1) Violet due to formation of I2
2) Orange due to formation of Br2
3) Colourless due to formation of F2
4) No colour change due to no reaction
76. Which of the following compounds will not undergo decomposition on passing electricity through aqueous
solution?
1) Sugar 2) Sodium acetate 3) Sodium chloride 4) Sodium bromide
77. ∞
Ionic mobility (𝑢 ) of an ion at infinite dilution is related to its ionic conductance (𝜆∞ ) by :
1) λ∞ = u∞ × Faraday 2) u∞ = λ∞ × Faraday 3) Faraday = u∞ × λ∞ 4) None of these
78. 2+ 2+
Zn | Zn (A = 0.1 M) || Fe (A = 0.01 M) | Fe.
The emf of the above cell is 0.2905 V. Equilibrium constant for the cell reaction is
1) 100.32/0.0591 2) 100.32/0.0295 3) 100.26/0.0295 4) 𝑒 0.32/0.0295
79. The value of Λ 𝑒𝑞 for NH4 Cl, NaOH and NaCl are respectively,149.74,248.1 and 126.4Ω−1 cm2 equiv −1 . The

value of Λ∞𝑒𝑞 of NH4 OH is


1) 371.44 2) 271.44
3) 71.44 4) Cannot be predicted from given data
80. The potential of the cell for the reaction,
𝑀 (s) + 2H + (1𝑀) ⟶ H2 (g) (1 atm) + 𝑀2+ (0.1 M)’
is 1.500 V. The standard reduction potential for 𝑀+ / 𝑀 (𝑠) couple is
1) 0.1470 V 2) 1.470 V 3) 14.70 V 4) None of these
81. The conductance of all the ions present in a solution containing 1 g equivalent in it is known as :
1) Conductivity
2) Equivalent conductivity
3) Molecular conductivity
4) None of the above
82. The value of molar conductance of HCl is greater than that of NaCl at a particular temperature and dilution
because :
1) mol. wt. ofHCl < mol. wt. of NaCl
2) 𝑢H+ > 𝑢Na+ (𝑢 is speed of ion)
3) HCl is acid
4) Ionisation of HCl is more than NaCl
83. Pure water does not conduct electricity because it
1) Is neutral 2) Is readily decomposed
3) Is almost totally unionized 4) Has a low boiling point
84. The reduction potential of hydrogen half-cell will negative if
1) 𝑝(𝐻2 ) = 1 atm and [H + ] = 2.0 M 2) 𝑝(𝐻2 ) = 1 atm and [H + ] = 1.0 M
3) 𝑝(𝐻2 ) = 2 atm and [H + ] = 1.0 M 4) 𝑝(𝐻2 ) = 2 atm and [H + ] = 2.0 M
85. The correct order 𝐸𝑀 °
2+ /𝑀 values with negative sign for the four successive elements Cr, Mn, Fe and Co is :

1) Cr >Mn> Fe > Co 2) Mn> Cr > Fe > Co 3) Cr > Fe >Mn> Co 4) Fe >Mn> Cr > Co


86. Which does not get oxidised by bromine water?
1) Fe2+ to Fe3+ 2) Cu+ to Cu2+ 3) Mn2+ to MnO−
4 4) Sn2+ to Sn4+
87. The standard oxidation potentials, 𝐸° for the half reactions are ;
Zn ⟶ Zn2+ + 2𝑒; 𝐸° = +0.76 V
Ag ⟶ Ag + + 𝑒; 𝐸° = −0.77 V
The standard e. m. f. of the cell,
Ag + + Zn ⟶ Zn2+ + Agis :
1) + 1.53 V 2) − 1.53 V 3) −0.01 V 4) + 0.01 V

Page|7
88. The ionic mobility of alkali metal ions in aqueous solution is maximum for :
1) K + 2) Rb+ 3) Li+ 4) Na+
89. The emf of the cell,
Ag | Ag + (0.1 M) || Ag + (1 M) | Ag at 298 K is
1) 0.0059 V 2) 0.059 V 3) 5.9 V 4) 0.59 V
90. Ionic mobility is equal to:
1) Speed of ions
2) Speed of ion under potential difference of 1 volt
3) Speed of ions under unit potential gradient
4) None of the above
91. A gas 𝑋 at 1 atm is bubbled through a solution containing a mixture of 1 M 𝑦 − and 1 M 𝓏 − at 25℃. If the
order of reduction potential is 𝓏 > 𝑦 > 𝑥 then
1) 𝑦 will oxidize 𝑥 and not 𝓏 2) 𝑦 will oxidize 𝑥 and 𝓏
3) 𝑦 will oxidize 𝓏 and not 𝑥 4) 𝑦 will reduce both 𝑥 and 𝓏
92. Sn4+ + 3𝑒 − ⟶ Sn2+, °
𝐸 = 0.13 𝑉
Br2 + 2𝑒 − ⟶ 2Br − , 𝐸 ° = 1.08 𝑉
Calculate 𝐾eq for the cell reaction for the cell formed by two electrodes.
1) 1041 2) 1032 3) 10−32 4) 10−42
93. Standard reduction potentials of the half reactions are given below :
F2 (g) + 2𝑒 − ⟶ 2F − (𝑎𝑞); 𝐸° = +2.85 V
Cl2 (g) + 2𝑒 − ⟶ 2Cl− (𝑎𝑞); 𝐸° = +1.36 V
− −
Br2 (𝑙) + 2𝑒 ⟶ 2Br (𝑎𝑞); 𝐸° = +1.06 V
− −
I2 (𝑠) + 2e ⟶ 2I (𝑎𝑞); 𝐸° = +0.53 𝑉
The strongest oxidising and reducing agents respectively are :
1) F2 and I − 2) Br2 and Cl− 3) Cl2 and Br − 4) Cl2 and I2
94. The standard electrode potential for the change ;
Sn(𝑠) + 2Fe3+ (𝑎𝑞) ⟶ 2Fe2+ (𝑎𝑞) + Sn2+ (𝑎𝑞) is :
° °
(Given 𝐸Fe 3+ /Fe2+ = 0.77 V and 𝐸Sn2+ /Sn = ⎯ 0.14 V)

1) 0.63 V 2) 1.40 V 3) 0.91 V 4) 1.68 V


95. 2+
During electrolysis of an aqueous solution of Cu sulphate, 0.635 g of copper was deposited at cathode.
The amount of electricity consumed in coulomb is :
1) 1930 2) 3860 3) 9650 4) 4825
96. ° °
𝐸𝐶𝑢 = 0.34 𝑉 , 𝐸𝑍𝑛 = 0.76 𝑉.ADaniel cell contains 0.1 M ZnSO4 solution and 0.01 M CuSO4 solution at its
electrodes. EMF of the cell is
1) 1.10 V 2) 1.04 V 3) 1.16 V 4) 1.07 V
97. Which of the following statements is correct? Galvanic cell converts
1) Chemical energy into electrical energy
2) Electrical energy into chemical energy
3) Metal from its elemental state to the combined state
4) Electrolyte into individual ions
98. I2 (𝑠)| I − (0.1 M)half-cell is connected to a H + (𝑎𝑞)| H2 (1 bar)| Pt half-cell and emf is found to be 0.7714 V.
If 𝐸I°2 /I− = 0.535 V, find the pH ofH + /H2 half-cell
1) 1 2) 2 3) 3 4) 5
99. 𝐸𝐹𝑒 3+/𝐹𝑒 = −0.036 V , 𝐸𝐹𝑒 2+ /𝐹𝑒 = −0.439 V. The value of standard electrode potential for the charge,
° °

Fe3+ (𝑎𝑞) + 𝑒 − ⟶ Fe2+ (𝑎𝑞) will be


1) -0.072 V 2) 0.385 V 3) 0.770 V 4) -0.270 V
100. The one which decreases with dilution is
1) Molar conductance 2) Conductance
3) Specific Conductance 4) Equivalent conductance

Page|8
101. An electric current is passed through silver nitrate solution using silver electrodes. 10.79 g of silver was
found to be deposited on the cathode. If the same amount of electricity is passed through copper sulphate
solution using copper electrodes, the weight of copper deposited on the cathode is
1) 1.6 g 2) 2.3 g 3) 3.2 g 4) 6.4 g
102. On electrolysing a solution of dilute H2 SO4 between platinum electrodes, the gas evolved at the anode and
cathode are respectively :
1) SO2 and O2 2) SO3 and H2 3) O2 and H2 4) H2 and O2
103. The molar conductivity of HCl, NaCl and CH3 COONa are 425, 188, 96𝑆cm2 mol−1 at 298 K. The molar
conductivity of CH3 COOH at the same temperature is …. 𝑆cm2 mol−1.
1) 333 2) 451 3) 325 4) 550
104. Agalvanic cell with electrode potential of ‘A’ = +2.23 V and ‘B’ = -1.43 V. The value of 𝐸cell
°
is
1) 3.66 V 2) 0.80 V 3) -0.80 V 4) -3.66 V
105. In electrochemical corrosion of metals, the metal undergoing corrosion :
1) Acts as anode 2) Acts as cathode 3) Undergoes reduction 4) None of these
106. Consider the following disproportionation
2ClO− −
3 ⇌ ClO2 + ClO4

If the initial concentration of perchlorate ion is 0.1 M what it would be at equilibrium at 298 K?
° °
(𝐸ClO − / ClO− = 0.36 V and 𝐸ClO− / ClO− = 0.33 V)
4 3 3 2
1) 0.1 M 2) 0.05 M 3) 0.07 M 4) 0.19 M
107. Give the products available on the cathode and the anode respectively during the electrolysis of an
aqueous solution of MgSO4 between inert electrodes.
1) H2 (g) and O2 (g) 2) O2 (g) and H2 (g) 3) O2 (g) and Mg(s) 4) O2 (g) and SO2 (g)
108. The number of coulombs required for the deposition of 107.870 g silver is
1) 96500 2) 48250 3) 1 4) 10000
109. In which of the following pairs, the constants/ quantities are not mathematically related to each other?
1) Gibbs free energy and standard cell potential
2) Equilibrium constant and standard cell potential
3) Rate constant and activation energy
4) Rate constant and standard cell potential
110. On passing 3 A of electricity for 50 min, 1.8 g metal deposits. The equivalent mass of metal is
1) 9.3 2) 19.3 3) 38.3 4) 39.9
111. The standard electrode potentials of Ag / Ag is +0.80 V and Cu /Cu is +0.34 V. These electrodes are
+ +

connected throughAsalt bridge and if


1) Copper electrode acts as Acathode then 𝐸cell °
is +0.46 V
2) Silver electrode acts as anode then 𝐸cell is -0.34 V
°

3) Copper electrode acts as anode then 𝐸cell°


is +0.46 V
4) Silver electrode acts as Acathode then 𝐸cell is -0.34 V
°

112. On passing 0.1 F of electricity through aluminium metal deposited at cathode is (Al = 27)
1) 0.3 g 2) 0.6 g 3) 0.9 g 4) 1.2 g
113. The number of electrons involved in redox reactions when a faraday of electricity is passed through an
electrolyte in solution is :
1) 6 × 1023 2) 8 × 1019 3) 69500 4) 6 × 10−23
114. 𝐸 0 of an electrode is :
1) Extensive property 2) Constitutive property 3) Colligative property 4) Intensive property
115. The emf of a galvanic cell, with electrode potentials of silver = + 0.80 V and that of copper = + 0.34 V, is
1) + 0.46 V 2) + 0.66 V 3) + 0.86 V 4) − 0.66 V
116. Indicator electrode is :
1) SHE
2) Calomel electrode
3) Ag/AgCl electrode
Page|9
4) Quinhydrone electrode
117. In electrolysis of aqueous copper sulphate, the gas at anode and cathode are
1) O2 and H2 2) H2 and O2 3) SO2 and H2 4) SO3 and O2
118. Beryllium is placed above magnesium in the II group. Beryllium dust, therefore, when added to MgCl2
solution will :
1) Have no effect
2) Precipitate Mg metal
3) Precipitate MgO
4) Lead to dissolution of Be metal
119. The process of zinc-plating on iron sheet is known as
1) Annealing 2) Roasting 3) Galvanisation 4) Smelting
120. A certain metal fails to liberate H2 gas from a moderately conc. HCl solution. However, it displaces Ag from
AgNO3 solution. Which among the followings may it be?
1) Mg 2) Fe 3) Cu 4) Cd
121. Passage of 96500 coulomb of electricity liberates ….litre of O2 at NTP during electrolysis.
1) 5.6 2) 6.5 3) 22.2 4) 11.2
122. Rust is
1) FeO + Fe(OH)2 2) Fe2 O3
3) Fe2 O3 + Fe(OH)2 4) Fe2 O3 and Fe(OH)3
123. The time required to coat a metal surface of 80 cm with 5 × 10−3 cm thick layer of silver (density 1.05 g
2

cm−3 with the passage of 3A current through a silver nitrate solution is :


1) 115 sec 2) 125 sec 3) 135 sec 4) 145 sec
124. 1 faraday of electricity will liberate 1 g-atom of the metal from the solution of :
1) NaCl 2) BaCl2 3) CuSO4 4) AlCl3
125. For I2 + 2e ⟶ 2I , standard reduction potential = + 0.54 volt. For2Br ⟶ Br2 + 2e− , standard
− −

oxidation potential = − 1.09 volt.For Fe ⟶ Fe2+ + 2e− , stabdard oxidation potential = + 0.44 volt. Which
of the following reactions is non-spontaneous?
1) Br2 + 2I − ⟶ 2Br − + I2
2) Fe + Br2 ⟶ Fe2+ + 2Br −
3) Fe + I2 ⟶ Fe2+ + 2I −
4) I2 + 2Br − ⟶ 2I − + Br2
126. A conductivity cell has two platinum electrodes of 1.2 cm2 area, separated by a distance of 0.8 cm. The cell
constant is :
1) 0.66 cm−1 2) 1.5 cm−1 3) 0.96 cm−1 4) 0.66 cm
127. Copper sulphate solution does not react with
1) Zinc 2) Iron 3) Silver 4) All of these
128. Faraday’s law of electrolysis fails when :
1) Temperature is increased
2) Inert electrodes are used
3) A mixture of electrolytes is used
4) In none of the above cases
129. When electric current is passed through an ionic hydride in molten state
1) Hydrogen is liberated at anode 2) Hydrogen is liberated at cathode
3) No change takes place 4) Hydride ions migrates towards cathode
130. On passing electricity through dilute H2 SO4 solution the amount of substance liberated at the cathode and
anode are in the ratio :
1) 1 : 8 2) 8 : 1 3) 16 : 1 4) 1 : 16
131. Given,
𝐸°𝐶𝑟3+/𝐶𝑟 = 0.72 𝑉 , 𝐸°𝐹𝑒 2+ /𝐹𝑒 = 0.42 𝑉.
The potential for the cell

P a g e | 10
Cr/Cr 3+ (0.1 M) || Fe2+ (0.01 M) | Fe is
1) 0.26 V 2) 0.399 V 3) -0.339 V 4) -0.26 V
132. 1.8 g of metal were deposited by a current of 3 amperes for 50 minute. The equivalent wt. of metal is :
1) 20.5 2) 25.8 3) 19.3 4) 30.7
133. The equivalent conductivity of 0.05 𝑁 solution of a monobasic acid is 15.8 mho cm2 eq −1.If equivalent
conductivity of theacid at infinite dilution is 350 mho cm2 eq −1 at the same temperature. What is its
degree of dissociation?
1) 0.04514 2) 0.4514 3) 4.514 4) 0.004514
134. The conductance of 1 cm3 of a solution is known as its :
1) Resistance
2) Conductivity
3) Equivalent conductivity
4) Molecular conductivity
135. Given that 𝐸 ° 3+ °
and 𝐸Fe °
3+ Fe are −0.36 V and − 0.439 V, respectively. The value of 𝐸Fe3+ ,Fe2+ Pt would
Fe Fe
be :
1) (−36 − 0.439)V
2) [3(−0.36) + 2(−0.439)]V
3) (−0.36 + 0.439)V
4) [3(−0.36) − 2(−0.439)]V
136. Ionic mobility ofAg + at infinite dilution is :
(𝜆0Ag+ = 5 × 10−3 𝑆 m2 eq−1 )
1) 5.2 × 10−8 2) 2.4 × 10−8 3) 1.52 × 10−8 4) 8.25 × 10−8
137. A cell in which electric current is produced by net oxidation and reduction process is called :
1) Voltaic cell 2) Electrolytic cell 3) Concentration cell 4) None of these
138. The standard reduction potential 𝐸 for the half reactions are as
°

Zn → Zn2+ + 2𝑒 − , 𝐸 ° = 0.76 𝑉
Cu → Cu2+ + 2𝑒 − , 𝐸 ° = 0.34 𝑉
The emf for the cell reaction,
Zn + Cu2+ → Zn2+ + Cu
1) 0.42 V 2) -0.42 V 3) -1.1 V 4) 1.1 V
139. The 𝐸° for OCl /Cl and Cl / Cl2 are 0.94 V and − 1.36 𝑉; 𝐸° for OCl / Cl2 is :
− − − 1 − 1
2 2
1) ⎯ 0.42 V 2) ⎯ 2.20 V 3) 0.52 V 4) 1.04 V
140. Calculate the equilibrium constant for the reaction, at 25℃
Cu (𝑠) + 2Ag + (𝑎𝑞) → Cu2+ (𝑎𝑞) + 2Ag (𝑠)
°
at 25 ℃ , 𝐸cell = 0.47 V , R = 8.314 JK −1
F = 96500 C is
1) 1.8 × 1015 2) 8.5 × 1015 3) 1.8 × 1010 4) 85 × 1015
141. In the electrolysis of which solution, OH − ions are discharged in preference to Cl− ions?
1) Dilute NaCl 2) Very dilute NaCl 3) Fused NaCl 4) Solid NaCl
142. MnO4 + 8H + 5𝑒 → Mn + 4H2 O; 𝐸° = 1.51 V
− + − 2+

MnO4 + 4H + + 2𝑒 − → Mn2++ 2H2 O; 𝐸° = 1.23 V


°
𝐸MnO − |MnO is
4 2
1) 1.70 V 2) 0.91 V 3) 1.37 V 4) 0.548 V
143. In the electrochemical reaction,
2Fe3+ + Zn → Zn2+ + 2 Fe2+
increasing the concentration of Fe2+
1) Increases cell emf 2) Increases the current flow
3) Decreases the cell emf 4) Alter the pH of the solution
144. The reduction electrode potential, E of 0.1 M solution of 𝑀+ ions

P a g e | 11
(𝐸𝑅𝑃 = −2.36 𝑉) is
1) -4.82 V 2) -2.41 V 3) +2.41 V 4) None of these
145. If 1 faraday of charge is passed through a solution of CuSO4 , the amount of copper deposited will be equal
to its :
1) Gram equivalent weight
2) Gram molecular weight
3) Atomic weight
4) Electrochemical equivalent
146. The limiting molar conductivities of NaCl, KBr and KCl are 126, 152 and 150
S cm2 mol−1 respectively. The Λ̊ for NaBr is :
1) 302 S cm2 mol−1 2) 176 S cm2 mol−1 3) 278 S cm2 mol−1 4) 128 S cm2 mol−1
147. For an electrolyte 𝐴𝑥𝐵𝑦, the molar conductivity at infinite dilution can be given by :
1) ΛM = 𝑥𝜆 𝐴𝑌+ + 𝑦𝜆 𝐵 𝑋−
1 1
2) ΛM = 𝜆 𝐴𝑌+ + 𝜆 𝐵 𝑋−
𝑥 𝑦
1 1
3) ΛM = 𝜆 𝐴𝑌+ + 𝜆 𝐵 𝑋−
𝑦 𝑥
 
4) Λ M = 𝜆 𝐴 + 𝜆 𝐵 𝑋−
𝑌+ 

148. 2Fe3+ + 3I − ⇌ 2Fe2+ + I3−


The standard reduction potentials in acidic conditions are 0.77 and 0.54 V respectively forFe3+ / Fe2+ and
I3− / I − couples. The equilibrium constant for the reaction is
1) 6.26 × 10−7 2) 5.33 × 10−4 3) 6.26 × 107 4) 5.33 × 104
149. The resistance of a decinormal solution of a salt occupying a volume between two platinum electrodes
1.80 cm apart and 5.4 cm2 in area was formed to be 32 ohm. The specific and equivalent conductivity
respectively in their proper units are :
1) 104.1 and 0.0104 2) 208.2 and 0.0208 3) 0.0104 and 104.0 4) None of these
150. The standard potentials at 25°C for the following half reactions are given ahead,
Zn2+ + 2e ⟶ Zn ; 𝐸° = −0.762 V
Mg 2+ + 2e ⟶ Mg; 𝐸° = −2.37 V
When zinc dust is added to the solution of MgCl2 :
1) ZnCl2 is formed
2) Zinc dissolves in the solution
3) No reaction takes place
4) Mg is precipitated
151. Astandard hydrogen electrode has zero electrode potential because
1) Hydrogen is easier to oxidise 2) This electrode potential is assumed to be zero
3) Hydrogen atom has only one electron 4) Hydrogen is the lightest element
152. The standard reduction potential for Fe |Fe and Sn2+ |Sn electrodes are −0.44 V and −0.14 V
2+

respectively. For the cell reaction, Fe2+ + Sn ⟶ Fe + Sn2+ , the standard e.m.f. is:
1) + 0.30 V 2) 0.58 V 3) + 0.58 V 4) − 0.30 V
153. In the problem 13, the dissociation constant of acid is :
1) 2.067 × 10−4 2) 1.02 × 10−4 3) 1.02 × 10−3 4) 1.02 × 10−5
154. Dipping iron article into a strongly alkaline solution of sodium phosphate
1) Does not affect the article 2) Forms Fe2 O3 . 𝑥H2 O on the surface
3) Forms iron phosphate film 4) Forms ferric hydroxide
155. The cathodic reaction of a dry cell is represented by
2MnO2 (𝑠) + Zn2 + 2𝑒 − ⟶ ZnMn2 O4 (𝑠)
If , there are 8 g of MnO2 in the cathodic compartment then the time for which the dry cell will continue to
give a current of 2 mA is
1) 25.675 day 2) 51.35 day 3) 12.8 day 4) 6.423 day

P a g e | 12
156. In which cell, electrical energy is converted into chemical energy?
1) Water voltameter 2) Silver voltameter 3) Coulometer 4) Either of these
157. The ionic conductance of Ba and Cl are respectively 127 and 76 Ω cm at infinite dilution. The
2+ − −1 2

equivalent conductance (in Ω−1 cm2) of BaCl2 at infinite dilution will be


1) 139.5 2) 203 3) 279 4) 101.5
158. On the basis of electrochemical theory of aqueous corrosion, the reaction occurring at the cathode is
1) O2 (g) + 4H + (𝑎𝑞) + 4𝑒 − → 2H2 O(𝑙) 2) H2 (g) + 2OH − (𝑎𝑞) → 2H2 O(𝑙) + 2𝑒 −
3) Fe(𝑠) → Fe2+ (𝑎𝑞) + 2𝑒 − 4) Fe2+ (𝑎𝑞) → Fe3+ (𝑎𝑞) + 𝑒 −
159. The factor temperature coefficient of e.m.f. is :
1) (𝜕𝐸/𝜕𝑇)𝑃 2) (𝜕𝐸/𝜕𝑇) 𝑇 3) (𝜕𝐸/𝜕𝑉) 𝑇 4) None of these
160. When electric current is passed through acidified water for 1930 s, 1120mL of H2 gas is collected (at STP)
at the cathode. What is the current passed in amperes?
1) 0.05 2) 0.50 3) 5.0 4) 50
161. A solution of sodium sulphate in water is electrolysed using inert electrodes. The products at the cathode
and anode are respectively
1) H2 , O2 2) O2 , H2 3) O2 , Na 4) O2 , SO2
162. The ionic conductivity of H and OH at 298 K are 349.8 and 198.5 mho cm2 eq −1 respectively. The
+ −

equivalent conductivity of H2 O at infinite dilution is :


1) 548.3 2) 151.3 3) 699.6 4) 54.83
163. The standard reduction potential for Fe /Fe andSn /Sn electrodes are – 0.44 and
2+ 2+
– 0.14 V
2+ 2+
respectively. For the given cell reaction, Fe + Sn → Fe + Sn , the standard emf is
1) 0.42 V 2) − 0.42 V 3) − 0.30 V 4) − 1.10 V
164. An electric current of 𝑐 ampere was passed through a solution of an electrolyte for ′𝑡′ second depositing 𝑃
g of the metal 𝑀 on the cathode. The equivalent weight 𝐸 of the metal will be :
𝑐 × 𝑡 𝑐 × 𝑃 96500 × 𝑃 𝑐 × 𝑡 × 9650
1) 𝐸 = 2) 𝐸 = 3) 𝐸 = 4) 𝐸 =
𝑃 × 96500 𝑡 × 96500 𝑐 ×𝑡 𝑃
165. Electrolysis of aq. Solution of LiCl shows :
1) pH < 7 2) pH = 7 3) pH > 7 4) No change
166. The equivalent conductivity of 0.1 𝑀 weak acid is 100 times less than that at infinite dilution. The degree
of dissociation of weak electrolyte at 0.1 𝑀 is :
1) 100 2) 10 3) 0.01 4) 0.001
167. Which will reduce zinc oxide to zinc?
1) Mg 2) Pb 3) Cu 4) Fe
168. Which process occurs in the electrolysis of aqueous solution of nickel chloride at nickel anode?
1) Ni2+ + 2𝑒 ⟶ Ni 2) 2H + + 2𝑒 ⟶ H2 3) 2Cl− ⟶ Cl2 + 2𝑒 4) Ni ⟶ Ni2+ + 2𝑒
169. At infinite dilution stage, the solution of CH3 COOH in water does not contain :
1) H + ion 2) CH3 COO− ion 3) CH3 COOH molecule 4) All of these
170. Electrolysis involves oxidation and reduction respectively at
1) Anode and cathode 2) Cathode and anode
3) At both the electrodes 4) None of these
171. Which of the following ions can be replaced by H inos when H2 gas is bubbled through the solutions
+

containing these ions?


1) Li+ 2) Ba2+ 3) Cu2+ 4) Be2+
172. The equivalent conductivity of a solution containing 2.54 g of CuSO4 per L is 91.0 Ω−1 cm2 eq−1 . Its
conductivity would be
1) 2.9 × 10−3 Ω−1 cm2 2) 1.8 × 10−2 Ω−1 cm2 3) 2.4 × 10−4 Ω−1 cm2 4) 3.6 × 10−3 Ω−1 cm2
173. The reaction at cathode during the electrolysis of aqueous solution of NaCl in Nelson cell is :
1) 2Cl− ⟶ Cl2 + 2𝑒
2) 2H + + 2𝑒 ⟶ H2
3) 2OH − ⟶ H2 + O2 + 2𝑒
4) Na+ + 𝑒 ⟶ Na
P a g e | 13
174. The amount of an ion discharged during electrolysis is not dependent of :
1) Resistance of solution
2) Time
3) Current strength
4) Electrochemical equivalent of the element
175. For the cell prepared from electrode 𝐴and𝐵; Electrode A ∶ Cr2 O2− 3+ °
7 | Cr , 𝐸red = +1.33 V and Electrode B :
°
Fe3+ |Fe2+ , 𝐸red = 0.77 V. Which of the following statement is correct?
1) The electrons will flow from 𝐵 to 𝐴 when connection is made
2) The e.m.f. of the cell will be 0.56 V
3) A will be positive electrode
4) All of the above
176. Saturated solution of KNO3 is used to make ‘salt-bridge’ because
1) Velocities of both K + and NO− 3 are nearly the same
2) Velocity of K + is greater than that of NO− 3
3) Velocity of NO− 3 is greater than that of K +

4) KNO3 is highly soluble in water


177. A current of strength 2.5 A was passed through CuSO4 solution for 6 min 26 s. The amount of copper
deposited is (At. Wt. of Cu = 63.5, 1F = 96500 C)
1) 0.3175 g 2) 3.175 g 3) 0.635 g 4) 6.35 g
178. The sum of the two transport number of ions for an electrolyte is always equal to :
1) 1 2) 2 3) 1/2 4) None of these
179. The atomic weight of Al is 27. When a current of 5 faraday is passed through a solution of Al3+ ions, the
wt.of Al deposited is :
1) 27 g 2) 36 g 3) 45 g 4) 9 g
180. When an electric current is passed through acidulated water, 112 mL of hydrogen gas at STP collects at the
cathode in 965 second. The current passed, in ampere is :
1) 1.0 2) 0.5 3) 0.1 4) 2.0
181. Which metal does not give the following reaction?
𝑀 + water or steam ⟶ oxide + H2 ↑
1) Iron 2) Sodium 3) Mercury 4) Magnesium
182. A current of strength 2.5 A was passed through CuSO4 solution for 6 min 26 s. The amount of copper
deposited is:
(Atomic weight of Cu = 63.5) (1 F = 96500 C)
1) 0.3175 g 2) 3.0175 g 3) 7.0135 g 4) 6.0275 g
183. The weight ratio of Al and Ag deposited using the same quantity of current is :
1) 9 : 108 2) 2 : 12 3) 108 : 9 4) 3 : 8
184. Galvanic cell is Adevice in which
1) Chemical energy is converted into electrical energy.
2) Electrical energy is converted into chemical energy.
3) Chemical energy is seen in the form of heat.
4) Thermal energy from an outside source is used to drive the cell reaction.
185. For which cell e.m.f. is independent of the concentration of electrolytes used?
1) Fe | FeO(𝑠)|KOH(𝑎𝑞)| NiO(𝑠)|Ni
2) Pt(H22)|OHCl |Pt(Cl2 )
3 (𝑠)|Ni 3) Zn|Zn(NO3 )2 || CuSO4 |Cu
4) Hg, HgCl2 | KCl ||AgNO3 |Ag
186. A certain quantity of electricity is passed through aqueous solution of AgNO3 and CuSO4 connected in
series. If Ag (at.wt.108) deposited at the cathode is 1.08 g then Cu deposited at the cathode is (at. wt. of Cu
is 63.53 ):
1) 6.354 g 2) 0.317 g 3) 0.6354 g 4) 3.177 g
187. For a cell reaction involving a two electron change, the standard emf of the cell is found to be 0.295 V at
25℃. The equilibrium constant of the reaction, at 25℃, will be
1) 10 2) 1 × 1010 3) 1 × 10−10 4) 10 × 10−2

P a g e | 14
188. 3 F electricity was passed through an aqueous solution of iron (II) bromide. The weight of iron metal (at.
Wt. = 56) deposited at the cathode (in g) is
1) 65 2) 84 3) 112 4) 168
189. Al2 O3 is reduced by electrolysis at low potentials and high currents. If 4.0 × 104 amperes of current is
passed through molten Al2 O3 for 6 hours, what mass of aluminium is produced? (Assume 100% current
efficiency. At mass of Al = 27 g mol−1 )
1) 1.3 × 104 g 2) 9.0 × 103 g 3) 8.05 × 104 g 4) 2.4 × 105 g
190. An ion is reduced to the element when it absorbs 6 × 1020 electrons. The number of equivalents of the ion
is :
1) 0.10 2) 0.01 3) 0.001 4) 0.0001
191. If the electrolyte used in problem 4 is Ba(NO3 )2 ,then molecular conductivity of solution is :
1) 628.56 mho−1 cm2 mol2) −1
628.56 mho cm2 mol−13) 6.28 mho cm2 mol−1 4) 6.28 mho−1 cm2 mol−1
192. Standard 𝐸° of the half cell Fe|Fe2+ is + 0.44 V and standard 𝐸° of half cell Cu | Cu2+ is −0.32 V then :
1) Cu oxidisesFe2+ ion 2) Cu2+ oxidises Fe 3) Cu reduces Fe2+ ion 4) Cu2+ reduces Fe
193. During the electrolysis of an electrolyte, the number of ions produced, is directly proportional to the
1) Time consumed 2) Mass of electrons
3) Quantity of electricity passed 4) Electrochemical equivalent of electrolytes
194. If the 𝐸cell for a given reaction has a negative value, then which of the following gives the correct
°

relationships for the value of ∆𝐺°and 𝐾eq?


1) ∆𝐺° > 0; 𝐾eq < 1 2) ∆𝐺° > 0; 𝐾eq > 1 3) ∆𝐺° < 0; 𝐾eq > 1 4) ∆𝐺° < 0; 𝐾eq < 1
195. Electrolytes, when dissolved in water, dissociate into their constituent ions. The degree of dissociation of a
weak electrolyte increases with
1) The presence of a substance yielding common ion
2) Decreasing temperature
3) Decreasing concentration of the electrolyte
4) Increasing concentration of the electrolyte
196. A current of 12 A is passed through an electrolytic cell containing aqueous NiSO4 solution. Both Ni and H2
gas are formed at the cathode. The current efficiency is 60%. What is the mass of nickel deposited on the
cathode per hour?
1) 7.883 g 2) 3.941 g 3) 5.91 g 4) 2.645 g
197. At 180 C the conductance of H + and CH3 COO− at infinite dilution are 315 and 35
mho cm2 eq−1 respectively. The equivalent conductivity ofCH3 COOH at infinite dilution is
……mho cm2 eq.−1 ∶
1) 350 2) 280 3) 30 4) 315
198. For the cell reaction Fe + 2Fe3+ = 3Fe2+ , which is not possible?
1) One cell can be constructed
2) Three different cells with different 𝐸cell
°
are possible
3) Three different cells with different number of electrons used in redox reaction are possible
4) Three different cells with same ∆𝐺° value are possible
199. The standard oxidation potentials of Zn and Ag in water at 25°C are,
Zn(𝑠) ⟶ Zn2+ + 2𝑒; 𝐸° = 0.76 V
Ag(𝑠) ⟶ Ag + + 𝑒; 𝐸° = −0.80 V
Which reaction actually takes place?
1) Zn(𝑠) + 2Ag + (𝑎𝑞) ⟶ Zn2+ Ag(𝑠)
2) Zn2+ + 2Ag + (𝑠) ⟶ 2Ag + (𝑎𝑞) + Zn(𝑠)
3) Zn(𝑠) + 2Ag(𝑠) ⟶ Zn2+ (𝑎𝑞) + Ag + (𝑎𝑞)
4) Zn2+ (𝑎𝑞) + Ag + (𝑎𝑞) ⟶ Zn(𝑠) + Ag(𝑠)
200. Given standard electrode potentials
𝐹𝑒 2+ + 2𝑒 − ⟶ 𝐹𝑒𝐸 ° = -0.440 V
𝐹𝑒 3+ + 3𝑒 − ⟶ 𝐹𝑒𝐸 ° = -0.036 V

P a g e | 15
The standard electrode potential (𝐸 ° ) for
Fe3+ + 𝑒 − ⟶ Fe2+ is
1) +0.772 V 2) -0.772 V 3) +0.417 V 4) -0.414 V
201. The standard reduction potentials at 298 K for the following half-cell reactions are given
Zn2+ (𝑎𝑞) + 2𝑒 − ⇌ Zn (𝑠); −0.762 𝑉
Cr 3+ (𝑎𝑞) + 3𝑒 − ⇌ Cr (𝑠); −0.74 𝑉
2H + (𝑎𝑞) + 2𝑒 − ⇌ H2 (𝑔); +0.00 𝑉
3+ (𝑎𝑞) − 2+ (𝑎𝑞);
Fe + 𝑒 ⇌ Fe +0.77 𝑉
Which one of the following is the strongest reducing agent?
1) Zn (s) 2) Cr(s) 3) H2 (s) 4) Fe2+ (aq)
202. Stainless steel does not rust because
1) Chromium and nickel combine with iron
2) Chromium forms an oxide layer and protects iron from rusting
3) Nickel present in it, does not rust
4) Iron forms Ahard chemical compound with chromium present in it
203. The amount of energy expanded during the passage of one ampere current for 100 second under a
potential of 115 V is :
1) 20 kJ 2) 11.5 kJ 3) 115 kJ 4) 0.115 kJ
204. Corrosion of iron is essentially an electrochemical phenomenon where the cell reactions are
Fe is oxidised to Fe2+ and dissolved oxugen in
1) 2) Fe is oxidised to Fe3+ and H2 O is reduced to O2−
2
water is reduced to OH −
Fe is oxidised to Fe2+ and H2 O is reduced Fe is oxidised to Fe2+ and H2 O is reduced
3) 4)
to O−2 to O2
205. The conductivity of 𝑁/50 solution of KCl in a cell at 250C is 0.002765 mho cm−1 . If the resistance of a cell
containing this solution is 400 ohm, the cell constant is :
1) 1.106 cm 2) 1.106 cm−1 3) 1 cm 4) 1 cm−1
206. Which represents a concentration cell?
PtH2 |HCl|| HCl |PtH2 PtH2 |HCl||Cl2 |Pt
1) 2) 3) Zn|Zn2+ ||Cu2+ |Cu 4) Fe|Fe2+ ||Cu2+ |Cu
𝑐1 𝑐2 𝑐1
207. Each of the three metals 𝑋, 𝑌 and 𝑍 were put in turn into aqueous solution of the other two.
𝑋 + Salt of 𝑌 (or 𝑍) = 𝑌 (or 𝑍) + Salt of 𝑋. Which observation is probably incorrect?
1) 𝑌 + Salt of 𝑋 = No action observed
2) 𝑌 + Salt of 𝑍 = Nothing can be decided
3) 𝑍 + Salt of 𝑋 = 𝑋 + Salt of 𝑍
4) 𝑍 + Salt of 𝑋 = No action observed
208. The electrolysis of a solution resulted in the formation of H2 at the cathode and Cl2 at the anode. The liquid
is:
1) Pure water
2) H2 SO4 solution
3) NaCl solution in water
4) CuCl2 solution in water
209. The variation of equivalent conductivity of weak electrolyte with √concentration is correctly shown in
figure :

1)  2)  3)  4)

210. The correct value of e.m.f. of cell is given by :


i) 𝐸cell = 𝐸OP anode − 𝐸RP cathode
ii) 𝐸cell = 𝐸OP anode + 𝐸RP cathode

P a g e | 16
iii) 𝐸cell = 𝐸RP anode + 𝐸RP cathode
iv) 𝐸cell = 𝐸OP anode − 𝐸OP cathode
1) (iii) and (i) 2) (i) and (ii) 3) (iii) and (iv) 4) (ii) and (iv)
211. The electrode potentials for
Cu2+ (𝑎𝑞) + 𝑒 − ⟶ Cu+ (𝑎𝑞)
andCu+ (𝑎𝑞) + 𝑒 − ⟶ Cu(𝑠)
°
are + 0.15 V and + 0.50 V respectively. The value of 𝐸Cu 2+ /Cu will be :

1) 0.150 V 2) 0.500 V 3) 0.325 V 4) 0.650 V


212. Same amount of electric current is passed through solutions of AgNO3 and HCl. If 1.08 g of silver is
obtained in the first case, the amount of hydrogen liberated at STP in the second case is
1) 224 cm3 2) 1.008 g 3) 112 cm3 4) 22400 cm3
213. For strong electrolytes the plot of molar conductance 𝑣𝑠√𝐶 is
1) Parabolic 2) Linear 3) Sinusoidal 4) Circular
214. What is the cell reaction occurring in Daniel cell (Galvanic cell)?
1) Cu(s) + ZnSO4 (aq) ⟶ CuSO4 (aq) + Zn(s)
2) Zn(s) + CuSO4 (aq) → Cu(s) + ZnSO4 (aq)
3) Ni(s) + ZnSO4 (aq) ⟶ NiSO4 (aq) + Zn(s)
4) 2Na(s) + CdSO4 (aq) ⟶ Na2 SO4 (aq) + Cd (s)
215. Which of the following statements (or equation) is correct?
1) The units of cell emf are V. cm−1
𝑛𝐹
2) ∆𝐺 = −
𝐸cell
3) In galvanic cell, chemical energy is transformed into electrical energy.
4) Oxidation state of Mn in potassium permanganate is +6
216. The conductance of a solution of an electrolyte is same as that of its conductivity. The cell used can be said
to have cell constant equal to :
1) 1 2) Zero 3) 100 4) 10
217. Which one of the following solutions will have highest conductivity?
1) 0.1 M CH3 COOH 2) 0.1 M NaCl 3) 0.1 M KNO3 4) 0.1 M HCl
218. Which gains electrons more easily?
1) H + 2) Na+ 3) K + 4) Mg 2+
219. Which of the following reactions is correct for a given electrochemical cell at 25 ℃?
Pt | Br2 (g) | Br − (g) || Cl− (𝑎𝑞) | Cl2 (g) | Pt
1) 2Br − (aq) + Cl2 (g) ⟶ 2Cl− (aq) + Br2 (g) 2) Br2 (g) + 2Cl− (aq) ⟶ 2Br − (aq) + Cl2 (g)
3) Br2 (g) + Cl2 (g) ⟶ 2Br − (aq) + 2Cl− (aq) 4) 2Br − (aq) + 2Cl− (aq) ⟶ Br2 (g) + Cl2 (g)
220. A silver cup is plated with silver by passing 965 C of electricity. The amount of Ag deposited is
1) 107.89 g 2) 9.89 g 3) 1.0002 g 4) 1.08 g
221. Reaction taking place at anode in dry cell is :
1) Zn2+ + 2𝑒 ⟶ Zn(𝑠) 2) Zn(𝑠) ⟶ Zn2+ + 2𝑒 3) Mn2+ + 2𝑒 ⟶ Mn(𝑠) 4) Mn(𝑠) ⟶ Mn2+ + 2𝑒
222. In a salt bridge, KCl is used because :
1) It is an electrolyte
2) It is good conductor of electricity
3) The transport number of K + and Cl− ions are nearly same or both have same ionic mobility.
4) It is ionic compound.
223. Calculate using appropriate molar conductance of the CH3 COOH from the molar conductances of
electrolytes listed below at infinite dilution in H2 O at 250C :
Electr KCl NaCl HCl NaOAc KNO3
ode
S Cm2 149.9 126.5 426.2 91.0 145.0
mol− 1
1) 51.2 2) 552.7 3) 390.7 4) 217.5

P a g e | 17
224. Lithium is generally used as an electrode in high energy density batteries. This is because:
1) Lithium is the lightest element
2) Lithium has quite high negative reduction potential
3) Lithium is quite reactive
4) Lithium does not corrode easily
225. Which of the following reactions is used to make a fuel cell?
1) 2H2 (g) + O2 (g) → 2H2 O(𝑙)
2) 2Fe(𝑠) + O2 (g) + 4H + (𝑎𝑞) → 2Fe2+ (𝑎𝑞) + 2H2 O(𝑙)
3) Pb(𝑠) + PbO2 (𝑠) + 2H2 SO4 (𝑎𝑞) → 2PbSO4 (𝑠) + H2 O(𝑙)
4) Cd(𝑠) + 2Ni(OH)3 (𝑠) → CdO(𝑠) + 2Ni(OH)2 + 2H2 O(𝑙)
226. Consider the following reaction :
2Ag + C6 H12 O6 + H2 O → 2Ag(𝑠) + C6 H12 O7 + 2H +
When ammonia is added to the solution, pH is raised to 11. Which half-cell reaction is affected by pH and
by how much?
°
Ag + + 𝑒 − → Ag; 𝐸red = 0.8 V
°
C6 H12 O6 + H2 O → C6 H12 O7 + 2H + + 2𝑒 − ; 𝐸red = −0.05 V
+ − °
Ag(NH3 )2 + 𝑒 → Ag(𝑠) + NH3 ; 𝐸red = 0.337 V
1) 𝐸oxid.
° °
will increase by a factor of 0.65 from 𝐸oxid. 2) 𝐸oxid.
° °
will decrease by a factor of 0.65 from 𝐸oxid.
3) 𝐸red.
° °
will increase by a factor of 0.65 from 𝐸red. 4) 𝐸red.
° °
will decrease by a factor of 0.65 from 𝐸red.
227. A cell with two electrodes, one of grey tin and the other white tin, both dipping in solution of (NH4 )2 SnCl6
showed zero e.m.f. at 18°𝐶. What conclusion may be drawn from this?
1) The e.m.f. developed at the electrode-solution phase boundary cancels the normal e.m.f.
2) Grey tin being non-metallic ceases to provide a reversible electrode reaction
3) Electrode surface develops a protective layer and the cell develops a very large internal resistance
4) The standard Gibbs energy change of the cell becomes zero
228. Molten NaCl conducts electricity due to the presence of :
1) Free electrons 2) Free molecules 3) Free ions 4) Atoms of Na and Cl
229. The number of electrons passing per second through a cross-section of Cu wire carrying 10 ampere is :
1) 6 × 1019 2) 8 × 1019 3) 1 × 1019 4) 1.6 × 1019
230. Consider the following 𝐸° values
𝐸°Fe3+/Fe2+ = +0.77 V
𝐸°Sn2+/Sn = − 0.14 V
Under standard conditions, the potential for the reaction
Sn(𝑠) + 2Fe3+ (𝑎𝑞) → 2Fe2+ (𝑎𝑞) + Sn2+ (𝑎𝑞) is
1) − 0.91 V 2) + 0.91 V 3) − 0.41 V 4) + 0.41 V
231. Electrolytes when dissolved in water dissociates into ions because
1) They are unstable
2) The water dissolves it
3) The force of repulsion increases
4) The force of electrostatic attraction are broken down by water
232. AgNO3 (𝑎𝑞) was added to an aqueous KCl solution gradually and the conductivity of the solution was
measured. The plot of conductance (Λ)𝑣𝑒𝑟𝑠𝑢𝑠 the volume of AgNO3 is

P a g e | 18
1) (P) 2) (Q) 3) (R) 4) (S)
233. The standard reduction potential of some electrodes are,
𝐸 0 (K + /K) = − 2.9 V,
𝐸 0 (Zn2+ /Zn) = −0.76 V, 𝐸 0 (H + /H2 ) = −0.00V,
𝐸 0 (Cu2+ /Cu) = +0.34 V
The Strongest oxidant is :
1) Copper 2) Zinc 3) Hydrogen 4) Cu2+
234. Which of the formula given below is correct?
1 1 1 4) All of these
1) 𝜅 = × 2) 𝜅 = 𝐶 × 3) 𝛬𝑒𝑞 = 𝜅 × 𝑉in mL
𝑅 𝑎 𝑎
235. Which aqueous solution will conduct an electric current quite well?
1) Glycerol 2) Sugar 3) Hydrochloric acid 4) Pure water
236. A lamp draws a current of 1.0 A. Find the charge in coulomb used by the lamp in 60 s.
1) 0.6 C 2) 60 C 3) 600 C 4) 0.006 C
237. Which one of the following solutions has highest conductance power?
1) 0.1 𝑀 CH3 COOH 2) 0.1 𝑀 NaCl 3) 0.1 𝑀 KNO3 4) 0.1 𝑀 HCl
238. Which one of the following nitrates will leave behind Ametal on strong heating?
1) Ferric nitrate 2) Copper nitrate 3) Manganese nitrate 4) Silver nitrate
239. For a given cell reaction; Cr + 3H2 O + OCl ⟶ Cr 3Cl + 6OH , the species undergoing reduction is :
− 3+ − −

1) Cr 2) Cr 6+ 3) OCl− 4) Cl−
240. Calculate the emf of the cell
Cu (𝑠) | Cu2+ (𝑎𝑞) || Ag + (𝑎𝑞) | Ag (𝑠)
Given,
° °
𝐸Cu 2+ /cu = + 0.34 𝑉 , 𝐸Ag+ /Ag = 0.80 𝑉 ,

1) +0.46 V 2) +1.14 V 3) +0.57 V 4) -0.46 V


241. Which substance is obtained in the solution on electrolysis of aqueous CuSO4 solution using graphite
electrodes?
1) H2 O 2) H2 SO4 3) Na2 SO4 4) Cu(OH)2
242. Standard electrode potential of NHE at 298 K is
1) 0.05 V 2) 0.10 V 3) 0.50 V 4) 0.00 V
243. Aluminium oxide may be electrolysed at 1000℃ to furnish aluminium metal (atomic mass = 27 u; 1 F =
96500 C). The cathode reaction is
Al3+ + 3𝑒 − ⟶ Al0
To prepare 5.12 kg of aluminium metal by this method would require
1) 5.49 × 101 C of electricity 2) 5.49 × 104 C of electricity
3) 1.83 × 107 C of electricity 4) 5.49 × 107 C of electricity
244. If a salt bridge is removed from the two half cells, the voltage
1) Drops to zero 2) Does not change 3) Increase gradually 4) Increase rapidly
245. If 3 F of electricity is passed through the solutions of AgNO3 , CuSO4 and AuCl3 , the molar ratio of the
cations deposited at the cathodes will be
1) 1:1:1 2) 1:2:3 3) 3:2:1 4) 6:3:2

P a g e | 19
246. When KMnO4 acts as an oxidizing agent and ultimately forms MnO2− 4 , MnO2 , Mn2 O3 andMn
2+
then the
number of electrons transferred in each case respectively, are
1) 4, 3, 1, 5 2) 1, 5, 3, 7 3) 1, 3, 4, 5 4) 3, 5, 7, 1
247. Four successive members of the first series of the transition metals are listed below. For which one of
°
them the standard potential (𝐸𝑀 2+ /𝑀 ) value has a positive sign?

1) Co (Z = 27) 2) Ni (Z = 28) 3) Cu (Z = 29) 4) Fe (Z = 26)


248. If the standard electrode potential of Cu / Cu electrode is 0.34 V, what is the electrode potential at 0.01
2+

M concentration of Cu2+?
(𝑇 = 298 𝐾)
1) 0.399 V 2) 0.281 V 3) 0.222 V 4) 0.176 V
249. Cu ion is not stable in aqueous solution because of disproportionation reaction. 𝐸 ° value for
+

° °
disproportionation of Cu+ is (given, 𝐸Cu 2+ / Cu+ = 0.15, 𝐸Cu2+ / Cu = 0.34 V)

1) + 0.38 V 2) − 0.38 V 3) + 0.49 V 4) − 0.49 V


250. For cell reaction,
Zn + Cu2+ ⟶ Zn2+ + Cu
Cell representation is
1) Zn | Zn2+ || Cu2+ | Cu 2) Cu | Cu2+ || Zn2+ | Zn
3) Cu | Zn2+ || Zn | Cu2+ 4) Cu2+ | Zn || Zn2+ | Cu
251. A cell necessarily does not contain :
1) An anode
2) A cathode
3) An electrolyte or a fuel
4) A porous diaphragm
252. Based on the data given below, the correct order of reducing power is :
Fe3+ (𝑎𝑞) + 𝑒 ⟶ Fe2+ (𝑎𝑞); 𝐸° = +0.77 V
Al3+ (𝑎𝑞) + 3𝑒 ⟶ Al(𝑠); 𝐸° = −1.66 V
Br2 (𝑎𝑞) + 2𝑒 ⟶ 2Br − (𝑎𝑞); 𝐸° = +1.08 V
1) Br − < Fe2+ < 𝐴𝑙 2) Fe2+ < 𝐴𝑙 < Br − 3) Al < Br − < Fe2+ 4) Al < Fe2+ < Br −
253. The electric conduction of a salt solution in water depends on the
1) Size of its molecules 2) Shape of its molecules
3) Size of solvent molecules 4) Extent of its ionization
254. 𝐸 values of Mg / Mg is -2.37 V, of Zn / Zn is -0.76 V and Fe2+ / Fe is -0.44 V.
° 2+ 2+

Which of the statements is correct?


1) Zn will reduce Fe2+ 2) Zn will reduce Mg 2+
3) Mg oxidises Fe 4) Zn oxidises Fe
255. Variation of equivalent conductivity with concentration of strong electrolyte is given by Hückel-Onsager
equation expressed as :
1) Λ 𝑀 = Λ∞ − 𝑏√𝑐 2) Λ ∞ = Λ𝑀 − 𝑏√𝑐 3) Λ M = 𝑏√𝑐 − Λ∞ 4) None of these
256. Cell constant of a conductivity cell is usually derived by using a solution of :
1) KCl 2) NaCl 3) NH4 Cl 4) LiCl
257. A depolariser used in dry cell batteries is :
1) Ammonium chloride 2) Manganese dioxide 3) Potassium hydroxide 4) Sodium phosphate
258. In a galvanic cell, which is wrong?
1) Anode has negative polarity
2) Cathode has positive polarity
3) Reduction takes place at anode
4) Reduction takes place at cathode
259. The potential of the following cell is 0.34 V at 25℃. Calculate the standard reduction potential of the
copper half-cell.
Pt | H2 (1 atm) | H + (1 M) || Cu2+ (1 M) | Cu
P a g e | 20
1) -3.4 V 2) +3.4 V 3) -0.34 V 4) +0.34 V
260. The standard emf for the given cell reaction, Zn + Cu → Cu + Zn is 1.10 V at 25℃. The emf for the cell
2+ 2+

reaction, when 0.1 M Cu2+ and 0.1 M Zn2+ solutions are used, at 25℃, is
1) 1.10 V 2) −1.10 V 3) 2.20 V 4) −2.20 V
261. The standard electrode potential of hydrogen electrode at 1 M concentration and hydrogen gas at 1atm
pressure is
1) 1 V 2) 6 V 3) 8 V 4) 0 V
262. Aluminium displaces hydrogen from dilute HCl whereas silver does not. The emf of Acell prepared by
combining Al/ Al3+ and Ag / Ag + is 2.46 V. The reduction potential of silver electrode is +0.80 V. The
reduction potential of aluminium electrode is
1) +1.66 V 2) -3.26 V 3) 3.26 V 4) -1.66 V
263. During electrolysis of water the volume of O2 liberated is 2.24dm3. The volume of hydrogen liberated,
under same conditions will be
1) 2.24 dm3 2) 1.12 dm3 3) 4.48 dm3 4) 0.56 dm3
264. Which of the following liberates hydrogen on reaction with dilute H2 SO4 ?
1) Al 2) Fe 3) Cu 4) Hg
265. 𝐸 for Fe + 2e → Fe is -0.44 V and 𝐸 for
° 2+ − °

Zn2+ + 2e− → Zn is -0.76 V thus


1) Zn is more electropositive than Fe 2) Zn is more electronegative than Fe
3) Fe is more electropositive than Zn 4) None of the above
266. The best way to prevent rusting of iron is
1) Making it cathode 2) Putting in saline water
3) Both (a) and (b) 4) None of these
267. The standard emf of the cell,
Cd (𝑠) | CdCl2 (𝑎𝑞) (0.1 M) || AgCl (𝑠) | Ag (𝑠)
In which the cell reaction is
Cd(𝑠) + 2AgCl (𝑠)
→ 2 Ag (𝑠) + Cd2+ (𝑎𝑞) + 2Cl− (𝑎𝑞)
is 0.6915 V at 0℃ and 0.6753 V at 25 ℃ . The enthalpy change of the reaction at 25℃ is
1) -176 kJ 2) -234.7 kJ 3) +123.5 kJ 4) -167.26kJ
268. Which metal is most readily corroded in moist air?
1) Copper 2) Iron 3) Silver 4) Nickel
269. The standard e.m.f. of a galvanic cell involving the cell reaction with 𝑛 = 2 is found to be 0.295 V at 25° C.
The equilibrium constant of the reaction is :
1) 2.0 × 1011 2) 4.0 × 1012 3) 1.0 × 102 4) 1.0 × 1010
270. SI unit of conductivity is :
1) ohm−1 cm−1 2) ohm−1 cm−1 or Sm−1 3) ohm m−1 4) ohm cm−1
271. On passing 1 F of electricity through the electrolytic cells containing Ag , Ni andCr 3+ ions solution, the
+ 2+

deposited Ag (at. wt. = 108), Ni (at. wt. = 59)and Cr (at. wt. = 52) is
Ag Ni Cr
1) 108 g 29.5 g 17.3 g 2) 108 g 59.5 g 52.0 g
3) 108 g 108 g 108 g 4) 108 g 117.5 g 166 g
272. By diluting a weak electrolyte , specific conductivity (𝐾𝑐 ) and equivalent conductivity (𝜆𝑐 ) change as
1) Both increase 2) 𝐾𝑐 increases , 𝜆𝑐 decreases
3) 𝐾𝑐 decreases , 𝜆𝑐 increases 4) Both decrease
273. 10 g atom of Ag can be oxidised to Ag during the electrolysis of AgNO3 solution using silver electrode
−2 +

by :
1) 965 coulomb 2) 96500 coulomb 3) 9650 coulomb 4) 96.500 coulomb
274. Consider the following 𝐸 ° values
𝐸°Fe3+/ Fe2+ = + 0.77 𝑉

P a g e | 21
𝐸°Sn2+/ Sn = − 0.14 𝑉
Under standard conditions the potential for the reaction
Sn (𝑠) + 2Fe3+ (𝑎𝑞) → 2Fe2+ (𝑎𝑞) + Sn2+ (𝑎𝑞) is
1) 1.68 V 2) 1.40 V 3) 0.91 V 4) 0.63 V
275. When same quantity of electricity is passed through aqueous AgNO3 and H2 SO4 solutions connected in
series, 5.04 × 10−2 g of H2 is liberated. What is the mass of silver (in grams) deposited? (Eq. wts.of
hydrogen = 1.008, silver = 108)
1) 54 2) 0.54 3) 5.4 4) 10.8
276. If the half-cell reaction 𝐴 + 𝑒 ⟶ 𝐴 has a large negative reduction potential, it follows that :

1) 𝐴 is readily reduced 2) 𝐴 is readily oxidized 3) 𝐴− is readily reduced 4) 𝐴− is readily oxidized


277. If mercury is used as cathode in the electrolysis of aqueous NaCl solution, the ions discharged at cathode
are :
1) H + 2) Na+ 3) OH − 4) Cl−
278. 1 volt coulomb is :
1) Equal to 1 joule 2) Equal to 107 erg 3) An unit of energy 4) All of these
279. The metal used to recover copper from Asolution of CuSO4 is
1) Fe 2) He 3) Na 4) Ag
280. The e.m.f. of the cell involving following changes,
Zn(𝑠) + Ni2+ (1𝑀) ⟶ Zn2+ (1𝑀) + Ni(𝑠)is 0.5105 V. The standard e.m.f. of the cell is :
1) 0.540 V 2) 0.4810 V 3) 0.5696 V 4) 0.5105 V
281. During the charging of lead storage battery, the reaction at anode is represented by :
1) Pb2+ + SO2− 4 ⟶ PbSO4
2) PbSO4 + H2 O ⟶ PbO2 + SO2− 4 + 2H
+

3) Pb ⟶ Pb2+ + 2𝑒
4) Pb2+ + 2𝑒 ⟶ Pb
282. 𝐸° for F2+ + 2𝑒 = 2F − is 2.8 V, 𝐸° for 1/2 F2 + 𝑒 = F − is:
1) 2.8 V 2) 1.4 V 3) − 2.8 V 4) − 1.4 V
283. Four alkali metals 𝐴, 𝐵, 𝐶 and 𝐷 are having respectively standard electrode potential as
−3.05, −1.66, −0.40 and 0.80. Which one will be the most reactive?
1) 𝐴 2) 𝐵 3) 𝐶 4) 𝐷
284. The weight of silver (eq.wt. = 108) displaced by that quantity of current which displaced 5600 mL of
hydrogen at STP is :
1) 54 g 2) 108 g 3) 5.4 g 4) None of these
285. TheΛ ofNH4 Cl, NaOH and NaCl are 129.8, 217.4 and 108.9 ohm cm eq.
∞ −1 2 −1
respectively. The 𝜆∞
−1 2 −1
ofNH4 OH is ……… ohm cm eq. .
1) 238.3 2) 218 3) 240 4) 260
286. The specific conductance of 0.1 N KCl solution at 23℃ is 0.012 ohm cm . The resistance of cell
−1 −1

containing the solution at the same temperature was found to be 55 ohm. The cell constant will be
1) 0.66 cm−1 2) 1.12 cm−1 3) 0.918 cm−1 4) 1.66 cm−1
287. When an acid cell is charged, then
1) Voltage of cell increases 2) Resistance of cell increases
3) Electrolyte of cell dilutes 4) None of the above
288. A certain current liberated 0.504 g of hydrogen in 2 h. How many grams of copper can be liberated by the
same current flowing for the same time in a copper sulphate solution?
1) 12.9 g 2) 15.9 g 3) 31.7 g 4) 36.9 g
289. The number of Faraday’s needed to reduce 4 g-equivalents of Cu to Cu metal will be
2+

1) 1 2) 2 3) 4 4) 8
290. Electrolytic conduction is due to migration of :
1) Protons 2) Electrons 3) Ions 4) All of these
291. The platinum electrodes were immersed in a solution of cupric sulphate and electric current was passed

P a g e | 22
through the solution. After some time, it was found that colour of copper sulphate disappeared with
evolution of gas at the electrode. The colourless solution contain
1) Copper sulphate 2) Copper hydroxide 3) Platinum sulphate 4) Sulphuric acid
292. The highest electrical conductivity of the following aqueous solutions is of
1) 0.1 M difluoroacetic acid 2) 0.1 M fluoroacetic acid
3) 0.1 M chloroacetic acid 4) 0.1 M acetic acid
293. In the electrolysis of water, 1 F of electrical energy would evolve
1) 1 mole of oxygen 2) 1 g atom of oxygen 3) 8 g of oxygen 4) 22.4 L of oxygen
294. KCl(𝑎𝑞) cannot be used as a salt bridge for the cell Cu(𝑠)|CuSO4 (𝑎𝑞)|| AgNO3 (𝑎𝑞)|Ag(𝑠) because :
1) CuCl2 is precipitated 2) Cl2 gas is given out 3) AgCl is precipitated 4) All of these
295. The standard potential at 25℃ for the following half-reactions are given against them Zn2+ + 2𝑒 − → Zn,
𝐸° = −0.762 V
Mg 2+ + 2𝑒 − → Mg, 𝐸° = −2.37 V
When zinc dust is added to the solution of MgCl2 ,
1) ZnCl2 is formed 2) Mg is precipitated
3) Zn dissolves in the solution 4) No reaction takes place
296. Metals can be prevented from rusting by :
1) Connecting iron to more electropositive metal, 𝑖. 𝑒., cathodic protection
2) Connecting iron to more electropositive metal, 𝑖. 𝑒., anodic protection
3) Connecting iron to less electropositive metal, 𝑖. 𝑒., anodic protection
4) Connecting iron to less electropositive metal, 𝑖. 𝑒., cathodic protection
297. Which one is correct about conductivity water?
1) The water whose own conductance is very small
2) The water obtained after 7-8 times distillation
3) Kohlrausch prepared the conductivity water for the first time
4) All of the above
298. Which defines the standard reduction electrode potential of Zn2+ ions?
1) Zn2+ (𝑎𝑞) + 2𝑒 ⟶ Zn(𝑠); [Zn2+ ] = 1𝑀
2) Zn(g) ⟶ Zn2+ + 2𝑒; [Zn2+ ] = 1𝑀
3) Zn (𝑎𝑞) ⟶ Zn(𝑠) + 2𝑒; [Zn2+ ] = 1𝑀
2+

4) Zn2+ (g) ⟶ Zn(𝑠) − 2𝑒; [Zn2+ ] = 1𝑀


299. The term infinite dilution refers when :
1) 𝛼 ⟶ 1, for weak electrolytes
2) An electrolyte is 100% dissociated
3) All interionic effects disappears
4) All of the above
300. An apparatus used for the measurement of quantity of electricity is known as a :
1) Calorimeter 2) Cathetometer 3) Coulometer 4) colorimeter
301. Resistance of 0.2 M solution of an electrolyte is 50 Ω . The specific conductance of the solution is 1.3 S m−1 .
If resistance of the 0.4M solution of the same electrolyte is 260Ω , its molar conductivity is
1) 6250 Sm2 mol−1 2) 6.25 × 10−4 Sm2 mol−1
3) 625 × 10−4 Sm2 mol−1 4) 62.5 Sm2 mol−1
302. The hydrogen electrode is dipped in Asolution of pH 3 at 25℃. The potential would be (the value of 2.303
RT /F is 0.059 V)
1) 0.177 V 2) 0.087 V 3) 0.059 V 4) -0.177 V
303. When during electrolysis of a solution of a AgNO3 , 9650 C of charge pass through the electroplating bath,
the mass of silver deposited on the cathode will be
1) 1.08 g 2) 10.8 g 3) 21.6 g 4) 108 g
304. Which of the following reactions cannot be Abase for electrochemical cell?
1) H2 + O2 ⟶ H2 O

P a g e | 23
2) AgNO3 + Zn → Zn (NO3 )2 + Ag
3) AgNO3 + NaCl → AgCl ↓ + NaNO2
KMnO4 + FeSO4 + H2 SO4 ⟶
4)
K 2 SO4 + Fe2 (SO4 )3 + MnSO4 + H2 O
305. How many electrons are there in one coulomb?
1) 6.02 × 1021 2) 6.24 × 1018 3) 6.24 × 1015 4) 6.02 × 1016
306. What flows in the internal circuit of Agalvanic cell?
1) Ions 2) Electrons 3) Electricity 4) Atoms
307. Which statement is not correct?
1) Conductance of an electrolytic solution increases with dilution
2) Conductance of an electrolytic solution decreases with dilution
3) Specific conductance of an electrolytic solution decreases with dilution
4) Equivalent conductance of an electrolytic solution increase with dilution
308. Standard free energies of formation (in kJ/mol) at 298 K are − 237.2, −394.4 and − 8.2 for H2 O(𝑙), CO2 (g)
and pentane (g), respectively. The value of 𝐸°cell for the pentane-oxygen fuel cell is :
1) 2.0968 V 2) 1.0968 V 3) 0.0968 V 4) 1.968 V
309. Cell constant has the unit:
1) cm 2) cm−1 3) cm2 4) cm sec −1
310. Cu (𝑎𝑞) is unstable in solution and undergoes simultaneous oxidation and reduction, according to the
+

reaction
2Cu+ (𝑎𝑞) ⇌ Cu2+ (𝑎𝑞) + Cu (𝑠)
choose correct 𝐸 ° for the above reaction if
° °
𝐸Cu 2+ /Cu = 0.34 V , 𝐸Cu2+ /Cu+ = 0.15 V

1) -0.38 V 2) +0.49 V 3) +0.38 V 4) -0.19 V


311. What is the quantity of electricity (in Coulombs) required to deposit all the silver from 250mL of 1
MAgNO3 solution?
1) 2412.5 2) 24125 3) 4825.0 4) 48250
312. What will be the electrode potential of that hydrogen electrode is filled with HCl solution of pH value 1.0?
1) -59.15 V 2) +59.15 3) +59.15 mV 4) -59.15 mV
313. The laws of electrolysis were proposed by
1) Kohlrausch 2) Faraday 3) Haber 4) Bergius
314. The apparatus in which electrical energy is converted into chemical energy is known as :
1) Voltameter 2) Coulometer 3) Both (a) and (b) 4) None of these
315. Which of the following electrolytic solutions has the least specific conductance?
1) 0.02 N 2) 0.2 N 3) 2 N 4) 0.002 N
316. The molar conductivity at infinite dilution of AgNO3 , NaCl and NaNO3 are 116.5, 110.3 and 105.2
mho cm2 mol−1 respectively. The molar conductivity of AgCl is :
1) 121.6 2) 111.4 3) 130.6 4) 150.2
317. Ionic mobility of of electricity is 1 𝑀 solution of :
1) CH3 COOH 2) H2 SO4 3) H3 PO4 4) Boric acid
318. The electrochemical cell stops working after sometimes because
1) Electrode potential of both the electrodes becomes zero
2) Electrode potential of both the electrodes becomes equal
3) One of the electrodes is eaten away
4) The cell reaction gets reversed
319. Zn2+ + 2𝑒 − → Zn(𝑠), 𝐸° = −0.76
Fe3+ + 𝑒 − → Fe2+ , 𝐸° = −0.77
Cr 3+ + 3𝑒 − → Cr, 𝐸° = −0.79
1
H + + 𝑒 − → H2 , 𝐸° = 0.00
2

P a g e | 24
Strongest reducing agent is
1) H2 2) Zn 3) Fe2+ 4) Cr
320. The weight ratio of Mg and Al deposited during the passage of same current through their molten salts :
1) 12 : 9 2) 9 : 12 3) 6 : 2 4) 2 : 3
321. The standard cell potential for the cell is : Zn | Zn2+ (1𝑀) || Cu2+ (1𝑀)|Cu
[𝐸° for Zn2+ /Zn = −0.76; 𝐸° for Cu2+ /Cu = +0.34]
1) − 0.76 + 0.34 = −0.42 V
2) − 0.34 − (− 0.76) = +0.42 V
3) 0.34 − (−0.76) = +1.10 V
4) −0.76 − (+0.34) = −1.10 V
322. Total charge on 1 mole of a monovalent metal ion is equal to :
1) 6.28 × 1018 coulomb 2) 1.6 × 10−19 coulomb 3) 9.65 × 104 coulomb 4) None of these
323. 1.05 g of lead ore containing impurity of Ag was dissolved in HNO3 and the volume was made 350 mL. An
Ag electrode was dipped in the solution. Pt(H2 )| H + (1 M)|| Ag + |Ag
°
The 𝐸cell is 0.503 V at 298 K. The percent of lead in the ore is (𝐸Ag+ |Ag = 0.80 V)

1) 0.033% 2) 0.050% 3) 0.066% 4) 0.13%


324. When 𝑋 amperes of current is passed through molten AlCl3 for 96.5 s. 0.09 g of aluminium is deposited.
What is the value of X?
1) 10 A 2) 20 A 3) 30 A 4) 40 A
325. If an iron rod is dipped in CuSO4 solution, then :
1) Blue colour of the solution turns red
2) Brown layer is deposited on iron rod
3) No change occurs in the colour of the solution
4) None of the above
326. The number of faraday required to liberate 1 mole of any element indicates :
1) Weight of element
2) Conductance of electrolyte
3) Charge on the ion of that element
4) None of the above
327. Which is the correct representation for Nernst equation?
0.059 [oxidant]
1) 𝐸𝑅𝑃 = 𝐸𝑅𝑃°
+ log
𝑛 [reductant]
0.059 [oxidant]
2) 𝐸𝑂𝑃 = 𝐸𝑂𝑃°
− log
𝑛 [reductant]
0.059 [reductant]
3) 𝐸𝑂𝑃 = 𝐸𝑂𝑃°
+ log
𝑛 [oxidant]
4) All of the above
328. Salts of 𝐴 (atomic weight 7), 𝐵 (atomic weight 27) and 𝐶 (atomic weight 48) were electrolyzed under
identical conditions using the same quantity of electricity. It was found that when 2.1 g of 𝐴 was deposited,
the weights of 𝐵 and 𝐶 deposited were 2.7 g and 7.2 g. The valencies of 𝐴, 𝐵 and 𝐶 are respectively :
1) 3,1 and 2 2) 1, 3 and 2 3) 3,1 and 3 4) 2,3 and 2
329. Cu(II)sulphate solution is treated separately with KCl and KI. In which case, Cu2+ be reduced to Cu+ ?
1) With KCl 2) With KI 3) With both (a) and (b) 4) None of these
330. A current is passed through two voltameters connected in series. The first voltameter contains 𝑋SO4 (𝑎𝑞)
while the second voltameter contains 𝑌2 SO4 (𝑎𝑞). The relative atomic masses ox 𝑋 and 𝑌 are in the ratio of
2 : 1. The ratio of the mass of 𝑋 liberated to the mass of 𝑌 liberated is :
1) 1 : 1 2) 1 : 2 3) 2 : 1 4) None of these
331. EMF of a cell in terms of reduction potential of its left and right electrodes is
1) 𝐸 = 𝐸left − 𝐸right 2) 𝐸 = 𝐸right − 𝐸left 3) 𝐸 = 𝐸left + 𝐸right 4) None of these
332. The standard reduction potential, 𝐸° for the half-reactions are as

P a g e | 25
Zn ⇌ Zn2+ + 2𝑒 − , 𝐸° =⊕ 0.76 V
Fe ⇌ Fe2+ + 2𝑒 − , 𝐸° = +0.41 V
The 𝐸°cell for the cell formed by these two electrodes is
1) −0.35 V 2) −1.17 V 3) +0.35 V 4) +1.17 V
333. How many coulomb of electricity are consumed when 100 mA current is passed through a solution of
AgNO3 for 30 minute during an electrolysis experiment?
1) 108 2) 18000 3) 180 4) 3000
334. The product obtained at anode when 50% H2 SO4 aqueous solution is electrolysed using platinum
electrodes is
1) H2 SO3 2) H2 S2 O8 3) O2 4) H2
335. The equivalent conductivity of two strong electrolytes at infinite dilution are :
Λ̊ CH3 COONa(aq.) = 91.0 S cm2 eq.−1
Λ̊ HCl(aq.) = 426.2 S cm2 eq.−1
What additional information one needs to calculate Λ̊ of an aqueous CH3 COOH?
1) Λ̊ofClCH2 COOH 2) Λ̊ofCH3 COOK 3) Λ̊ofNaCl 4) Λ̊of H +
336. Which is not true for a standard hydrogen electrode?
1) The hydrogen ion concentration is 1𝑀
2) Temperature is 25°C
3) Pressure of hydrogen is 1 atmosphere
4) It contains a metallic conductor which does not adsorb hydrogen
337. On passing 0.5 F electricity through molten sodium chloride, sodium deposited at cathode will be
1) 29.25 g 2) 11.50 g 3) 58.50 g 4) 0.00 g
338. The correct order of molar conductivity at infinite dilution of LiCl, NaCl and KCl is
1) LiCl > 𝐾𝐶𝑙 > 𝑁𝑎𝐶𝑙 2) KCl > 𝑁𝑎𝐶𝑙 > 𝐿𝑖𝐶𝑙 3) LiCl > 𝑁𝑎𝐶𝑙 > 𝐾𝐶𝑙 4) NaCl > 𝐾𝐶𝑙 > 𝐿𝑖𝐶𝑙
339. In the electrolysis of CuSO4 , the reaction Cu + 2e ⟶ Cu, takes place at :
2+ −

1) Anode 2) Cathode 3) In solution 4) None of these


340. During electrolysis of fused CaH2 , H2 is liberated at :
1) Anode 2) Cathode 3) Either electrode 4) Not at all
341. The standard electrode potential for the half – cell reactions are
Zn2+ + 2𝑒 − ⟶ Zn; 𝐸 ° = -0.76 V
Fe2+ + 2𝑒 − ⟶ Fe; 𝐸 ° = -0.44 V
The emf of the cell reaction,
Fe2+ + Zn ⟶ Zn2+ + Fe is
1) -0.32 V 2) -1.20 V 3) +1.20 V 4) +0.32 V
342. The value of electronic charge is equal to :
Faraday
1)
Av. number
2) Faraday × Av.number
Av. number
3)
Faraday
4) None of these
343. Standard electrode potentials of Fe2+ + 2𝑒 ⟶ Fe and Fe+ + 3𝑒 ⟶ Fe are ⎯ 440 V and ⎯0.036 V
respectively. The standard electrode potential (𝐸°) for Fe3+ + 𝑒 ⟶ Fe2+ is:
1) − 0.476 V 2) − 0.404 V 3) + 0.404 V 4) + 0.772 V
344. The oxidation number of S in Na2 S4 O6 is
1) 2.5 for each S atom
2) +2 and +3 (two S have +2 and other two have +3)
3) +2 and +3 (three S have +2 and one S has +3)
4) +5 and 0 (two S have +5 and other two have zero)
345. The standard emf for the cell reaction,

P a g e | 26
2Cu+ (𝑎𝑞) → 2Cu (𝑠) + Cu+ (𝑎𝑞)
is +0.36 V at 298 K. The equilibrium constant of the reaction is
1) 5 × 106 2) 1.4 × 1012 3) 7.4 × 1012 4) 1.2 × 106
346. Which of the following metal can replace zinc from ZnSO4 solution?
1) Cu 2) Hg 3) Fe 4) Al
347. If a strip of copper metal is placed in a solution of ferrous sulphate :
1) Copper will precipitate out
2) Iron will precipitate out
3) Both copper and iron will be dissolved
4) No reaction will take place
348. Which of the following is displaced by Fe?
1) Ag 2) Zn 3) Na 4) All of these
349. The amount of substance deposited by the passage of 1 A of current for 1 s is equal to
1) Equivalent mass 2) Molecular mass
3) Electrochemical equivalent 4) Specific equivalent
350. The metal that does not displace hydrogen from an acid is
1) Ca 2) Al 3) Zn 4) Hg
351. Two electrolytic cells, one containing acidified ferrous chloride and another acidified ferric chloride are
connected in series. The ratio of iron deposited at cathodes in the two cells when electricity is passed
through the cells will be :
1) 3 : 1 2) 2 : 1 3) 1 : 1 4) 3 : 2
352. During electrolysis of fused sodium chloride, the reaction of the electrodes are:
Anode Cathode
1
1) Na+ + 𝑒 ⟶ NaCl− ⟶ Cl + 𝑒
2
1
2) Na ⟶ Na+ + 𝑒 Cl2 + 𝑒 ⟶ Cl−
2
1
3) Cl− ⟶ Cl2 + 𝑒 Na+ + 𝑒 ⟶ Na
2
1
4) Cl2 + 𝑒 ⟶ Cl− Na ⟶ Na+ + 𝑒
2
353. In an electrolytic cell of Ag |AgNO3 | Ag, when current is passed, the concentration of AgNO3 :
1) Increases 2) Decreases 3) Remains same 4) None of these
354. The equivalent conductivity of KCl at infinite dilution is 130 mho cm eq . The transport number of Cl−
2 −1

ion in KCl at the same temperature is 0.505. The transport number of K + ion is :
1) 0.495 2) 0.505 3) 0.0495 4) Cannot be predicted
355. The standard reduction potential for the half-cell having reaction
NO− + −
3 (𝑎𝑞) + 2H (𝑎𝑞) + 𝑒 → NO2 (g) + H2 O
Is 0.78 V. What will be the reduction potential of the half-cell is a neutral solution?
1) 0.78 V 2) 0.89 V 3) 0.36 V 4) 0.59 V
356. The standard H electrode is written as :
1) Pt, H2 , H + (𝑎 = 1)
2) PtH2 /H + (𝑎 = 1)
3) PtH2 (g)(1 atm)/H + (𝑎 = 1)
4) None of the above
357. The Edison storage cell is represented as :
Fe(𝑠) + FeO(𝑠)| KOH(𝑎𝑞)| Ni2 O3 (𝑠)|Ni2 O3 (𝑠)| Ni(𝑠)
The half reactions are Ni2 O3 (𝑠) + H2 O(𝑙) + 2𝑒 − → 2NiO(𝑠) + 2OH − ; 𝐸° = +0.40 V
FeO(𝑠) + H2 O(𝑙) + 2𝑒 − → Fe(𝑠) + 2OH − ; 𝐸° = −0.87 V
Choose the incorrect statement
1) 𝐸anode increases with increase in concentration of OH −

P a g e | 27
2) 𝐸cathode decreases with increase in concentration of OH −
°
3) 𝐸cell = 1.27 V
4) 𝐸cell increases with increase in concentration of FeO
358. In an aqueous solution, hydrogen (H2 ) will not reduce :
1) Fe3+ 2) Cu2+ 3) Zn2+ 4) Ag +
359. The minimum equivalent conductance in fused state is shown by :
1) MgCl2 2) BeCl2 3) CaCl2 4) SrCl2
360. The cell reaction is spontaneous, when
1) 𝐸red
°
is negative 2) 𝐸red
°
is positive 3) Δ𝐺 ° is negative 4) Δ𝐺 ° is positive
361. The charge in coulomb on Cu2+ ion is :
1) 3.2 × 10−19 2) 2.3 × 10−12 3) 0.23 × 10−19 4) 0.32 × 10−19
362. The equilibrium constant for the reaction given below at 298 K is :
Zn(𝑠) + Fe2+ (𝑎𝑞) ⟶ Zn2+ (𝑎𝑞) + Fe(𝑠);
°
𝐸cell = 2905 V at 298 K
1) 𝑒 0.32/0.0295 2) 100.595/0.76 3) 100.0250/0.32 4) 100.32/0.295
363. The specific conductance (𝜅) of an electrolyte of 0.1 N concentration is related to equivalent conductance
(Λ) by the following formula
1) Λ = κ 2) Λ = 10κ 3) Λ = 100𝜅 4) Λ = 10000𝜅
364. Hydrogen-oxygen fuel cells are used in spacecraft to supply :
1) Power for heat and light
2) Power for pressure
3) Oxygen
4) None of the above
365. Limiting molar conductivity of NH4 OH, i.e.,Λ̊ m (NH4 OH) is equal to :
1) Λ̊ m (NH4 OH) + Λ̊ m (NH4 Cl) − Λ̊ m (HCl)
2) Λ̊ m (NH4 Cl) + Λ̊ m (NaOH) − Λ̊ m (NaCl)
3) Λ̊ m (NH4 Cl) + Λ̊ m (NaCl) − Λ̊ m (NaOH)
4) Λ̊ m (NaOH) + Λ̊ m (NaCl) − Λ̊ m (NH4 Cl)
366. Faraday’s laws hold good at :
1) All pressures 2) Only at 298 K 3) In different solvents 4) All of these
367. 20 g of chlorine are evolved in 6 hour from sodium chloride solution by the current of :
1) 5 ampere 2) 10 ampere 3) 2.5 ampere 4) 50 ampere
368. The Λ ofNH4 OH at infinite dilution is …. 𝑆 cm eq. .Given𝜆OH = 174, 𝜆∞
∞ 2 −1 ∞ − ∞
Cl− = 66 and 𝜆NH4 Cl =
130 S cm2 eq −1 ∶
1) 238 2) 218 3) 198 4) 160
369. Is the reaction, 2Al + 3Fe ⟶ 2Al + 3Fe possible?
2+ 3+

1) No, because standard oxidation potential of Al < 𝐹𝑒


2) Yes, because standard oxidation potential of Al > 𝐹𝑒
3) Neither (a) nor (b)
4) Data are unpredictable
370. The calomel electrode is a :
1) Standard hydrogen electrode
2) Reference electrode
3) Platinum electrode
4) Mercury electrode
371. A dilute solution of Li2 SO4 is electrolyzed. The products formed at the anode and cathode, respectively are
:
1) S and Li 2) O2 and Li 3) SO2 and O2 4) O2 and H2
372. When the sample of copper with zinc impurity is to be purified by electrolysis, the appropriate electrodes
are
P a g e | 28
Cathode Anode
1) Pure zinc pure copper
2) Impure sample pure copper
3) Impure zinc impure sample
4) Pure copper impure sample
373. How many faradays of electricity are required to electrolyse1 mole CuCl2 to copper metal and chlorine
gas?
1) 1 F 2) 2 F 3) 3 F 4) 4 F
374. Given the limiting molar conductivity as
⋀0m (HCl) = 425.9Ω−1 cm2 mol−1
⋀0m (NaCl) = 126.4Ω−1 cm2 mol−1
⋀0m (CH3 COONa) = 91Ω−1 cm2 mol−1
The molar conductivity , at infinite dilution, of acetic acid (in Ω−1 cm2 mol−1) will be
1) 481.5 2) 390.5 3) 299.5 4) 516.9
375. Strong electrolytes are those which
1) Conduct electricity 2) Dissolve readily in water
3) Dissociate into ions at high dilution 4) Completely dissociate into ions at all dilutions
376. Consider the following cell reaction
2Fe(𝑠) + O2 (𝑔) + 4H + (𝑎𝑞) ⟶
2Fe2+ (𝑎𝑞) + 2H2 O(𝑙), 𝐸° = 1.67 𝑉
At[Fe2+ ] = 10−3 M , P (O2 ) = 0.1 atm and pH = 3, the cell potential at 25℃ is
1) 1.47 V 2) 1.77 V 3) 1.87 V 4) 1.57 V
377. Use of electrolysis is not done in
1) Production of Na 2) Production of water
3) Purification of metals 4) Production of KOH
378. Conductivity (unit Siemen) is directly proportional to area of the vessel and the concentration of the
solution in it and is inversely proportional to the length of the vessel. Then, the units of the constant of
proportionality is
1) S 2 m2 mol 2) S 2 m2 mol−2 3) S m2 mol−1 4) S m mol−1
379. Which is correct about fuel cells?
1) Cells continuously run as long as fuels are supplied
2) These are more efficient and free from pollution
3) These are used to provide power and drinking water to astronauts in space programme
4) All of the above
380. Given :
i) Cu2+ + 2e− ⟶ Cu, 𝐸° = 0.337 V
ii) Cu2+ + e− ⟶ Cu+ , 𝐸° = 0.153 V
Electrode potential, 𝐸° for the reaction,
Cu+ + 𝑒 − ⟶ Cu, will be :
1) 0.38 V 2) 0.52 V 3) 0.90 V 4) 0.30 V
381. The cell reaction, Zn + Cu2+ ⟶ Zn2+ + Cu is best represented by :
1) Cu/Cu2+ || Zn2+ /Zn 2) Zn/Zn2+ ||Cu2+ /Cu 3) Cu2+ /Cu || Zn/Zn2+ 4) Pt/Zn2+ || Pt/Cu2+
382. Which one is correct?
1) Ni displaces zinc from its solution
2) Zn displaces iron from its solution
3) Ag displaces copper from its solution
4) Cu displaces nickel from its solution
383. For the cell,
T1 | T1+ (0.001 M) || Cu2+ (0.1 M) | Cu
𝐸cell at 25℃ is 0.83 V. 𝐸cell can be increased

P a g e | 29
1) By decreasing [Cu2+] 2) By increasing [Cu2+]
3) By increasing [T1 ]+
4) None of these
384. In electrolysis, oxidation takes place at:
1) Anode
2) Cathode
3) The anode as well as cathode
4) The surface of electrolyte solution
385. The calomel electrode is reversible with respect to :
1) Hg 2+
2 2) H + 3) Hg 2+ 4) Cl−
386. The molecular conductivity and equivalent conductivity are same for the solution of :
1) 1 𝑀 NaCl 2) 1 𝑀 Ba(NO3 )2 3) 1 𝑀 La(NO3 )3 4) 1 𝑀 Th(NO3 )4
387. Galvanised iron sheets have coating of :
1) Cu 2) Sn 3) Zn 4) Carbon
388. What is the effect of dilution on the equivalent conductance of strong electrolyte?
1) Decreases on dilution 2) Remains unchanged
3) Increases on dilution 4) None of these
389. The standard 𝐸red values of A, B and C are +0.68 V, - 2.54 V, - 0.50 V respectively. The order of their
°

reducing power is
1) 𝐴 > 𝐵 > 𝐶 2) 𝐴 > 𝐶 > 𝐵 3) 𝐶 > 𝐵 > 𝐴 4) 𝐵 > 𝐶 > 𝐴
390. In the electrolysis of acidulated water, it is desired to obtain 1.12 cc of hydrogen per second under STP
condition. The current to be passed is
1) 1.93 A 2) 9.65 A 3) 19.3 A 4) 0.965 A
391. The ratio of weights of hydrogen and magnesium deposited by the same amount of electricity from H2 SO4
and MgSO4 in aqueous solution are :
1) 1 : 8 2) 1 : 12 3) 1 : 16 4) None of these
392. For which case Λ values 𝑣𝑠√𝑐 show a straight line?
1) KCl 2) HCOOH 3) CH3 NH2 4) CH3 COOH
393. At pH = 2, 𝐸°Quinhydrone = 1.30 𝑉, 𝐸Quinhydrone will be ∶

1) 1.36 V 2) 1.30 V 3) 1.42 V 4) 1.20 V


394. Reduction potentials of A, B, C, and D are 0.8 V, 0.79 V, 0.34 V and -2.37 V respectively. Which element
displaces all the other three elements?
1) B 2) A 3) D 4) C
395. The molar conductivity of NaCl, HCl and CH3 COONa at infinite dilution are 126.45, 426.16 and 91
ohm−1 cm2 mol−1 respectively. The molar conductivity of CH3 COOH at infinite dilution is :
1) 201.28 ohm−1 cm2 mol−1
2) 698.28 ohm−1 cm2 mol−1
3) 390.71 ohm−1 cm2 mol−1
4) 540.48 ohm−1 cm2 mol−1
396. During the electrolysis of a solution of AgNO3 , 9650 C of charge is passed through the electroplating bath.
The mass of silver deposited at the cathode will be
1) 108 g 2) 10.8 g 3) 1.08 g 4) 18.10 g
397. If the H concentration is decreased from 1 M to 10 M at 25 ℃ for the couple MnO−
+ −4 2+
4 / Mn , then the
oxidising power of the MnO− 4 / Mn
2+
couple decreases by
1) -0.18 V 2) 0.18 V 3) 0.38 V 4) -0.38 V

P a g e | 30
398. The formula α = Λv
is valid for :
Λ∞
1) Weak electrolytes 2) Strong electrolytes 3) Salts 4) None of these
399. The e.m.f. of the cell Zn | Zn (1 𝑀)|| Cu |Cu(1𝑀) is 1.1 volt. If the standard reduction potential of
2+ 2+

Zn2− | Zn is − 0.78 volt, what is the oxidation potential of Cu | Cu2+ ?


1) + 1.86 V 2) 0.32 V 3) − 0.32 V 4) −1.86 V
400. Which one will liberate Br2 from KBr?
1) HI 2) I2 3) Cl2 4) SO2
401. The laws of electrolysis ware proposed by
1) Kohlraush 2) Faraday 3) Nernst 4) Berthelot
402. In a galvanic cell, the electrons flow from
1) Anode to cathode through the external circuit 2) Anode to cathode through the solution
3) Cathode to anode through the external circuit 4) Cathode to anode through the solution
403. Daniel cell, anode and cathode are respectively
1) Zn | Zn2+ and Cu2+ | Cu 2) Cu | Cu2+ and Zn2+ | Zn 3) Fe | Fe2+ and Cu2+ | Cu 4) Cu | Cu2+ and Fe2+ | Fe
404. The relationship between Gibbs’ free energy change (Δ𝐺) and emf (𝐸) of a reversible electrochemical cell
is given by
1) 𝛥𝐺 = 𝑛𝐹𝐸 2) 𝛥𝐺 = 𝑛𝐹/𝐸 3) 𝛥𝐺 = −𝑛𝐹𝐸 4) 𝛥𝐺 = 𝐸/𝑛𝐹
405. Conductivity of a strong electrolyte
1) Decreases on dilution 2) Increases on dilution
3) Does not change considerably on dilution 4) Depends on density
406. I. Cu + 2HCl → CuCl2 + H2 (g)
°
[𝐸𝐶𝑢 2+ /𝐶𝑢 = +0.34 𝑉 ]

II. Zn + 2HCl → ZnCl2 + H2 (g)


°
[𝐸𝑍𝑛 2+ /𝑍𝑛 = −0.76 𝑉 ]

1
III. Ag + 2HCl → AgCl + H (g)
2 2
°
[𝐸𝐴𝑔 + /𝐴𝑔 = +0.80 𝑉 ]

Which of the following reaction is feasible ?


1) (ii) 2) (i) 3) (iii) 4) All of these
407. The same amount of electricity was passed through two separate electrolytic cells containing solutions of
nickel nitrate and chromium nitrate respectively. If 0.3 g of nickel was deposited in the first cell, the
amount of chromium deposited is :
(at.wt. Ni = 59, Cr = 52)
1) 0.1 g 2) 0.17 g 3) 0.3 g 4) 0.6 g
408. Which of the following statements are correct concerning redox properties?
I AmetalM for which 𝐸 ° for the half reaction
𝑀𝑛+ + 𝑛𝑒 − = 𝑀, is very negative will be Agood reducing agent.
II The oxidizing power of the halogens decreases from chlorine to iodine.
III The reducing power of hydrogen halides increases from hydrogen chloride to hydrogen
iodide.
1) I , II and III 2) I and II 3) I only 4) II and III only
409. The oxidation potential values of A, B, C and D are -0.03, +0.108 V,-0.07 V and +0.1 V respectively. The
non-spontaneous cell reaction takes place between
1) AandB 2) B and D 3) D and A 4) B and C
410. Cu + 2𝑒 ⟶ Cu; log[Cu ] 𝑣𝑠. 𝐸red graph is of the type as shown in figure where 𝑂𝐴 = 0.34 V, then
2+ 2+

electrode potential of the half cell of Cu |Cu2+ (0.1 𝑀) will be :

P a g e | 31
0.0591 3) 0.34 V 4) None of these
1) −0.34 + V 2) 0.34 + 0.0591 V
2
411. Which of the following is correct?
1) Zinc acts as cathode in Daniell cell
2) In a Li − Zn couple, zinc acts as anode
3) Copper will displace iron in solution
4) Zinc displaces tin from its solution
412. Standard electrode potential of cell H2 | H + ||Ag + | Ag is
1) 0.8 V 2) -0.8 V 3) -1.2 V 4) 1.2 V
413. The increase in equivalent conductivity of a weak electrolyte solution with dilution is attributed to :
1) Increase in degree of dissociation
2) Increase in ionic mobility
3) Both (a) and (b)
4) None of the above
414. The equivalent conductivity of monobasic acid at infinite dilution is 348 ohm−1 cm2 eq−1 . If the resistivity
of the solution containing 15 g acid (mol. wt. 49) in 1 litre is 18.5 ohm cm, what is the degree of
dissociation of acid?
1) 45.9% 2) 40.2% 3) 60.4% 4) 50.7%
415. The standard emf of a cell involving one electron change is found to be 0.591 V and 25 ℃. The equilibrium
constant of the reaction is (𝐹 = 96500 C mol−1)
1) 1.0 × 101 2) 1.0 × 105 3) 1.0 × 1010 4) 1.0 × 1030
416. In which cell, liquid function potential need to be eliminated?
1) Pt/H2 (P ) |HCl|Pt/H2 (P2 )
1

2) Pt/H2 | HCl| HCl| Pt/H2


𝑐1 𝑐2
3) Nicad cell
4) Lead storage battery
417. During the electrolysis of fused NaCl, which reaction occurs at anode?
1) Chloride ions are oxidized 2) Sodium ions are oxidized
3) Chloride ions are reduced 4) Sodium ions are reduced
418. The factor which is not affecting the conductivity of any solution is
1) Temperature 2) Dilution 3) Nature of electrolyte 4) None of these
419. For the following cell with hydrogen electrodes at two different pressures 𝑝1 and𝑝2
Pt (H2 ) | H + (𝑎𝑞) | 𝑃𝑡 (H2 )
𝑝1 1M 𝑝2
emf is given by
𝑅𝑇 𝑝1 𝑅𝑇 𝑝1 𝑅𝑇 𝑝2 𝑅𝑇 𝑝2
1) log 𝑒 2) log 𝑒 3) log 𝑒 4) log 𝑒
𝐹 𝑝2 2𝐹 𝑝2 𝐹 𝑝1 2𝐹 𝑝1
420. 3 faraday of electricity is passed through molten Al2 O3 , aqueous solution of CuSO4 and molten NaCl taken
in three different electrolytic cells. The amount of Al, Cu and Na deposited at the cathodes will be in the
ratio of :
1) 1 mole : 2 mole : 3 mole
2) 3 mole : 2 mole : 1 mole

P a g e | 32
3) 1 mole : 1.5 mole : 3 mole
4) 1.5 mole : 2 mole : 3 mole
421. For which electrolyte the evaluation of 𝐴∞ is not possible by extrapolation of
Λ vs √𝑐 curves to zero concentration?
1) KCl 2) NH4 OH 3) NaCl 4) K 2 SO4
422. If the current is passed into the solution of an electrolyte:
1) Anions move towards anode, cations towards cathode
2) Anions and cations both move towards anode
3) Anions move towards cathode, cations towards anode
4) No movement of ions takes place
423. For Acell given below,
Ag | Ag + || Cu2+ | Cu
- +
Ag + e ⟶ Ag, 𝐸 ° = 𝑥
+ −

Cu2+ + 2e− ⟶ Cu, 𝐸 ° = 𝑦


°
𝐸cell is
1) 𝑥 + 2𝑦 2) 2𝑥 + 𝑦 3) 𝑦 – 𝑥 4) 𝑦 – 2𝑥
424. The element that is easiest to be reduced is :
1) Fe 2) Cu 3) Ag 4) Sn
425. The electrochemical equivalent of silver is 0.0011180 g. When an electric current of 0.5 ampere is passed
through an aqueous silver nitrate solution of 200 sec, the amount of silver deposited is:
1) 1.1180 g 2) 0.11180 g 3) 5.590 g 4) 0.5590 g
426. Which graph correctly correlates 𝐸Cell as a function of concentrations for the cell (for different values of M
and Mʹ)?
Zn(𝑠) + Cu2+ (𝑀) ⟶ Zn2+ (𝑀ʹ) + Cu(𝑠);
𝐸°Cell = 1.10 𝑉
[Zn2+ ]
𝑋 − axis ∶ log10 , 𝑌 − axis ∶ 𝐸Cell
[Cu2+ ]

1) 2) 3) 4)

427. If the molar conductance value of Ca2+ and Cl− at infinite dilution are respectively 118.88 ×
10−4 m2 mho mol−1 and 77.33 × 10−4 m2 mho mol−1 then that of CaCl2 is (in m2 mho mol−1 )
1) 118.88 × 10−4 2) 154.66 × 10−4 3) 273.54 × 10−4 4) 196.21 × 10−4
428. The best conductor of electricity is a 0.1 M solution of:
1) Boric acid 2) Sulphuric acid 3) Acetic acid 4) Propionic acid
429. If 𝐸Fe2+/Fe = −0.441 V and 𝐸Fe3+/Fe2+ = 0.771 V, the standard e.m.f. of the reaction Fe + 2Fe3+ ⟶ 3Fe2+
° °

will be :
1) 1.212 V 2) 0.111 V 3) 0.330 V 4) 1.653 V
430. The reaction taking place at anode when an aqueous solution of CuSO4 is electrolysed using inert Pt
electrode :
1) 2SO2− 2−
4 ⟶ S2 O3 + 2𝑒
2) Cu + 2𝑒 ⟶ Cu
2+

3) 2H2 O ⟶ O2 + 4H + + 4𝑒
4) 2H + + 2𝑒 ⟶ H2
431. The cell reaction for the given cell is spontaneous if :

P a g e | 33
Pt Cl2 |Cl− (1𝑀)||Cl− (1𝑀)| Pt Cl2
P1 P2
1) 𝑃1 > 𝑃2 2) 𝑃1 < 𝑃2 3) 𝑃1 = 𝑃2 4) 𝑃1 = 1 atm
432. What is the time (in sec) required for depositing all the silver present in 125mL of 1 M AgNO3 solution by
passing a current of 241.25 A ? (1F = 96500 C)
1) 10 2) 50 3) 1000 4) 100
433. The limiting molar conductivities Λ∘ for NaCl,KBr and KCl are 126,152 and 150 S cm2 mol−1 respectively.
The Λ∘ for NaBr is
1) 128 S cm2 mol−1 2) 176 S cm2 mol−1 3) 278 S cm2 mol−1 4) 302 S cm2 mol−1
434. The conductivity of saturated solution of CaF2 is 3.86 × 10−5 mho cm−1 and that of water used for
solution is 0.15 × 10−5 . The conductivity of CaF2 alone is :
1) 3.71 × 10−5 2) 4.01 × 10−5 3) 3.7 × 10−4 4) 3.86 × 10−4
435. The limiting molar conductivities Λ° for NaCl, KBr and KCl are 126, 152 and 150 S cm2 mol−1 respectively.
The Λ° for NaBr is
1) 128 S cm2 mol−1 2) 248 S cm2 mol−1 3) 328 S cm2 mol−1 4) 348 S cm2 mol−1
436. The metal that cannot be obtained by electrolysis of the aqueous solution of its salts is :
1) Ag 2) Cr 3) Cu 4) Al
437. The SI unit for ionic mobility is:
1) m2 volt −1 sec −1 2) cm2 volt −1 sec −1 3) cm volt −1 sec −1 4) cm−2 volt −1 sec −1
438. When 1 faraday of electricity is passed through CuSO4 solution, number of atoms formed is :
1) 6.02 × 1023 2) 3.01 × 1023 3) 2 4) 6.02 × 1023
439. e.m.f. of cell Ni|Ni2+ (0.1𝑀)|| Au3+ (1.0 𝑀)|Au 𝑖𝑠 … . ., if 𝐸° for Ni2+ |Ni is − 0.25 V, 𝐸° for Au3+ |Au is 1.50 V.
1) + 1.25 V 2) −1.75 V 3) + 1.75 V 4) + 4.0 V
440. In a cell containing zinc electrode and standard hydrogen electrode(SHE), the zinc electrode acts as :
1) Anode
2) Cathode
3) Neither cathode nor anode
4) Both anode and cathode
441. Standard reduction electrode potentials of three metals 𝐴, 𝐵 and 𝐶 are respectively + 0.5 V, − 3.0 V and –
1.2 V. The reducing powers of these metals are
1) A > 𝐵 > 𝐶 2) C > 𝐵 > 𝐴 3) A > 𝐶 > 𝐵 4) B > 𝐶 > 𝐴
442. 𝐸1 , 𝐸2 , and 𝐸3 are the emfs of the following three galvanic cells respectively
IV. Zn (s) | 𝑍𝑛2+ (0.1 M) || Cu2+ (1 M) | Cu (s)
V. Zn (s) | 𝑍𝑛2+ (1 M) || Cu2+ (1 M) | Cu (s)
VI. Zn (s) | 𝑍𝑛2+ (1 M) || Cu2+ (0.1 M) | Cu (s)
Which one of the following is true?
1) 𝐸2 > 𝐸1 > 𝐸3 2) 𝐸1 > 𝐸2 > 𝐸3 3) 𝐸3 > 𝐸1 > 𝐸2 4) 𝐸3 > 𝐸2 > 𝐸1
443. Which of the following expressions correctly represents the equivalent conductance at infinite dilution of
Al2 (SO4 )3 . Given that Λ∞ ∞
Al3+ and Λ SO2− are the equivalent conductances at infinite dilution of the respective
4
ions?
∞ ∞ ∞ ∞ ∞ ∞ 1 1
1) 2Λ Al3+ + 3Λ SO2−
4
2) 2Λ Al3+ + Λ SO2−
4
3) (Λ Al3+ + Λ SO2−
4
)×6 4) 3 Λ∞ ∞
Al3+ + 2 Λ SO2−
4
444. The amount of electricity required to produce one mole of copper from copper sulphate solution will be
1) 1 F 2) 2.33 F 3) 2 F 4) 1.33 F
445. If ‘𝐹’ is faraday and ‘𝑁’ is Avogadro number, then charge of electron can be expressed as
𝐹 𝑁
1) 𝐹 × 𝑁 2) 3) 4) 𝐹 2 𝑁
𝑁 𝐹
446. A solution containing one mole per litre of each Cu(NO3 )2 , AgNO3 , Hg 2 (NO3 )2 and Mg(NO3 )2 is being
electrolysed by using inert electrodes. The values of standard electrode potentials in volts (reducing
potentials) are Ag/Ag + = +0.80, 2Hg/Hg 2+ = +0.79, Cu/Cu2+ = +0.34, Mg/Mg 2+ = −2 With increasing
voltage, the sequence of deposition of metals on the cathode will be

P a g e | 34
1) Ag, Hg, Cu 2) Cu, Hg, Ag 3) Ag, Hg, Cu, Mg 4) Mg, Cu, Hg, Ag
447. The elementwhich can displace three other halogens from their compound is
1) F 2) Cl 3) Br 4) I
448. When an electrolytic solution conducts electricity, current is carried out by :
1) Electrons 2) Cations and anions 3) Neutral atoms 4) None of these
449. In the problem 15, ionic conductance of K ion is :
+

1) 64.35 2) 60.20 3) 262.26 4) 26.22


450. In the electrochemical cell, H2 (g)1 atm|H + (1 M)|| Cu2+ (1 M)| Cu(𝑠)
Which one of the following statements is true?
1) H2 is anode, Cu is cathode 2) Cu is anode, H2 is cathode
3) Oxidation occurs at Cu electrode 4) Reduction occurs at H2 electrode
451. For the reduction of silver ions with copper metal, the standard cell potential is 0.46 V at 25°C. The value
of standard Gibbs energy ∆𝐺° will be :
1) − 89.0 kJ 2) − 89.0 J 3) − 44.5 kJ 4) − 98.0 kJ
452. The resistance of N solution is found to be 2.5 × 103 Ω . The equivalent conductance of the solution is
10
(cell constant = 1.25cm−1 )
1) 2.5Ω−1 cm2 equiv −1 2) 5.0Ω−1 cm2 equiv −1
3) 2.5Ω−1 cm−2 equiv −1 4) 5.0Ω−1 cm−2 equiv −1
453. The corrosion of iron object is favoured by :
1) Presence of H + ion
2) Presence of moisture in air
3) Presence of impurities in iron object
4) All of the above
454. The algebraic sum of potentials of two electrodes of a galvanic cell is called :
1) Potential difference 2) Ionic difference 3) e.m.f. 4) Electrode difference
455. When Zn piece is kept in CuSO4 solution, copper gets precipitated because:
1) Standard reduction potential of zinc is more than copper
2) Standard reduction potential of zinc is less than copper
3) Atomic number of zinc is larger than copper
4) Atomic number of zinc is lower than copper
456. The cell reaction of Acell is
Mg (𝑠) + Cu2+ (𝑎𝑞) ⟶ Cu (𝑠) + Mg 2+ (𝑎𝑞) .
If the standard reduction potentials of Mg and Cu are – 2.37 and + 0.34 V respectively. The emf of the cell
is
1) 2.03 V 2) -2.03 V 3) +2.71 V 4) -2.71 V
457. Iron sheets are galvanized to :
1) Prevent action of O2 and H + on Fe
2) Prevent oxidation of Fe
3) Prevent rusting
4) All of the above
458. Whether tin can displace lead from aqueous lead bromide solution?
1) No
2) Yes, because standard reduction potential of Sn <Pb
3) Yes, because standard reduction potential of Sn >Pb
4) None of the above
459. The cell constant is
𝑙 𝑎 𝜅
1) 2) 3) 𝑎 × 𝑙 4)
𝑎 𝑙 𝑅
460. The standard electrode potential is measured by
1) Electrometer 2) Voltmeter 3) Pyrometer 4) Galvanometer

P a g e | 35
461. Hydrogen gas is not liberated when the following metal is added to dil. HCl
1) Ag 2) Zn 3) Mg 4) Sn
462. In acidic medium MnO4 is converted to Mn . The quantity of electricity in faraday required to reduce 0.5
− 2+

mole of MnO− 4 to Mn
2+
would be
1) 2.5 2) 5 3) 1 4) 0.5
463. The number of coulombs required to reduce 12.3 g of nitrobenzene to aniline is
1) 115800 C 2) 5790 C 3) 28950 C 4) 57900 C
464. Rust is a mixture of :
1) FeO and Fe(OH)2 2) FeO and Fe(OH)3 3) Fe2 O3 and Fe(OH)3 4) Fe3 O4 and Fe(OH)3
465. 0.04 𝑁 solution of a weak acid has conductivity 4.23 × 10 mho cm . If the degree of dissociation of acid
−4 −1

at this dilution is 0.0612, then equivalent conductivity at infinite dilution is ……mho cm2 eq.−1 ∶
1) 172.8 2) 180 3) 190 4) 160
466. Ionic mobility of Ag is
+

( 𝜆Ag+ = 5 × 10−1 Ω−1 cm2 equiv −1)


1) 5.2 × 10−9 2) 2.4 × 10−9 3) 1.52 × 10−9 4) 8.25 × 10−9
467. If the ∆𝐺 of a cell reaction AgCl + 𝑒 − → Ag + Cl− is −21.20 kJ, the standard emf of cell is
1) 0.239 V 2) 0.220 V 3) −0.320 V 4) −0.110 V
468. The resistance of 0.01 𝑁 solution of an electrolyte was found to be 210 ohm at 298 K, using a conductivity
cell of cell constant 0.66 cm−1 . The equivalent conductivity of solution is :
1) 314.28 mho cm2 eq.−1 2) 3.14 mho cm2 eq.−1 3) 314.28 mho−1 cm2 eq.−14) 3.14 mho−1 cm2 eq.−1
469. The electrode potential measures the :
1) Tendency of the electrode to gain or lose electrons
2) Tendency of the cell reaction to occur
3) Difference in the ionisation potential of electrode and metal ion
4) Current carried by an electrode
470. Conductance ‘𝐶’ (in 𝑆) in directly proportional to the area of the electrode and concentration and inversely
proportional to length of separation of electrode, the unit of constant of proportionality is:
1) S m mol−1 2) S m2 mol−1 3) S −2 m2 mol 4) S 2 m2 mol2
471. Electrode potential of Zn /Zn is – 0.76 V and that of Cu /Cu is + 0.34 V. The emf of the cell constructed
2+ 2+

between these two electrodes is


1) 1.10 V 2) − 1.10 V 3) 2.20 V 4) − 2.20 V
472. The electrode potential of a glass electrode depends upon :
1) Concentration of chloride ions
2) Concentration of hydrogen ions
3) Concentration of KCl solution
4) None of the above
473. The 𝐸 ° ofFe2+ / Fe and Sn2+ /Sn are -0.44 V and -0.14 V respectively. If cell reaction is
Fe + Sn2+ → Fe2+ + Sn
then emf of the cell is
1) +0.30 V 2) -0.58 V 3) +0.58 V 4) -0.30 V
474. The standard emf of Agalvanic cell involving cell reaction with 𝑛 = 2 is found to be 0.295 V at 25 ℃. The
equilibrium constant of the reaction would be
(Given, 𝐹 = 96500 C mol−1 , 𝑅 = 8.314 JK −1 mol−1 )
1) 2.0 × 1011 2) 4.0 × 1012 3) 1.0 × 102 4) 1.0 × 1010
475. 5 A is passed through a solution of zinc sulphate for 40 min. Find the amount of zinc deposited at the
cathode
1) 4.065 g 2) 8.065 g 3) 16.065 g 4) 32.065 g
476. The value of equilibrium constant for a feasible cell reaction is :
1) < 1 2) Zero 3) = 1 4) > 1
477. More electropositive elements have :

P a g e | 36
1) Positive reduction potential
2) Tendency to gain electrons
3) Negative reduction potential
4) Negative oxidation potential
478. Molar conductivities (Λ̊ 𝑚 ) at infinite dilution of NaCl, HCl and CH3 COONa are 126.4, 425.9 and 91.0
S cm2 mol−1 respectively. Λ̊ 𝑚 for CH3 COOH will be :
1) 425.5 S cm2 mol−1 2) 180.5 S cm2 mol−1 3) 290.8 S cm2 mol−1 4) 390.5 S cm2 mol−1
479. The approximate e.m.f. of a dry cell is :
1) 2.0 V 2) 1.2 V 3) 6 V 4) 1.5 V
480. In the electrolysis of CuCl2 solution using Cu electrodes the mass of cathode increases by 3.18 g. What
happened at the other electrode?
1) 0.05 mole of Cu2+ ions passed into solution
2) 0.112 litre of Cl2 was liberated
3) 0.56 litreO2 was liberated
4) 0.1 mole of Cu2+ ions passed into the solution
481. An electrochemical cell is set up as follows
Pt(H2 , 1 atm)| 0.1 M HCl | |0.1 M acetic acid |
(H2 , 1 atm)Pt Emf
of this cell will not be zero because
The pH of 0.1 M HCl and 0.1 M acetic acid is 2) Acids used in two compartments are different
1)
not the same
3) Emf of a cell depends on the molarities 4) The temperature is constant
of acids used
482. The conductivity of 𝑁/10 KCl solution at 200C is 0.0212 ohm−1 cm−1 and the resistance of cell containing
this solution at 200 C is 55 ohm. The cell constant is:
1) 2.173 cm−1 2) 1.166 cm−1 3) 4.616 cm−1 4) 3.324 cm−1
483. Standard electrode potential of cell H2 |H + ||Ag + |Ag is (Given, 𝐸°Ag+/Ag = 0.80 V)
1) 0.4 V 2) 0.8 V 3) 1.4 V 4) 1.8 V
484. According to Kohlrausch’s law the limiting value of equivalent conductivity of an electrolyte A2 B is given
by :
∞ ∞ 1 1 ∞ ∞
1) λ𝐴+ + λ𝐵−2 2) λ𝐴∞+ + λ∞ 𝐵 −2 3) λ𝐴∞+ + λ∞ 𝐵 −2 4) 2λ𝐴+ + λ𝐵−2
2 2
485. Normal Al − AlCl3 coupled with normal hydrogen electrode gives an e.m.f. of 1.66 V. The standard
oxidation electrode potential of aluminium is :
1) − 1.66 V 2) + 1.66 V 3) − 0.83 V 4) + 0.83 V
486. When 9.65 C of electricity is passed through a solution of silver nitrate (atomic weight of Ag = 107.87
taking as 108), the amount of silver deposited is
1) 5.8 mg 2) 10.8 mg 3) 15.8 mg 4) 20.8 mg
487. Electrolysis of dilute aqueous NaCl solution was carried out by passing 10mA current. The time required
to liberate 0.01 mole of H2 gas at the cathode is (1F = 96500 C mol−1 )
1) 9.65 × 104 𝑠 2) 19.3 × 104 𝑠 3) 28.95 × 104 𝑠 4) 38.6 × 104 𝑠
488. The standard reduction potentials at 298 K for the following half reactions are given against each
Zn2+ (𝑎𝑞) + 2𝑒 − ⟶ Zn (𝑠); 𝐸 ° = − 0.762 𝑉
Cr 3+ (𝑎𝑞) + 3𝑒 − ⟶ Cr (𝑠); 𝐸 ° = − 0.740 𝑉
2H + (𝑎𝑞) + 2𝑒 − ⟶ H2 (𝑔); 𝐸 ° = 0.00 𝑉
Fe3+ (𝑎𝑞) + 𝑒 − ⟶ Fe2+ (𝑎𝑞); 𝐸 ° = + 0.762 𝑉
The strongest reducing agent is
1) Zn (s) 2) Cr (s) 3) 𝐻2 (g) 4) Fe2+ (aq)
489. Which colourless gas evolves when NH4 Cl reacts with zinc in Adry cell battery?
1) NH3 2) N2 3) H2 4) Cl2

P a g e | 37
490. Which of the following expression is correct?
° °
1) 𝛥𝐺° = − 𝑛𝐹𝐸cell 2) 𝛥𝐺° = + 𝑛𝐹𝐸cell
°
3) 𝛥𝐺° = −2.303 𝑅𝑇 𝑛𝐹𝐸cell 4) 𝛥𝐺° = −𝑛𝐹 log𝐾𝑐
491. Amount of electricity that can deposit 108 g of silver from AgNO3 solution is
1) 1 F 2) 1 A 3) 1 C 4) None of these
492. The units of equivalent conductance, are
1) Ω cm2 equiv −1 2) Ω cm2 equiv 3) Ω−1 cm2 equiv −1 4) Ω cm2 equiv
493. The correct order of chemical reactivity with water according to electrochemical series is:
1) K > Mg > Zn > Cu 2) Mg > Zn > Cu > K 3) K > Zn > Mg > Cu 4) Cu > Zn > Mg > K
494. A hydrogen electrode placed in a buffer solution of CH3 COONa and acetic acid in the ratio’s 𝑥 ∶ 𝑦 and 𝑦 ∶ 𝑥
has electrode potential values 𝐸1 volt and 𝐸2 volt respectively at 25°C. The p𝐾a values of acetic acid is
(𝐸1 and𝐸2 are oxidation potential) :
𝐸 + 𝐸2
1) 1
0.118
𝐸2 − 𝐸1
2)
0.118
𝐸1 + 𝐸2
3)
0.118
𝐸1 − 𝐸2
4)
0.118
495. Passage of three faraday of charge through aqueous solution of AgNO3 , CuSO4 , Al(NO3 )3 and NaCl will
deposit metals at the cathode in the molar ratio of:
1) 1 : 2 : 3 : 1 2) 6 : 3 : 2 : 6 3) 6 : 3 : 0 : 0 4) 3 : 2 : 1 : 0
496. The rusting of iron takes place as follows
1
2H + + 2e− + O2 ⟶ H2 O(𝑙);
2
𝐸 ° = +1.23 𝑉
Fe2+ + 2e− ⟶ Fe (s); 𝐸 ° = −0.44 𝑉
Calculate Δ𝐺° for the net process.
1) -322 kJ mol−1 2) -161 kJ mol−1 3) -152 kJ mol−1 4) -76 kJ mol−1
497. Electr KCl 𝐊𝐍𝐎𝟑 HCl NaO NaCl
olyte Ac
∞ 𝟐
𝚲 (𝐒 𝐜𝐦149. 145. 426. 91.0 126.
𝐦𝐨𝐥−𝟏 ) 9 0 2 5

Calculate Λ∞HOAc using appropriate molar conductances of the electrolytes listed above at infinite dilution
in H2 O at 25℃.
1) 217.5 2) 390.7 3) 552.7 4) 517.2
498. The emf of a Daniell cell at 298 K is 𝐸1 , Zn| ZnSO4 || CuSO4 | Cu. When the concentration of
(0.01 M) (1.0 M)
ZnSO4 is 1.0 M and that of CuSO4 is 0.01 M, the emf changed to 𝐸2 . What is the relationship between
𝐸1 and 𝐸2 ?
1) 𝐸1 = 𝐸2 2) 𝐸1 > 𝐸2 3) 𝐸1 < 𝐸2 4) 𝐸2 = 0 ≠ 𝐸1
499. A hypothetical electrochemical cell is shown below;

The e.m.f. measured is + 0.20 V. The cell reaction is :


1) The cell reaction cannot be predicted
2) 𝐴 + 𝐵+ ⟶ 𝐴+ + 𝐵
3) 𝐴+ + 𝐵 ⟶ 𝐴 + 𝐵+
4) 𝐴+ + 𝑒 − ⟶ 𝐴; 𝐵+ + 𝑒 − ⟶ 𝐵
500. An increase in equivalent conductivity of strong electrolyte with dilution is mainly due to:

P a g e | 38
1) Increase in ionic molility ions
2) 100% ionisation of electrolyte at normal dilution
3) Increase in both i.e., no. of ions and ionic mobility
4) Increase in no. of ions
501. On the basis of the following 𝐸° values, the strongest oxidizing agent is :
[Fe(CN)6 ]4− → [Fe(CN)6 ]3− + 𝑒 − ; 𝐸° = −0.35 V
Fe2+ ⟶ Fe3+ + 𝑒 − ;𝐸° = 0.77 V
1) Fe2+ 2) Fe3+ 3) [Fe(CN)6 ]3− 4) [Fe(CN)6 ]4−
502. When a lead storage battery is charged, it acts as
1) A primary cell 2) A galvanic cell 3) A concentration cell 4) An electrolytic cell
503. A cell constructed by coupling a standard copper electrode and a standard magnesium electrode has emf
of 2.7 V. If the standard reduction potential of copper electrode is +0.34 V then that of the magnesium
electrode is
1) + 2.36 V 2) − 2.36 V 3) + 3.26 V 4) − 3.26 V
504. When Alead storage battery is discharged
1) Lead sulphate is consumed 2) SO2 is evolved
3) Lead is formed 4) Sulphuric acid is consumed
505. The number of electrons passing per second through a cross-section of copper wire carrying 10−6 ampere
:
1) 6.2 × 1023 2) 6.2 × 1012 3) 6.2 × 1010 4) None of these
506. 4.5 g of Al (at. mass 27 amu) is deposited at cathode from Al solution by a certain quantity of charge. The
3+

volume of H2 produced at STP from H + ions in solution by the same quantity of charge will be :
1) 11.2 L 2) 44.8 L 3) 5.6 L 4) 22.4 L
507. Number of faraday required to liberate 8 g of H2 is :
1) 8 2) 16 3) 4 4) 2
508. Which is correct representation for a cell at equilibrium?
1) ∆𝐺° = −2.303 𝑅𝑇 log 𝐾𝑒𝑞.
2.303𝑅𝑇
2) 𝐸° = log 𝐾𝑒𝑞.
𝑛𝐹
3) −∆𝐺° = 𝑅𝑇 ln 𝐾𝑒𝑞.
4) All of the above.
509. By passing 9.65 A current for 16 min 40 s , the volume of O2 liberated at STP will be
1) 280 mL 2) 560 mL 3) 1120 mL 4) 2240 mL
510. When an electric current is passed through an aqueous solution of sodium chloride :
1) H2 is evolved at the anode
2) Oxygen is evolved at the cathode
3) Its pH progressively decreases
4) Its pH progressively increases
511. Using the following data, for the electrode potentials calculate ∆𝐺°, in kJ, for the indicated reaction
5Ce4+ (𝑎𝑞) + Mn2+ (𝑎𝑞) + 4H2 O(𝑙) → 5Ce3+ (𝑎𝑞) + MNO− +
4 (𝑎𝑞) + 8H (𝑎𝑞)
MnO− + − 2+
4 (𝑎𝑞) + 8H (𝑎𝑞) + 5e → Mn (𝑎𝑞) + 4H2 O(𝑙)𝐸° = +1.51 V
Ce4+ (𝑎𝑞) + e− → Ce3+ (𝑎𝑞)𝐸° = +1.61 V
1) −36.24 2) −48.25 3) −31.54 4) −19.65
512. Maximum number of mole of oxygen gas that can be obtained by the electrolytic decomposition of 90 g of
water will be:
1) 1 2) 2.5 3) 5 4) 9
513. Standard reduction potential for, Li | Li, Zn |Zn, H |H2 and Ag | Ag is − 3.05, −0.762, 0.00 and +
+ 2+ + +

80 V. Which has highest reducing capacity?


1) Ag 2) H2 3) Zn 4) Li
514. At 25°C, the standard e.m.f. of cell having reactions involving a two electron change is found to be 0.295 V.

P a g e | 39
The equilibrium constant of the reaction is :
1) 29.5 × 10−2 2) 10 3) 1010 4) 29.5 × 1010
515. The standard reduction potentials for Zn2+ /Zn, Ni2+ /Ni and Fe2+ /Fe are − 0.76, −0.23 and −0.44 V
respectively. The reaction 𝑋 + 𝑌 2+ ⟶ 𝑋 2+ + 𝑌 will be spontaneous when :
1) 𝑋 = Ni, 𝑌 = Zn 2) 𝑋 = Fe, 𝑌 = Zn 3) 𝑋 = Zn, 𝑌 = Ni 4) 𝑋 = Ni, 𝑌 = Fe
516. The emf of the cell, (𝐸Zn2+ /Zn = −0.76 𝑉)
Zn / Zn2+ (1 M) || Cu2+ (1 M) | Cu
(𝐸Cu2+ /Cu = +0.34 𝑉) will be
1) +1.10 V 2) -1.10 V 3) +0.42 V 4) -0.42 V
517. The acid used in lead storage battery is
1) H2 SO4 2) H3 PO4 3) HCl 4) HNO3
518. When lead accumulator is charged, it is :
1) An electrolytic cell 2) A galvanic cell 3) A daniell cell 4) None of these
519. The increase in equivalent conductivity of an strong electrolytic solution with dilution is attributed to :
1) Increase in number of ions per unit volume
2) Increase in molecular attraction
3) Increase in degree of dissociation
4) Increase in ionic mobility
520. Blocks of magnesium metal are often strapped to the steel hulls of ocean going ships in order to:
1) Provide cathodic protection
2) Protect oxidation of steel
3) Both (a) and (b) are correct
4) Neither (a) nor (b) is correct
521. The electrolytic bath used in gold plating of copper articles contains :
1) Molten gold 2) CuSO4 3) AuCl3 4) AuCl3 + NaCN
522. What will be pH of aqueous solution of electrolyte in electrolytic cell during electrolysis of CuSO4 (𝑎𝑞)
between graphite electrodes?
1) pH = 14.0 2) pH > 7.0 3) pH < 7.0 4) pH = 7.0
523. Electrolytic reduction of alumina to aluminium by Hall-Heroult process is carried out :
1) In the presence of NaCl
2) In the presence of fluoride
3) In the presence of cryolite, which forms a melt with lower melting temperature
4) In the presence of cryolite, which forms a melt with higher melting temperature
524. The art of electroplating was given by :
1) Faraday 2) Edison 3) Graham 4) Brugan
525. A galvanic cell is composed of two hydrogen electrodes, one of which is a standard one. In which of the
following solutions should the other electrode be immersed to get maximum e.m.f.?
1) 0.1 𝑀 HCl 2) 0.1 𝑀 CH3 COOH 3) 0.1 𝑀 H3 PO4 4) 0.1 𝑀 H2 SO4
526. The standard e.m.f. of a galvanic cell can be calculated from :
1) The size of the electrode
2) The pH of thesolution
3) The amount of metal in the anode
4) The 𝐸° values of the two half cells
527. In Acell that utilises the reaction,
Zn (𝑠) + 2H + (𝑎𝑞) → Zn2+ (𝑎𝑞) + H2 (𝑔)
addition of H2 SO4 to cathode compartment, will
1) Lower the E and shift the equilibrium to the left
2) Lower the E and shift the equilibrium to the right
3) Increase the E and shift the equilibrium to the right
4) Increase the E and shift the equilibrium to the left

P a g e | 40
528. The emf of the cell
Ni | Ni2+ (1.0 M) || Au3+ (1.0 M) | Au
is [E ° (Ni2+ / Ni) =-0.25 V and
𝐸 ° (Au3+ / Au) =+1.5 V]
1) 2.00 V 2) 1.25 V 3) -1.25 V 4) 1.75 V
529. The standard reduction potential 𝐸 ° for half reactions are
Zn = Zn2+ + Ze 𝐸 ° = + 0.76 V
Fe = Fe2+ + Ze 𝐸 ° = + 0.41 V
The emf of the cell reaction
Fe2+ + Zn = Zn2+ + Fe is
1) - 0.35 V 2) +0.35 3) +1.17 V 4) -1.`17 V
530. Electrode potential of hydrogen electrode is …….. volt.
1) 0 2) +1 3) -1 4) None of these
531. The number of faraday required to generate 1 g-atom of Mg from MgCl 2 is :
1) 1 2) 2 3) 3 4) 4
532. What is the potential of platinum wire dipped into Asolution of 0.1 M in Sn and 0.01 M in Sn4+ ?
2+

0.059
1) 𝐸 ° 2) 𝐸 ° + 0.059 3) 𝐸 ° + 4) 𝐸 ° - 0.059
2
533. Consider the reaction, 𝑀𝑛+ (𝑎𝑞) + 𝑛𝑒 ⟶ 𝑀0 (𝑠). The standard reduction potential values of the metals
𝑀1 , 𝑀2 and 𝑀3 are −0.34 V, −3.05 V and −1.66 V respectively. The order of their reducing power will be :
1) 𝑀1 > 𝑀2 > 𝑀3 2) 𝑀3 > 𝑀2 > 𝑀1 3) 𝑀1 > 𝑀3 > 𝑀2 4) 𝑀2 > 𝑀3 > 𝑀1
534. H2 cannot be displaced by
1) Li+ 2) Sr 2+ 3) Al3+ 4) Ag +
535. The speed of ions during passage of current depends upon :
1) Nature of ion 2) Potential gradient 3) Dilution of solution 4) All of these
536. For the electrochemical cell, 𝑀 |𝑀 +
||𝑋 − |𝑋
,
𝐸 ° (𝑀+ | 𝑀) = 0.44 𝑉, 𝐸 ° = (𝑋 | 𝑋 − ) = 0.33 𝑉 . From this datAone can deduce that
1) 𝐸cell
°
= −0.77 𝑉
2) 𝑀 + 𝑋 − → 𝑀 + 𝑋 is the spontaneous reaction
+

3) 𝑀 + 𝑋 → 𝑀+ + 𝑋 − is the spontaneous reaction


4) 𝐸cell
°
= 0.77 𝑉
537. The amount of electricity required to liberate 1 g-equiv of Cu is
1) 96500 F 2) 1 F 3) 1 C 4) 96500 A
538. The hydrogen electrode is dipped in a solution of pH = 3 at 25°C. The reduction potential of the cell would
be :
1) 0.177 V 2) ⎯ 0.177 V 3) 0.087 V 4) 0.059 V
539. If Mg + 2𝑒 ⟶ Mg(𝑠);
2+
𝐸 = −2.37 V,
2+
Cu + 2𝑒 ⟶ Cu(𝑠); 𝐸 = + 0.34 V?
The e.m.f. of the cell Mg | Mg 2+ ||Cu2+ |Cuis ∶
1) 2.71 V 2) 2.30 V 3) 2.80 V 4) 1.46 V
540. The charge required for reduction of 1 mole of Cr2 O2− 7 ions to Cr 3+
is
1) 96500 C 2) 2 × 96500 C 3) 3 × 96500 C 4) 6 × 96500 C
541. `forSn /Sn couple is + 0.15 V and that for the Cr / Cr couple is − 0.74 V. These two couples in their
4+ 2+ 3+

standard state are connected to make a cell. The cell potential will be :
1) + 1.83 V 2) + 1.19 V 3) + 0.89 V 4) + 0.18 V
542. The Gibbs energy for the decomposition of Al2 O3 at 500°C is as follows :
2 4
Al O
3 2 3
⟶ 3 Al + O2 ;
∆𝑟 𝐺 = +960 kJ mol−1 .
The potential difference needed for the electrolytic reduction of aluminium oxide (Al2 O3 ) at 500°C is at
least :
P a g e | 41
1) 4.5 V 2) 3.0 V 3) 2.5 V 4) 5.0 V
543. When 1 F of electricity is passed through acidulated water, O2 evolved is
1) 11.2 dm3 2) 5.6 dm3 3) 22.4 dm3 4) 1.0 dm3
544. It is impossible to measure the actual voltage of any half cell by itself because:
1) Both half cell reactions takes place simultaneously
2) Of resistance of wire
3) A reaction does not take place on its own
4) None of the above
545. A current of 𝑖 ampere was passed for 𝑡 second through three cells 𝑃, 𝑄 and 𝑅 connected in series. These
contain respectively silver nitrate, mercuric nitrate and mercurous nitrate. At the cathode of the cell 𝑃,
0.216 g of Ag was deposited. The weights of mercury deposited in the cathode of 𝑄 and 𝑅 respectively are :
1) 0.4012 and 0.8024 g 2) 0.4012 and 0.2006 g 3) 0.2006 and 0.4012 g 4) 0.1003 and 0.2006 g
546. Which loses charge at cathode?
1) Ions
2) Cations
3) Anions
4) Both anions and cations
547. The equivalent conductances of two strong electrolytes at infinite dilution in H2 O (where ions move freely
through a solution) at 25℃ are given below
Λ°CH3 COONa = 91.0 S cm2 /equiv
Λ°HCl = 426.2 S cm2 /equiv
What additional information/quantity one needs to calculate Λ° of an aqueous solution of acetic acid?
1) Λ° of NaCl
2) Λ° of CH3 COOK
3) The limiting equivalent conductance of H + (λ°H+ )
4) Λ° of chloroacetic acid (ClCH2 COOH)
548. For the redox reaction,
Zn (𝑠) + Cu2+ (0.1 M) →
Zn2+ (1 M) + Cu (𝑠)
° °
taking place in a cell, 𝐸cell is 1.10 V. 𝐸cell for the cell will be
𝑅𝑇
(2.303 = 0.0591)
𝐹
1) 2.14 V 2) 1.80 V 3) 1.07 V 4) 0.82 V
549. The metal that forms a self protecting film of oxide to prevent corrosion is
1) Na 2) Al 3) Cu 4) Au
550. The emf of the cell Zn | Zn (0.01 M) || Fe (0.001 M) | Fe at 298 K is 0.2905. The value of equilibrium
2+ 2+

constant for the cell reaction is


0.32 0.32 0.32 0.26
1) 10100.0298 2) 𝑒 0.0295 3) 100.0591 4) 100.0295
551. The conductivity of a 0.1 𝑁 KCl solution at 230C is 0.012 ohm−1 cm−1 .The resistance of the cell containing
the solution at the same temperature was found to be 55 ohm. The cell constant will be :
1) 0.918 cm−1 2) 0.66 cm−1 3) 1.142 cm−1 4) 1.12 cm−1
552. Which of the following is an additive property?
1) Conductance 2) Viscosity 3) Surface tension 4) None of these
553. Limiting molar ionic conductivities of a uni-univalent electrolyte are 57 and 73. The limiting molar
conductivity of the solution will be :
1) 130 𝑆cm2 mol−1 2) 65 𝑆cm2 mol−1 3) 260 𝑆cm2 mol−1 4) 187 𝑆cm2 mol−1
554. The 𝐸° for half-cell Fe/Fe2+ andCu/Cu2+are−0.44 V and+0.32 V respectively, then
1) Cu2+ oxidises Fe 2) Cu oxidises FeFe2+ 3) Cu reduces Fe2+ 4) Cu2+ oxidises Fe2+
555. The standard reduction electrode potential values of the elements 𝐴, 𝐵 and 𝐶 are + 0.68, ⎯2.50 and ⎯ 0.50
V respectively. The order of their reducing power is :

P a g e | 42
1) 𝐴 > 𝐵 > 𝐶 2) 𝐴 > 𝐶 > 𝐵 3) 𝐶 > 𝐵 > 𝐴 4) 𝐵 > 𝐶 > 𝐴
556. 1 coulomb of charge passes through solution of AgNO3 and CuSO4 connected in series and the
concentration of two solution being in the ratio 1 : 2. The ratio of amount of Ag and Cu deposited on Pt
electrode is :
1) 107.9 : 63.54 2) 54 : 31.77 3) 107.9 : 31.77 4) 54 : 63.54
557. Calculate the volume of hydrogen at NTP obtained by passing a current of 0.4 ampere through acidified
water for 30 minute :
1) 0.0836 litre 2) 0.1672 litre 3) 0.0432 litre 4) 0.836 litre
558. Passage of 1 faraday of electricity through a solution of CuSO4 , deposits :
1) 1 mole of Cu 2) 1 g-atom of Cu 3) 1 molecule of Cu 4) 1 g equivalent of Cu
559. Two different electrolytic cells filled with molten Cu(NO3 )2 and molten Al(NO3 )3 respectively are
connected in series. When electricity is passed 2.7 g Al is deposited on electrode. Calculate the weight of Cu
deposited on cathode.
[Cu = 63.5; Al = 27.0 g mol−1 ]
1) 190.5 g 2) 9.525 g 3) 63.5 g 4) 31.75 g
560. For Acell reaction involving Atwo-electron change, the standard emf of the cell is found to be 0.295 V at
25℃. The equilibrium constant of the reaction at 25 ℃ will be
1) 1 × 10−10 2) 29.5 × 10−2 3) 10 4) 1 × 1010
561. A student made the following observations in the laboratory,
i) Clean copper metal did not react with 1 molar Pb(NO3 )2 solution.
ii) Clean lead metal dissolved in a 1 molar AgNO3 solution and crystals of Ag metal appeared.
iii) Clean silver metal did not react with 1 molar Cu(NO3 )2 solution.
The order of decreasing reducing character of the three metals is :
1) Cu, Pb, Ag 2) Cu, Ag, Pb 3) Pb, Cu, Ag 4) Pb, Ag, Cu
562. Thestandardoxidation potentials of the electrodes Ag | Ag , Sn | Sn , Ca | Ca , Pb | Pb2+ are ⎯ 0.8, 0.136,
+ 2+ 2+

2.866 and 0.126 V respectively. The most powerful oxidising agent among these metal ions is :
1) Pb2+ 2) Ca2+ 3) Sn2+ 4) Ag +
563. Given 𝑙/𝑎 = 0.5cm−1 , 𝑅 = 50 ohm, 𝑁 = 1.0. The equivalent conductance of the electrolytic cell is
1) 10Ω−1 cm2 g equiv −1 2) 20Ω−1 cm2 g equiv −1
3) 300Ω cm g equiv
−1 2 −1
4) 100Ω−1 cm2 g equiv −1
564. The resistance of 0.01 𝑁 solution of an electrolyte was found to be 210 ohm at 298 K, using a conductivity
cell of cell constant 0.66 cm−1 . The conductivity of solution is :
1) 3.14 × 10−3 mho cm−1
2) 3.14 × 10−3 mho−1 cm
3) 3.14 mho cm−1
4) 3.14 mho−1 cm−1
565. Ni / Ni2+ [1.0 M] || Au3+ [1.0 M] / Au where 𝐸 °
for Ni2+ /Ni is − 0.250 V; and 𝐸 ° for
Au3+ / Auis 0.150 V. The emf of the cell is
1) +1.25 V 2) -1.75 V 3) +1.75 V 4) +0.4 V
566. The resistance of 1N solution of acetic is 250Ω ,when measured in a cell having a cell constant of
1.15cm−1 .The equivalent conduction (in ohm−1 cm2 equiv−1 ) of 1N acetic acid is
1) 2.3 2) 4.6 3) 9.2 4) 18.4
567. 1 mole of Al is deposited by 𝑋 coulomb of electricity passing through aluminium nitrate solution. The
number of mole of silver deposited by 𝑋 coulombof electricity from silver nitrate solution is :
1) 3 2) 4 3) 2 4) 1
568. The equilibrium constant for the reaction : Cu + 2Ag (𝑎𝑞) ⟶ Cu (𝑎𝑞) + 2Ag; 𝐸° = 0.46 V at 298 K is :
+ 2+

1) 2.0 × 1010 2) 4.0 × 1010 3) 4.0 × 1015 4) 2.4 × 1010


569. Quantity of charge is measured in :
1) ampere-sec. 2) ampere 3) ampere sec −1 . 4) amphere−1 sec.

P a g e | 43
570. The number of electrons involved in the reaction when a faraday of electricity is passed through an
electrolyte in solution is :
1) 12 × 1046 2) 96500 3) 8 × 1016 4) 6.02 × 1023
571. The unit of electrochemical equivalent is :
1) gram 2) Gram/ampere 3) Kg/coulomb 4) Coulomb/gram
572. For the cell reaction, Cu𝑐2 (𝑎𝑞) + Zn(𝑠) ⟶ Zn𝑐1 (𝑎𝑞) + Cu(𝑠), the change in free energy (∆𝐺) at a given
2+ 2+

temperature is a function of :
1) In 𝑐1 2) In (𝑐2 /𝑐1 ) 3) In (𝑐1 + 𝑐2 ) 4) In 𝑐2
573. Which of the following statements is not applicable to electrolytic conductors?
1) A single stream of electrons flows from cathode to anode
2) Show a positive temperature coefficient for conductance
3) New products show up at the electrodes
4) Ions are responsible for carrying the current
574. A metal having negative reduction potential when dipped in the solution of its own ions, has a tendency :
1) To pass into the solution
2) To be deposited from the solution
3) To become electrically positive
4) To remain neutral
575. The amount of silver deposited on passing 2 F of electricity through aqueous solution of AgNO3 is
1) 54 g 2) 108 g 3) 216 g 4) 324 g
576. The 𝐸𝑀3+/𝑀2+ values for Cr, Mn, Fe and Co are -0.41 V, +1.57 V, +0.77 V and +1.97 V respectively. For
°

which one of these metals the change in oxidation state from +2 to +3 is easiest?
1) Cr 2) Mn 3) Fe 4) Co
577. When Cu reacts with AgNO3 solution, the reaction takes place is
1) Oxidation of Cu 2) Reduction of Cu 3) Oxidation of Ag 4) Reduction of NO− 3
578. The specific conductivity of 0.1 N KCl solution is 0.0129 Ω−1 cm−1 . The resistance of the solution in the cell
100Ω. The cell constant of the cell will be
1) 1.10 2) 1.29 3) 0.56 4) 2.80
579. Which of the following statements is true for fuel cells?
1) They are more efficient 2) They are free from pollution
3) They run till reactants are active 4) All of the above
580. For the cell Zn|Zn || Cu |Cu, if the concentration of Zn2+ and Cu2+ ions is doubled, the e.m.f. of the cell :
2+ 2+

1) Doubles 2) Reduces to half 3) Remains same 4) Becomes zero


581. Given, standard electrode potentials
Fe2+ + 2𝑒 − → Fe, 𝐸° = −0.440 V
Fe3+ + 3𝑒 − → Fe, 𝐸° = −0.036 V
The standard electrode potential (𝐸 ° ) for
Fe3+ + 𝑒 − → Fe2+ is
1) + 0.771 V 2) − 0.771 V 3) + 0.417 V 4) − 0.417 V
582. During the electrolysis of molten NaCl solution, 230 g of sodium metal is deposited on the cathode, then
how many moles of chlorine will be obtained at anode?
1) 10.0 2) 5.0 3) 35.5 4) 17.0
583. The resistance of 0.5 𝑁 solution of an electrolyte in a conductivity cell was found to be 45 ohms. The
equivalent conductivity of the same solution is …..if the electrodes in the cell are 2.2 cm apart and have an
area of 3.8 cm2 .
1) 25.73 2) 30.75 3) 35.75 4) 15.75
584. The passage of electricity in the Daniell cell when Zn and Cu electrodes are connected:
1) From Cu to Zn inside the cell
2) From Cu to Zn outside the cell
3) From Zn to Cu outside the cell

P a g e | 44
4) None of the above
585. The number of coulombs required to reduce 12.3 g of nitrobenzene to aniline, is
1) 96500 C 2) 5790 C 3) 95700 C 4) 57900 C
586. Kohlrausch’s law states that at :
1) Infinite dilution each ion makes definite contribution to equivalent conductance of an electrolyte
depending on the nature of the other ion of the electrolyte
2) Infinite dilution, each ion makes definite contribution to conductance of an electrolyte whatever be the
nature of the other ion of the electrolyte
3) Infinite dilution, each ion makes definite contribution to equivalent conductance of an electrolyte,
whatever be the nature of the other ion of the electrolyte
4) Finite dilution, each ion makes definite contribution to equivalent conductance of an electrolyte,
whatever be the nature of the other ion of the electrolyte
587. How many kJ of energy is evolved, when a current of 2.00 A passes for 200 s under the potential of 230 V?
1) 56 kJ 2) 86 kJ 3) 36 kJ 4) 92 kJ
588. The standard electrode potential of Zn2+ /Zn andAg + /Ag are−0.763 V and+0.799 V respectively. The
standard potential of the cell is
1) 1.56 V 2) − 1.56 V 3) 0.036 V 4) − 0.036 V
589. When 1 F of electricity is passed through acidulated water, O2 evolved is
1) 1.0 dm3 2) 5.6 dm3 3) 11.2 dm3 4) 22.4 dm3
590. The charge required to liberate one gram equivalent of an element is
1) 96500 F 2) 1 F 3) 1 C 4) None of these
591. Small quantities of solutions of compounds 𝑇𝑋, 𝑇𝑌 and 𝑇𝑍 are put into separate test tubes containing X, Y
and Z solutions. 𝑇𝑋does not react with any of these. 𝑇𝑌reacts with both X and Z. 𝑇𝑍 reacts with X. The
decreasing order of ease of oxidation of the anions 𝑋 − , 𝑌 − , 𝑍 − is
1) 𝑌 − , 𝑍 − , 𝑋 − 2) 𝑍 − , 𝑋 − , 𝑌 − 3) 𝑌 − , 𝑋 − , 𝑍 − 4) 𝑋 − , 𝑍 − , 𝑌 −
592. The amount of silver deposited by passing 241.25C of current through silver nitrate solution is
1) 2.7 g 2) 2.7 mg 3) 0.27 g 4) 0.54 g
593. In Agalvanic cell, the electrons flow from
1) Anode to cathode through the solution 2) Cathode to anode through the solution
3) Anode to cathode through the external circuit 4) Cathode to anode through the external circuit
594. In an electrolytic cell, the anode and cathode are respectively represented as :
1) Positive electrode, negative electrode
2) Negative electrode, positive electrode
3) Both positive and negative electrode
4) None of the above
595. Which one of the following condition will increase the voltage of the cell represented by the equation?
Cu (𝑠) + 2Ag + (𝑎𝑞) ⇌ Cu2+ (𝑎𝑞) + 2 Ag (𝑠)
1) Increase in the dimension of Cu electrode 2) Increase in the dimension of Ag electrode
3) Increase in the concentration of Cu ions 2+ 4) Increase in the concentration of Ag + ions
596. The resistance of 0.01 𝑁 solution of an electrolyte was found to be 210 ohm at 298 K. Its conductance is :
1) 4.76 × 10−3 mho 2) 4.76 mho 3) 210 mho 4) None of these
597. Several blocks of magnesium are fixed to the bottom of Aship to
1) Keep away the sharks 2) Make the ship lighter
3) Prevent action of water and salt 4) Prevent puncturing by under- seArocks
598. Which of the following will form a cell with the highest voltage?
1) 0.1 M Ag + , 2 M Co2+ 2) 2 M Ag + , 2 M Co2+ 3) 1 M Ag + , 1 M Co2+ 4) 2 M Ag + , 0.1 M Co2+
599. The standard reduction potential of Zn and Ag in water at 298 K are,
Zn2+ + 2e− ⇌ Zn; E ° = −0.76 V and
Ag + + 𝑒 − ⇌ Ag ; 𝐸 ° = +0.80 𝑉. Which of the following reactions take place?
1) Zn2+ (aq) + 2Ag (s) → 2Ag + (aq) + Zn (s) 2) Zn (s) + 2Ag + (aq) → Zn2+ (aq) + 2Ag (s)

P a g e | 45
3) Zn2+ (aq) + Ag + (aq) → Zn (s) + Ag (s) 4) Zn (s) + Ag (s) → Zn2+ (aq) + Ag + (aq)
600. In electrolytic purification, which of the following is made of impure metal?
1) Anode 2) Cathode 3) Both (a) and (b) 4) None of these
601. When a quantity of electricity is passed through CuSO4 solution, 0.16 g of copper gets deposited. If the
same quantity of electricity is passed through acidulated water, then the volume of H2 liberated at STP will
be [Given , atomic weight of Cu = 64]
1) 4.0 cm3 2) 56 cm3 3) 604 cm3 4) 8.0 cm3
602. The number of electrons required to deposit 1 g atom of Al(at. wt. = 27) from a solution of AlCl3 are :
1) 1 𝑁 2) 2 𝑁 3) 3 𝑁 4) 4 𝑁
603. The cell reaction of the galvanic cell
Cu (𝑠) | Cu2+ (𝑎𝑞) || Hg 2+ (𝑎𝑞) | Hg (𝑙) is
1) Hg + Cu2+ ⟶ Hg 2+ + Cu 2) Hg + Cu2+ ⟶ Cu+ + Hg +
3) Cu + Hg → CuHg 4) Cu + Hg 2+ ⟶ Cu2+ + Hg
604. Specific conductivity of a solution
1) Increases with dilition 2) Decreases with dilution
3) Remains unchanged with dilution 4) Depends on mass of electrolyte
605. The standard reduction potential of the reaction,
H2 O + 𝑒 − ⟶ 1/2 H2 + OH at 298 K is
𝑅𝑇 𝑅𝑇
1) 𝐸 ° = Iln𝐾𝑤 2) 𝐸 ° = − In[PH 2 ]1/2 [OH − ]
𝐹 𝐹
𝑅𝑇 [𝑝𝐻 2 ]1/2 𝑅𝑇
3) 𝐸 ° = − In 4) 𝐸 °
= − In 𝐾𝑤
𝐹 [H + ] 𝐹
606. The correct order of the mobility of the alkali metal ions in aqueous solution is :
1) K + > 𝑅b+ > 𝑁a+ > 𝐿i+
2) Rb+ > K + > 𝑁a+ > 𝐿i+
3) Li+ > 𝑁a+ > K + > 𝑅b+
4) Na+ > K + > Rb+ > 𝐿i+
607. In a concentration cell :
1) Two electrodes are of different elements
2) Two electrolytic solutions of the same electrolyte but having different concentrations are used
3) Electrolyte of one strength but electrodes of two different concentrations are used
4) Both (b) and (c)
608. The degree of ionisation of weak electrolytes is influenced by :
1) Temperature
2) Concentration of electrolyte
3) Nature of solvent
4) All of the above
609. The units of equivalent conductivity is
1) S cm2 2) ohm cm2 (g − equivalent)
3) ohm cm 4) ohm−1 cm2 (g − equivalent)−1
610. ⋀∞ −1 2
ClCH2 COONa = 224Ω cm g equiv
−1

⋀∞ −1 2
NaCl = 38.5Ω cm g equiv
−1

⋀∞ −1 2
HCl = 203Ω cm g equiv
−1

What is the value of 𝜆ClCH2 COOH =?


1) 288.5Ω−1 cm2 g equiv−1 2) 289.5Ω−1 cm2 g equiv−1
3) 388.5Ω−1 cm2 g equiv−1 4) 59.5Ω−1 cm2 g equiv −1
611. Which one of the following has the highest molar conductivity?
1) Diaminedichloroplatinum (III) 2) Tetraaminedichlorocobalt (III) chloride
3) Potassium hexacyanoferrate (II) 4) Hexaaquochromium (III) bromide
612. Calculate the volume of H2 gas at NTP obtained by passing 4 A through acidified H2 O for 30 min is

P a g e | 46
1) 0.0836 L 2) 0.0432 L 3) 0.1672 L 4) 0.836 L
613. Which of the following is not correct?
1) Aqueous solution of NaCl is an electrolyte.
2) The units of electrochemical equivalent are g-coulomb.
3) In the Nernst equation, 𝑛 represents the number of electrons transferred in the electrode reaction.
4) Standard reduction potential of hydrogen electrode is zero volt.
614. What weight of copper will be deposited by passing 2 faraday of electricity through a solution of Cu(II)
salt?
1) 35.6 g 2) 63.5 g 3) 6.35 g 4) 3.56 g
615. How many faraday are needed to reduce a mole of MnO4 of Mn ?
− 2+

1) 4 2) 5 3) 3 4) 2
616. Given, the data at 25 ℃ ,
Ag + I − ⟶ AgI + e− ; 𝐸 ° = 0.152 𝑉
Ag → Ag + + e− ; 𝐸 ° = −0.800𝑉
What is the value of log 𝐾sp for AgI?
𝑅𝑇
(2.303 = 0.059 𝑉)
𝐹
1) - 8.12 2) +8.612 3) -37.83 4) -16.13
617. Chlorine cannot displace :
1) Fluorine from NaF 2) Iodine from NaI 3) Bromine from NaBr 4) None of these
618. The molar conductivity of acetic acid at infinite dilution is 390.7 and for 0.1 𝑀 acetic acid solution is
5.2mho cm2 mol−1. The degree of dissociation of 0.1 𝑀 CH3 COOH solution is :
1) 13.3% 2) 0.0133% 3) 1.33% 4) 133%
619. the amount of sodium deposited by 5 ampere current for 10 minute from fused NaCl is :
1) 0.715 g 2) 71.5 g 3) 5.17 g 4) 0.517 g
620. Standard reduction potential of an element is equal to :
1) + 1 × its reduction potential
2) − 1 × its standard oxidation potential
3) 0.00 V
4) + 1 × its standard oxidation potential
621. Molar conductance of electrolytic solution Λ 𝑚 is
1) ∝ 𝑙 2) ∝ (1/𝐴) 3) ∝ (1/𝐶) 4) ∝ (√𝐶)
622. The oxidation potential of Mg and Al are + 2.37 and + 1.66 volt respectively. The Mg in chemical reactions
:
1) Will be replaced by Al
2) Will replace Al
3) Will not be able to replace Al
4) None of the above
623. The emf of the cellMg| Mg 2+ (0.01 M)|| Sn2+ (0.1 M)|Sn at 298 K is (Given, 𝐸Mg °
2+ ,Mg =

°
−2.34 V, −2.34 V, 𝐸Sn2+ ,Sn = −0.14 V)

1) 2.23 V 2) 1.86 V 3) 1.56 V 4) 3.26 V


624. The cell reaction for the given cell is spontaneous if :
Pt(H2 )|H + (1𝑀)||H + (1𝑀)|Pt(H2 )
𝑃1 𝑃2
1) 𝑃1 > 𝑃2 2) 𝑃1 < 𝑃2 3) 𝑃1 = 𝑃2 4) 𝑃1 = 1 atm
625. The Gibbs energy for the decomposition of Al2 O3 at 500°C is as follows :
2 4
Al2 O3 ⟶ Al + O2 , ∆r 𝐺 = +966 kJ mol−1 .
3 3
The potential difference needed for electrolytic reduction of Al2 O3 at 500°C is atleast :
1) 5.0 V 2) 4.5 V 3) 3.0 V 4) 2.5 V
626. Reduction potential of four elements 𝑃, 𝑄, 𝑅, 𝑆 is −2.90, +0.34, +1.20and −0.76. Reactivity decreases in
P a g e | 47
the order
1) 𝑃 > 𝑄 > 𝑅 > 𝑆 2) 𝑆 > 𝑅 > 𝑄 > 𝑃 3) 𝑃 > 𝑆 > 𝑄 > 𝑅 4) 𝑄 > 𝑆 > 𝑅 > 𝑃
627. The standard oxidation potentials of Zn, Cu, Ag and Ni electrodes are + 0.76, -0.34,
-0.80 and +0.25 V respectively. Which of the following reaction will provide maximum voltage?
1) Cu + 2 Ag + (𝑎𝑞) → Cu2+ (𝑎𝑞) + 2 Ag 2) Zn + 2 Ag + (𝑎𝑞) → Zn2+ (𝑎𝑞) + 2 Ag
3) H2 + Ni2+ (𝑎𝑞) → 2H + (𝑎𝑞) + Ni 4) Zn + Cu2+ (𝑎𝑞) → Zn2+ (𝑎𝑞) + Cu
628. The factors which influence the conductance of solution.
1) Solute-solute interaction
2) Solute-solvent interaction
3) Temperature
4) All of the above
629. Agalvanic cell is constructed using the redox reaction,
1
H (𝑔) + AgCl(𝑠) ⇌ H + (𝑎𝑞) + Cl− (𝑎𝑞) + Ag (𝑠)
2 2
It is represented as
Pt | H2 (g) | HCl solution || AgNO3 solution |
1)
Ag
Ag | AgCl(s)|KCl solution | |HCl solution|
2)
| H2 (g) | Pt
3) Pt | H2 (g) | KCl solution || AgCl(s) | Ag
4) Pt | H2 (g) , HCl solution || AgCl(s) | Ag
630. In a cell that utilizes the reaction
Zn(𝑠) + 2H + (𝑎𝑞) → Zn2+ (𝑎𝑞) + H2 (g)
addition of H2 SO4 to cathode compartment will
1) Lower the 𝐸 and shift equilibrium to the right
2) Lower the 𝐸 and shift equilibrium to the left
3) Increase the 𝐸 and shift equilibrium to the right
4) Increase the 𝐸 and shift equilibrium to the left
631. Zn2+ ⟶ Zn(𝑠); 𝐸 ° = −0.76 𝑉
Cu2+ ⟶ Cu(𝑠); 𝐸 ° = −0.34 𝑉
Which of the following is spontaneous?
1) Zn2+ + Cu ⟶ Zn + Cu2+ 2) Cu2+ + Zn ⟶ Cu + Zn2+
3) Zn2+ + Cu2+ ⟶ Zn + Cu 4) None of the above

P a g e | 48
CV EDUCATION SOLUTIONS
NEET/JEE
Date :10/03/2019
Time : 10:31:00 CHEMISTRY
Marks : 2524
3.ELECTROCHEMISTRY

: ANSWER KEY :
1) 1 2) 1 3) 3 4) 3 161) 1 162) 1 163) 3 164) 3
5) 4 6) 2 7) 1 8) 3 165) 3 166) 3 167) 1 168) 4
9) 1 10) 2 11) 3 12) 1 169) 3 170) 1 171) 3 172) 1
13) 1 14) 4 15) 1 16) 2 173) 2 174) 1 175) 4 176) 1
17) 1 18) 3 19) 1 20) 3 177) 1 178) 1 179) 3 180) 1
21) 3 22) 3 23) 3 24) 3 181) 3 182) 1 183) 1 184) 1
25) 2 26) 2 27) 4 28) 4 185) 1 186) 2 187) 2 188) 2
29) 1 30) 4 31) 4 32) 4 189) 3 190) 3 191) 2 192) 2
33) 2 34) 4 35) 2 36) 1 193) 3 194) 1 195) 3 196) 1
37) 4 38) 4 39) 1 40) 2 197) 1 198) 1 199) 1 200) 1
41) 2 42) 1 43) 2 44) 3 201) 1 202) 2 203) 2 204) 1
45) 1 46) 3 47) 3 48) 4 205) 2 206) 1 207) 3 208) 3
49) 3 50) 2 51) 2 52) 1 209) 4 210) 4 211) 3 212) 3
53) 2 54) 4 55) 3 56) 1 213) 2 214) 2 215) 3 216) 1
57) 3 58) 1 59) 4 60) 1 217) 4 218) 1 219) 1 220) 4
61) 4 62) 4 63) 4 64) 1 221) 2 222) 3 223) 3 224) 2
65) 1 66) 1 67) 2 68) 3 225) 1 226) 1 227) 4 228) 3
69) 2 70) 4 71) 1 72) 3 229) 1 230) 2 231) 4 232) 4
73) 4 74) 4 75) 1 76) 1 233) 4 234) 4 235) 3 236) 2
77) 1 78) 2 79) 2 80) 2 237) 4 238) 4 239) 3 240) 1
81) 2 82) 2 83) 3 84) 3 241) 1 242) 4 243) 4 244) 1
85) 2 86) 3 87) 1 88) 2 245) 4 246) 3 247) 3 248) 2
89) 2 90) 3 91) 1 92) 2 249) 1 250) 1 251) 2 252) 1
93) 1 94) 3 95) 1 96) 1 253) 4 254) 1 255) 1 256) 1
97) 1 98) 3 99) 3 100) 3 257) 2 258) 3 259) 4 260) 1
101) 3 102) 3 103) 1 104) 1 261) 4 262) 4 263) 3 264) 1
105) 1 106) 4 107) 1 108) 1 265) 1 266) 1 267) 4 268) 2
109) 4 110) 2 111) 3 112) 3 269) 4 270) 2 271) 1 272) 3
113) 1 114) 4 115) 1 116) 4 273) 1 274) 3 275) 3 276) 4
117) 1 118) 1 119) 3 120) 3 277) 2 278) 4 279) 1 280) 4
121) 1 122) 3 123) 2 124) 1 281) 2 282) 1 283) 1 284) 1
125) 4 126) 1 127) 3 128) 4 285) 1 286) 1 287) 1 288) 2
129) 1 130) 1 131) 1 132) 3 289) 3 290) 3 291) 4 292) 1
133) 1 134) 2 135) 4 136) 1 293) 3 294) 3 295) 4 296) 1
137) 1 138) 1 139) 3 140) 2 297) 4 298) 1 299) 4 300) 3
141) 2 142) 1 143) 3 144) 2 301) 2 302) 4 303) 2 304) 4
145) 1 146) 4 147) 1 148) 3 305) 2 306) 1 307) 2 308) 2
149) 3 150) 3 151) 2 152) 4 309) 2 310) 3 311) 2 312) 4
153) 2 154) 1 155) 2 156) 4 313) 2 314) 3 315) 4 316) 1
157) 1 158) 1 159) 1 160) 3 317) 2 318) 2 319) 4 320) 1

P a g e | 49
321) 3 322) 3 323) 1 324) 1 525) 4 526) 4 527) 3 528) 4
325) 2 326) 3 327) 4 328) 2 529) 2 530) 1 531) 2 532) 3
329) 2 330) 1 331) 2 332) 3 533) 4 534) 4 535) 4 536) 2
333) 3 334) 2 335) 3 336) 4 537) 2 538) 2 539) 1 540) 2
337) 2 338) 2 339) 2 340) 1 541) 3 542) 3 543) 2 544) 1
341) 4 342) 1 343) 4 344) 4 545) 3 546) 2 547) 1 548) 3
345) 4 346) 4 347) 4 348) 1 549) 2 550) 1 551) 2 552) 1
349) 3 350) 4 351) 4 352) 3 553) 1 554) 1 555) 4 556) 3
353) 3 354) 1 355) 3 356) 3 557) 1 558) 4 559) 2 560) 4
357) 4 358) 3 359) 2 360) 3 561) 3 562) 4 563) 1 564) 1
361) 1 362) 4 363) 4 364) 1 565) 4 566) 2 567) 1 568) 3
365) 2 366) 1 367) 3 368) 1 569) 1 570) 4 571) 3 572) 2
369) 2 370) 2 371) 4 372) 4 573) 1 574) 1 575) 3 576) 1
373) 2 374) 2 375) 4 376) 4 577) 1 578) 2 579) 4 580) 3
377) 2 378) 3 379) 4 380) 2 581) 1 582) 2 583) 1 584) 2
381) 2 382) 2 383) 2 384) 1 585) 4 586) 3 587) 4 588) 1
385) 4 386) 1 387) 3 388) 3 589) 2 590) 2 591) 1 592) 3
389) 4 390) 2 391) 4 392) 1 593) 3 594) 1 595) 4 596) 1
393) 3 394) 3 395) 3 396) 2 597) 3 598) 4 599) 2 600) 1
397) 3 398) 1 399) 3 400) 3 601) 4 602) 3 603) 4 604) 2
401) 2 402) 1 403) 1 404) 3 605) 1 606) 2 607) 4 608) 4
405) 3 406) 1 407) 2 408) 1 609) 4 610) 3 611) 3 612) 4
409) 1 410) 1 411) 4 412) 1 613) 2 614) 2 615) 2 616) 4
413) 3 414) 4 415) 3 416) 2 617) 1 618) 4 619) 1 620) 2
417) 1 418) 4 419) 2 420) 3 621) 3 622) 2 623) 1 624) 1
421) 2 422) 1 423) 3 424) 3 625) 4 626) 3 627) 2 628) 4
425) 2 426) 2 427) 3 428) 2 629) 4 630) 3 631) 2
429) 1 430) 3 431) 2 432) 2
433) 1 434) 1 435) 1 436) 4
437) 1 438) 2 439) 3 440) 1
441) 4 442) 4 443) 4 444) 3
445) 2 446) 1 447) 1 448) 2
449) 1 450) 1 451) 1 452) 2
453) 4 454) 3 455) 2 456) 3
457) 4 458) 2 459) 1 460) 2
461) 1 462) 1 463) 4 464) 3
465) 1 466) 1 467) 2 468) 1
469) 1 470) 2 471) 1 472) 2
473) 1 474) 4 475) 1 476) 4
477) 3 478) 4 479) 4 480) 1
481) 1 482) 2 483) 2 484) 3
485) 2 486) 2 487) 2 488) 1
489) 3 490) 1 491) 1 492) 3
493) 1 494) 1 495) 3 496) 1
497) 2 498) 2 499) 2 500) 1
501) 2 502) 4 503) 2 504) 4
505) 2 506) 3 507) 1 508) 4
509) 3 510) 4 511) 2 512) 2
513) 4 514) 3 515) 3 516) 1
517) 1 518) 1 519) 4 520) 3
521) 4 522) 3 523) 3 524) 1
P a g e | 50
CV EDUCATION SOLUTIONS
NEET/JEE
Date :10/03/2019
Time : 10:31:00 CHEMISTRY
Marks : 2524
3.ELECTROCHEMISTRY

: HINTS AND SOLUTIONS :


1 (1) 9 (1)
° ° ° °
𝐸𝑂𝑃 for Li is more, 𝐸cell = 𝐸𝑂𝑃𝐿
+ 𝐸𝑅𝑃𝑅
= − 𝜙𝐿 + 𝜙𝑅 .
Li ⟶ Li+ + 𝑒 10 (2)
Thus, Li is strong oxidant. Ag + + 𝑒 ⟶ Ag
2 (1) 11 (3)
[H + ] = 𝑐. 𝛼 = 0.0133 × 0.1 Because conductance increases when the
= 0.00133 M. dissociation is more
3 (3) 12 (1)
Ag is below Cu in electrochemical series. Metal Zn has more +ve𝐸𝑂𝑃 °
and thus possesses more
placed above replaces other placed below in tendency to get oxidise and act as reducing agent.
series. 13 (1)
4 (3) ° 0.059 product
𝐸cell = 𝐸cell - 𝑛
log [reactant]
Ionisation depends upon concentration,
°
temperature, nature of solute and on nature of Given , 𝐸Cr 3+ /Cr = -0.74 V

°
solvent. Ionisation increases on increasing 𝐸Fe 2+ /Fe = -0.44 V

dilution. Cr | Cr 2+ (0.1 M) |Fe2+ | (0.01 M) / Fe


5 (4) ∴ Cr 3+ /Cr is anode andFe2+ / Fe is cathode.
𝑊 ∝ 𝐸;if𝑄 = constant. °
𝐸cell = 𝐸𝐶° - 𝐸𝐴°
6 (2) = (- 0.44) – (0.74)
Pb is above Ag and below Ni in electrochemical = - 0.44 + 0.74
series. = + 0.30 V
7 (1) Cell reaction is
∆𝐺 = ∆𝐻 − 𝑇∆𝑆, ∆𝐺 = −𝑛𝐸𝐹 and 2Cr + 3Fe2+ → 2Cr 3+ + 3Fe
𝜕(∆𝐺)
∆𝐺 = ∆𝐻 + 𝑇 [ ] number of electrons in cell reaction = 6
𝜕𝑇 𝑃 ° ° 0.059 product
𝐸𝑐𝑒𝑙𝑙 = 𝐸𝑐𝑒𝑙𝑙 - 𝑛
log [reactant]
𝜕𝐸
∴ ∆𝐻 = −𝑛𝐹 [𝐸 − 𝑇 ( ) ] 0.059 (Cr3+ )2
𝜕𝑇 𝑃 =+ 0.30 V - 6 log [(Fe2+)3 ]
𝜕𝐸
∴ ∆𝑆 = 𝑛𝐹 (𝜕𝑇 ) 0.059 (0.1)2
𝑃 = 0.30 - log [ ]
6 (0.01)3
8 (3) 0.059
= 0.30 - 6 log 104
For the given cell, 𝑀|𝑀+ || 𝑋 − | 𝑋, the cell reaction
0.059
is derived as follows : = 0.30 - 6 × 0.60
RHS :Reduction∶ 𝑋 + 𝑒 − → 𝑋 − …(i) = 0.30 – 5.9 × 10 −3

LHS :Oxidation∶ 𝑀 → 𝑀+ + 𝑒 − …(ii) = 0.2941 V


Add Eqs. (i) an (ii) 14 (4)
𝑀 + 𝑋 → 𝑀+ + 𝑋 − There will be no passage of current and ions will
The cell potential = − 0.11 V show simply diffusion.
Since, 𝐸cell = −ve, the cell reaction derived above 15 (1)
is non-spontaneous. In fact, the reverse reaction Smallest ion possesses maximum mobility.
will occur spontaneously 17 (1)

P a g e | 51
Given, current (𝑖) = 0.25 mA = 0.025 A 2.303𝑅𝑇 1
𝐸H = − log +
Time (𝑡) = 60 s 𝐹 [H ]
𝑄 = 𝑖 × 𝑡 = 60 × 0.025 = 1.5 C
= −0.059 pH = −0.059 × 3 = −0.177 V
1.5 × 6.023 × 1023
No. of electrons =
96500 28 (4)
𝑒 − = 9.36 × 1018 Pt(𝑠), H2 (𝑔) | H + (1 𝑀) || Ag + (𝑎𝑞) | Ag (𝑠)
Ca → Ca2+ + 2𝑒 − °
EMF of cell = 0.62 V, 𝐸cell = 0.80 𝑉
2𝑒 − are required to deposite one Ca atom + −
H2 → 2H + 2e (at anode)
9.36 × 1018 𝑒 − will be used to deposite
2Ag + + 2e− → 2Ag (at cathode)
9.36 × 1018
= = 4.68 × 1018 H2 + 2Ag + → 2 Ag + 2H + (overall reaction)
2
18 (3) 2.303𝑅𝑇 [H + ]2
𝐸cell = 𝐸 ° − log
0.059 [H + ]RHE 2𝐹 [Ag + ]2 [H2 ]
𝐸cell = log + 2.303𝑅𝑇 1
1 [H ]LHE
𝐸cell = 𝐸 ° − log
10−3 2𝐹 [Ag + ]2
= 0.059log = 0.059 × 5 = 0.295 V
10−8 2.303 × 0.06 1
20 (3) 0.62 = 0.80 + log
2 [Ag + ]2
° 0.059 [Ag + ]2 2 × 2.303 × 0.06
𝐸cell = 𝐸cell + log 0.62 = 0.80 + log[Ag + ]
2 [Sn2+ ] 2
21 (3) −0.18 = 0.1382 log [Ag + ]
𝐸cell = 𝐸𝑂𝑃𝐿 + 𝐸𝑅𝑃𝑅 =−𝐸𝑅𝑃𝐿 + 𝐸𝑅𝑃𝑅 . [Ag + ] = 0.05 m
100
22 (3) ∴ Mole of Ag + in 100 mL = 0.05 × 1000
H2 undergoes oxidation and AgCl(Ag + ) undergoes 100
reduction Wt. of Ag + in 100 mL = 0.05 × × 108
1000
% of Ag in 1.08 g alloy
23 (3) 0.05 × 100 × 108
In electrolytic cell, flow of electron is possible = × 100
1000 × 1.08
from cathode to anode through internal supply. = 50 %
25 (2) 30 (4)
Pure water is almost unionised so, it does not In concentration cell net redox change is zero and
conduct electricity. the decrease in free energy during transfer of
26 (2) matter is responsible for electrical work.
Current (𝑖) = 1.5 𝐴 31 (4)
Time (𝑡) 10 min = 10 × 60 = 600 s Rusting of iron is catalysed by [H + ]
Quantity of electricity passed 𝑄 = 𝑖 × 𝑡
= (1.5 𝐴) × (600 𝑠) 33 (2)
= 900 𝐶 Given,
Copper is deposited as Zn2+ → Zn , 𝐸 ° = − 0.76 𝑉
Cu2+ + 2𝑒 − ⟶ Cu(𝑠) Cu2+ → Cu , 𝐸 ° = 0.34 𝑉
2 moles of electrons or 2 × 96500 𝐶 of current Ag + → Ag , 𝐸 ° = 0.8 𝑉
deposit copper = 63.56 g Cell reaction of (I) is
Zn + Cu2+ → Zn2+ + Cu
900 C of current will deposit copper °
𝐸cell = 𝐸oxidation + 𝐸reduction
63.56 = + 0.76 + 0.34
= × 900 = + 1.10 V
2 × 96500
Cell reaction of (II) is
= 0.296 g Zn + Ag + → Zn2+ + Ag
°
𝐸cell = 0.76 + 0.8
27 (4)
= + 1.56 V
Reduction potential of hydrogen electrode,
Cell reaction of (III) is
Cu + Ag + → Cu2+ + Ag

P a g e | 52
°
𝐸cell = −0.34 + 0.8 𝑅𝑇 𝑝2
𝐸cathode = − ln + 2
= + 0.46 2𝐹 [H ]
°
So, the correct order of 𝐸cell of these cell is
𝑅𝑇 [H + ]2
II > I > III. 𝐸anode = − ln
2𝐹 𝑝1
34 (4)
2H + + 2𝑒 − → H2 𝐸cell = 𝐸anode + 𝐸cathode
According to Nernst equation,
0.0591 1 𝑅𝑇 [H + ]2 𝑅𝑇 𝑝2
𝐸 = 𝐸° + log + 2 =− ln − ln
𝑛 [H ] 2𝐹 𝑝1 2𝐹 [H + ]2
0.0591
𝐸 = 0 − log[H + ]2 𝑅𝑇 𝑝2 𝑅𝑇 𝑝1
2 =− ln = ln
= - 0.0591 pH 2𝐹 𝑝1 2𝐹 𝑝2
35 (2)
43 (2)
NaCl + KNO3 ⟶ NaNO3 + KCl
Cell is completely discharged, it means
KCl – NaCl
equilibrium gets established,
Molar conductivities 152 128
𝐸cell = 0
Difference = 24 S cm2 mol−1 Zn | Zn2+ (1 M) || Cu2+ (1 M) | Cu
Molar conductivities of KNO3 = 111
Cell reaction : Zn + Cu2+ ⇌ Zn2+ + Cu
Molar conductivities of NaNO3 = 111 – 24
[Zn2+ ]
= 87 S cm2 mol−1 𝐾eq =
[Cu2+ ]
36 (1)
° °
We know,
𝐸𝑂𝑃 of Mg > 𝐸𝑂𝑃 of Al. 0.0591
° °
37 (4) 𝐸cel𝑙 = 𝐸cell − log 𝐾eq
𝑛
𝜆∞ ∞
𝑐 = 𝜇𝑐 × Faraday; 0.0591
°
𝜆∞ ∞ 𝐸cell = log 𝐾eq
𝑎 = 𝜇𝑎 × Faraday; 𝑛

Also, 𝜆electrolyte = 𝜆∞ 𝑎 × 𝜆𝑐 .

Or 1.10 =
0.0591
log 𝐾eq
2
38 (4) [Zn2+ ] 2.20
1𝐹 = 𝑁 × 𝑒 = 𝐸 g = 96500 C 𝐾𝑒𝑞 = 2+
= antilog
[Cu ] 0.0591
39 (1) = antilog 37.3
° °
𝐸𝑜𝑝 of K >𝐸𝑂𝑝 of Al. 44 (3)
40 (2) For gold plating the electrolyte K [Au(CN)2 ] is
Eq. of H2 = Eq. of Cu used.
0.504 𝑊
∴ 1
= 63.5/2 45 (1)
∴ 𝑊𝐶𝑢 = 16 g Among Cu, Al, Fe and Zn, Al occupies highest
41 (2) position in electrochemical series.
Half-cell reaction is Al displaces Cu, Fe and Zn from their salts.
46 (3)
Cr2 O2−
7
+ −
+ 14H + 6𝑒 → 2Cr 3+
+ 7H2 O A thin film of Cr2 O3 is formed on Cr Surface.
47 (3)
°
𝐸Cr2 O2−
7 /Cr
3+ − 𝐸
Cr2 O2− / Cr3+ 𝐸° n𝐸 °
7

Mn2+ + 2e− → Mn -1.18 V -2.36


0.0591 [Cr 3+ ]2 3+ − 2+
=− log Mn + e → Mn 1.51 V 1.51 V
𝑛 [Cr2 O2−
7 ][H ]
+ 14
3+
Mn + 3e → Mn −
− 0.28 − 0.85
0.0591 48 (4)
= log[10−3 ]14 = −0.414 V According to Kohlrausch’s law
6
O
42 (1) ∥
Anode reaction H2 (𝑝1 ) → 2H + [NaOAc = CH3 C − O− Na+ ]
Λ∘CH3 COOH = Λ∘CH3 COO− + Λ∘H+ …(i)
Cathode reaction 2H + → H2 (𝑝2 )
Λ∘HCl = Λ∘H+ + Λ∘Cl− ..(ii)
P a g e | 53
Λ∘CH3 COONa = Λ∘CH3 COO− + Λ∘Na+ …(iii) 60 (1)
Thus, on adding (ii) and (iii), if Λ∘Na+
and Λ∘Cl− are MnO− −
4 will oxidise Cl ion according to equation
subtracted we can obtained the value of Λ∘HOAc . Mn7+ + 5𝑒 ⟶ Mn2+
Thus, additional value required is Λ∘NaCl . 2Cl− ⟶ Cl2 + 2𝑒
° ° °
49 (3) Thus, 𝐸cell = 𝐸𝑂𝑃Cl− /Cl
+ 𝐸𝑅𝑃 − 2+
2 MnO4 /Mn
Al → Al3+ + 3𝑒 − = −1.40 + 1.51 = 0.11 Vor reaction is
feasible MnO−4 will oxidise Fe
2+
to Fe3+
The charge required = 3 × 96500 C
Mn7+ + 5𝑒 ⟶ Mn2+
50 (2) Fe2+ ⟶ Fe3+ + 𝑒
° °
Cu is above Ag in electrochemical series and thus, ∴ 𝐸cell = 𝐸𝑂𝑃 2+ 3+
+ 𝐸MnO−4 /Mn2+
Fe /Fe
Cu + 2Ag + ⟶ Cu2+ + 2Ag reaction occurs. = −0.77 + 1.51
51 (2) = 0.74 V or reaction is feasible
Solid NaCl does not conduct electricity due to Thus, MnO− 2+
to Fe3+ in
4 will not oxidise Fe
absence of free ions. aqueous HCl medium but it will also oxidise Cl− to
52 (1) Cl2 .Suitable oxidant should not oxidise Cl− to
Transport number of an ion Cl2 and should only oxidiseFe2+ to Fe3+ in redox
current carried by that ion
= titrations.
total current carried by both the ions 61 (4)
53 (2) As Cr has maximum oxidation potential value,
𝜆∞ ∞
K+ = 𝜇K+ × F therefore its oxidation should be easiest
64.35
or𝜇K∞+ =
96500 62 (4)
= 6.67 × 10−4 cm2 sec −1 volt −1 𝑅 = 100Ω
54 (4) 1 𝑙
𝜅 = ( )
C is also calomel electrode. 𝑅 𝑎
𝑙
55 (3) (cell constant) = 1.29 × 100m−1
𝑎
1000
Λ°eq = κ × Given, 𝑅 = 520Ω; 𝐶 = 0.2 M
normality
µ (molar conductivity) =?
0.005 × 1000 𝜇 = 𝜅 × 𝑉
= = 500 ohm−1 cm2 equiv −1 1 𝑙
0.01 (𝜅 can be calculated as 𝜅 = ( ) now cell
𝑅 𝑎

56 (1) constant is known)


1 1000
pH of solution increases due to formation of LiOH Hence, µ = 520 × 129 × 0.2 × 10−6 m3
or due to increase in [OH − ] because H + ions are = 12.4 × 10−4 S m2 mol−1
discharged at cathode in preference to Li+. 63 (4)
57 (3) 3
° ° 2Fe + O2 ⟶ Fe2 O3
Calculate 𝐸cell for each. For (C)𝐸cell = 3.6 V. 2 Brown
58 (1) Cu + CO2 + H2 O ⟶ CuCO3 . Cu (OH)2
Green
Eq. of Al = Eq. of Na; Ag + H2 S ⟶ Ag 2 S
1.8 𝑊 Black
∴ 27/3 = 23
64 (1)
∴ 𝑊𝑁𝑎 = 4.6 g We know that, ∆𝐺 ° = −𝑛𝐹 . 𝐸 °
59 (4) Where, 𝑛 = 1 (number of transferred electron in
Salt bridge is used to remove or eliminate liquid the cell reaction)
junction potential arised due to different relative F =96500 C (Faraday’s constant)
speed of ions of electrolytes at the junction of two 𝐸 ° = ? (the standard electrode potential of cell)
electrolytes in an electrochemical cell. Thus, a salt ∆𝐺 ° = - 21.20 kJ (standard free energy)
bridge such as KCl is placed in between two ∴ -21.20 × 1000 J = - 1 × 96500 × 𝐸 °
electrolytes. A salt used for this purpose should 21200
Or 𝐸 ° = 96500 = 0.220 V
have almost same speeds of its cation and anion.
65 (1)
P a g e | 54
2H + + 2𝑒 ⟶ H2 ; 96500 × 60
= h
2OH − ⟶ H2 O + 1/2O2 + 2𝑒 20 × 5 × 60 × 60
66 (1) = 16.08 h
°
𝐸cell = 𝐸𝐶 − 𝐸𝐴 71 (1)
= - 0.45 – (-2.37) ° 0.0591
𝐸cell = log 𝐾𝑐
= -1.92 V 2
0.0591
Mg (s) | Mg 2+ (aq)𝑥 M || Fe2+ (aq) 0.01 M | Fe (s) 0.59 = log 𝐾𝑐
2
The cell reaction is 0.59 × 2
Mg + Fe2+ → Mg 2+ + Fe = log 𝐾𝑐
0.059
° 0.059 [Mg 2+ ] ∴ log 𝐾𝑐 =20
𝐸cell = 𝐸cell − log
𝑛 [Fe2+ ] 𝐾𝑐 = antilog 20 = 1020
0.059 𝑥 72 (3)
1.92 = 1.92 – log 1.6 𝑊
𝑛 0.01 Eq. of O2 = Eq. of Ag or = 108 ;
−0.059 𝑥 8
0 = log ∴ 𝑊Ag = 21.6 g
2 0.01
∴ 𝑥 = 0.01 M 73 (4)
67 (2) The substances which have lower reduction
Any cell (like fuel cell), works when potential potentials are stronger reducing agents. Hence,
difference is developed. the order of strength of reducing agent is
68 (3) increases in the following order
96500 C current produces 12 g Mg Cl− < Fe2+ < H2 < 𝑍𝑛
9.65 C current produces strength of reducing agent increases
12 × 9.65 74 (4)
= = 1.2 × 10−3 g Mg
96500 The metal placed below in electrochemical series
1.2 × 10−3 does not react with that metal salt solution which
= = 5 × 10−5 mol Mg
24 metal is placed above in series.
𝑅 — 𝑋 + 𝑀𝑔 ⟶ 𝑅𝑀𝑔𝑋 Grignard reagent 75 (1)
5 × 10−5 5 × 10−5 Cl2 + 2Kl ⟶ 2KCl + I2 ;iodine will be liberated
Mol mol first to impart violet colour to CHCl3 layer.
Hence, number of moles of 𝑅Mg𝑋 produced = 76 (1)
5 × 10−5 . Because in it covalent bonding is present
69 (2)
0.059 [H + ]2RHE 𝑃1 77 (1)
° °
𝐸cell = 𝐸𝑂𝑃 + 𝐸𝑅𝑃H + log Ionic conductance is ionic mobility × Faraday.
H
2 [H + ]2LHE 𝑃2
0.059 (10−3 )2 ° 78 (2)
°
= log (𝐸 = 𝐸𝑅𝑃 ) For all Zn | Zn2+ (A = 0.1 M) || Fe2+ (A =
2 (10−2 )2 𝑂𝑃
= − 0.059 V 0.01 M) | Fe.
Thus, cell reaction is non-spontaneous. The cell reaction,
70 (4) Zn(𝑠) → Zn2+ (𝑎𝑞) + 2𝑒 −
𝑤 = 60 g
Fe2+ (𝑎𝑞) + 2𝑒 − → Fe (𝑠)
𝑖 = 5A
atomic weight
Equivalent weight of Ca = Zn(𝑠) + Fe2+ (𝑎𝑞) → Zn2+ (𝑎𝑞) + Fe (𝑠)
valency
On applying Nernst equation,
40
= = 20
2 0.0591 [Zn2+ ]
°
According to first law of Faraday electrolysis 𝐸cell = 𝐸cell − log10
𝑛 [Fe2+ ]
equivalent weight
𝑤 = 𝑍𝑖𝑡 = ×𝑖 ×𝑡
96500 ° 0.0591 0.1
20 0.2905 = 𝐸cell − − log10
∴ 60 = ×5 ×𝑡 2 0.01
96500
96500 × 60 °
0.2905 = 𝐸cell − 0.0295 × log10 10
𝑡 = 𝑠
20 × 5
P a g e | 55
° (b) 1atm 1.0 M 1.0 0
0.2905 = 𝐸cell − 0.0295 × 1
(c) 2atm 1.0 M 2.0 -ve
°
∴ 𝐸𝑐𝑒𝑙𝑙 = 0.2905 + 0.0295 = 0.32 V (d) 2atm 2.0 M 0.50 +ve
°
𝐸red = 0.00 V for standard hydrogen electrode
At equilibrium (𝐸𝑐𝑒𝑙𝑙 = 0) If 𝑄 > 1, then 𝐸𝑟𝑒𝑑 = -ve.
0.0591 Thus, correct answer is (c).
°
𝐸cell = 𝐸cell − log10 𝐾𝑐 85 (2)
𝑛
°
𝐸𝑀 2+ /𝑀 values follow the order with negative sign
° 0.0591
∴ 0 = 𝐸cell − 𝑛
log10 𝐾𝑐 Mn < 𝐶𝑟 < 𝐹𝑒 < 𝐶𝑜
°
0.0591
𝐸𝑅𝑃 -1.18 -0.74 -0.44 -0.27 V
°
or𝐸cell = 2
log10 𝐾𝑐 86 (3)
MnO− 4 is itself a strong oxidant.
0.0591
0.32 = log10 𝐾𝑐 87 (1)
2 ° ° °
𝐸cell = 𝐸𝑂𝑃 Zn
+ 𝐸𝑅𝑃 Ag
= 0.76 + 0.77 = +1.53 V.
or 𝐾𝑐 = 100.32/0.0295 88 (2)
The hydrated ion size is Li+ + +
aq > 𝑁a aq > K aq >
79 (2)
𝑅b+aq . Larger is ion, lesser is its mobility.
Λ∞ ∞ ∞
eq (NH4 OH) = Λ eq (NH4 Cl) + Λ eq (NaOH)
89 (2)
− Λ∞eq (NaCl)
Ag / Ag + (0.1 M) || Ag + (1 M) / Ag
= (149.74 + 248.1 – 126.4) anode cathode
= 271.44 Ω−1 cm2 eq−1
2.303𝑅𝑇 𝐶
80 (2) 𝐸𝑐𝑒𝑙𝑙 = log 1
𝑛𝐹 𝐶2
Given, product = 0.1 M and reactant = 1 M
0.0591 [products]
°
𝐸cell °
= E𝑐𝑒𝑙𝑙 − log Here, 𝑛 = number of electrons in cell reaction = 1
2 [reactants] 𝐶1 = concentration of cathodic electrolyte = 1 M
° 0.1
1.50 = 𝐸cell − 0.02955 log [ ] 𝐶2 = concentration of anodic electrolyte = 0.1 M
1 0.059 1
°
𝐸cell = 1.470 𝑉 ∴ 𝐸𝑐𝑒𝑙𝑙 = 1
log 0.1
°
𝐸cell = 𝐸H° + /H2 − 𝐸𝑀
° 𝐸𝑐𝑒𝑙𝑙 = 0.059 V
/𝑀 2+
90 (3)
𝐸°𝑀 /𝑀2+ = − 1.470 speed of ions
° Ionic mobility =
So,𝐸𝑀 2+ /𝑀 = 1.470 𝑉 pot.gradient

81 (2) 91 (1)
It is definition of equivalent conductivity. The tendency to gain electron is in the order 𝓏 >
82 (2) 𝑦>𝑥
H + is lightest ion and thus, possesses maximum Thus, 𝑦 + 𝑒 − → 𝑦 −
velocity among monovalent ions.
𝑥 → 𝑥− + 𝑒−
84 (3)
Reduction hydrogen half-cell is 92 (2)
H + | (𝑥𝑀) | Pt (H2 ) Cell reaction will be
+ Br2 + Sn2+ → 2Br − + Sn4+ , 𝐸 ° = 0.95 V
Pressure𝑝H2 ° 0.059
𝐸cell = log 𝐾eq
Half – cell reaction is 2
2H + (𝑎𝑞) + 2e− ⟶ H2 (g) 0.059
0.95 = log 𝐾eq
Reaction quotient 2
0.95 × 2
= 𝒬 = 𝑃𝐻2 /[H + ]2 , 𝑛 = 2 = log 𝐾eq
0.0591 0.0591 0.059
𝐸red = 𝐸red°
− log 𝒬 = 0 − log 𝒬 𝐾eq ≈ 1032
𝑛 2
𝑷𝑯𝟐 [𝐇 + ] 𝓠 𝑬𝐫𝐞𝐝 93 (1)
° °
(a) 1atm 2.0 M 0.25 +ve The given values are𝐸𝑅𝑃 . More is 𝐸𝑅𝑃 more is the

P a g e | 56
tendency to gain electron or to show reduction or = number of equivalents of copper formed
to show strong oxidant nature. In AgNO3 , Ag is in+1 oxidation state
94 (3) In CuSO4 , Cu is in+2 oxidation state
° ° ° 108
𝐸cell = 𝐸𝑂𝑃Sn
+ 𝐸𝑅𝑃Fe
= 0.14 + 0.77 = 0.91 V Equivalent weight of Ag = = 108
1
95 (1) Equivalent weight of Cu =
63.6
= 31.8
𝐸×𝑖×𝑡 2
𝑊= 𝑤1 𝐸1
96500 =
63.5 ×𝑖×𝑡 𝑤2 𝐸2
or 0.635 = 2 ×96500
10.79 108
∴ 𝑖 × 𝑡 = 1930 C ∴ =
𝑤Cu 31.8
96 (1) 10.79×31.8
Or𝑤Cu = = 3.2 g
Cu2+ + 2e− → Cu, 𝐸 ° = 0.34 108
102 (3)
Zn2+ + 2e− → Zn, 𝐸 ° = 0.76
2H + + 2𝑒 ⟶ H2 (cathode)
In the cell,
2OH − ⟶ H2 O + (1/2)O2 + 2𝑒 (Anode)
Cu | Cu2+ || Zn2+ | Zn
anode cathode
103 (1)
∞ ∞
In the cell , Λ∞ ∞
CH3 COOH = Λ CH3 COONa + Λ HCl − Λ NaCl
°
𝐸cell °
= 𝐸cathode °
− 𝐸anode 104 (1)
°
= 0.76 – (-0.34) As ‘𝐴’ has more 𝐸red value than 𝐵, 𝐴 will act as
= 1.10 V cathode in the galvanic cell.
98 (3) Hence,
° ° °
The cell reaction is 𝐸cell = 𝐸cathode - 𝐸anode
= (2.23) – (-1.43)
H2 (g) + I2 (𝑠) ⇌ 2H + (𝑎𝑞) + 2I − (𝑎𝑞) = 2.23 + 1.43 = 3.66 V
[H + ]2 [I− ]2 105 (1)
0.0591
0.7714 = 0.535 − log Metal undergoes oxidation during corrosion and
2 pH2
thus, act as anode.
∴ pH = 3 106 (4)
Oxidation half-cell
99 (3)
Given, ClO− − −
3 → ClO2 + 2𝑒 ; 𝐸°𝑐𝑒𝑙𝑙 = −0.36 V

Fe3+ + 3e− ⟶ Fe; 𝐸1° = - 0.036 V ….(i)


°
Reduction half-cell
2+ −
Fe + 2e ⟶ Fe; 𝐸2 = - 0.439 V ….(ii)
We need to calculate ClO− − −
3 + 2𝑒 → ClO2 ; 𝐸°𝑐𝑒𝑙𝑙 = 0.33 V
3+ − 2+ °
Fe + e ⟶ Fe E3 = ? …..(iii)
𝑅𝑇
We can obtain the (III) by subtracting II from I but 𝐸°cell = 0.33 − 0.36 = −0.03 = ln 𝐾
2𝐹
𝐸3° , we can not obtain that way because electrode
potential is intensive property. That’s when we 0.059
or −0.03 = log 𝐾or𝐾 = 0.1
2
determine 𝐸3° calculating
∆𝐺3 = ∆𝐺1 − ∆𝐺2 2ClO− −
3 ⇌ ClO4 + ClO2

(△G is an extenwise property)


∆𝐺3 = 3 × 0.036 𝐹 – 2 × 0.439 𝐹 0.1 − 2𝑥𝑥 𝑥
∆𝐺3 = 0.108 𝐹 – 0.878 𝐹
°
𝑥2
−1 × 𝐹 × 𝐸3 = −0.770 𝐹 = 0.1
°
(0.1 − 2𝑥)2
𝐸3 = 0.770 𝐹
100 (3) or𝑥 = 1.9 × 10−2
The number of ions per cc decreases with dilution
and therefore, specific conductance decreases 107 (1)
with dilution. During the electrolysis of an aqueous solution of
101 (3) MgSO4 between inert electrodes, the products
Number of equivalents of silver formed available on the cathode and the anode are H2 (g)

P a g e | 57
and O2 (g) respectively. liberate H2 from
108 (1) HCl because 𝐸𝑂𝑃°
Cu
°
> 𝐸𝑂𝑃Ag
°
and 𝐸𝑂𝑃Cu
= −ve.
96500C or 1 Faraday charge is required for the 121 (1)
deposition of 1 g-equivalent of a substance. 96500 C or 1𝐹 will liberate 1 eq. of O2 or 1/4 mole
109 (4) O2 or 5.6 litre O2 at NTP.
°
1. ∆𝐺 ° = −𝑛𝐹𝐸cell 122 (3)
2.303 𝑅𝑇 Rust is Fe2 O3 and Fe(OH)3 .
°
2. 𝐸𝑐𝑒𝑙𝑙 = 𝑛𝐹
log 𝐾𝑐 123 (2)
Weight of Ag required = 80 × 5 × 10−3 ×
3. 𝑘 = 𝐴𝑒 −𝐸𝑎/𝑅𝑇
1.05(wt. = 𝑣 × 𝑑)
110 (2) = 0.42 g
𝐸𝑖𝑡
𝐸 × 𝑖 × 𝑡 𝐸 × 3 × 50 × 60 ∵ 𝑊 = 96500
𝑤metal = =
96500 96500 ∴ 0.42 =
108 × 3 × 𝑡
96500
96500 × 𝑤 96500 × 1.8 ∴ 𝑡 = 125 sec
∴𝐸= = = 19.3
3 × 50 × 60 3 × 50 × 60 124 (1)
1 faraday deposits 1 equivalent whichis also 1 g
111 (3)
° ° ° atom for Na.
𝐸cell = 𝐸cathode − 𝐸anode
° °
125 (4)
𝐸Ag+ /Ag − 𝐸Cu+ /Cu ° °
𝐸° for reaction in (d) = 𝐸𝑂𝑃Br
+ 𝐸𝑅𝑃1
= −1.09 +
= - 0.80 – 0.34 (−0.54)
= + 0.46 = −1.63 V
112 (3) Since, 𝐸° is negative and thus, reaction is non-
At cathode, spontaneous.
Al3+ + 3𝑒 − → Al 126 (1)
1 0.8
Cell constant = = = 0.66 cm−1 .
𝑎 1.2
27
𝐸Al = =9 127 (3)
3
CuSO4will react with elements placed above it in
𝑤Al = 𝐸Al × no.of faradays electrochemical series and it would not react with
elements places below it in electrochemical series.
= 9 × 0.1 = 0.9 g ∴ CuSO4 reacts with Zn and Fe placed above it.
CuSO4 + Fe → FeSO4 + Cu
113 (1)
CuSO4 + Zn → ZnSO4 + Cu
1 faraday involves charge of 1 mole electrons.
∴ CuSO4 does not react with Ag, placed below it.
114 (4)
CuSO4 + Ag → no reaction.
It does not depend upon mass.
128 (4)
116 (4)
Faraday’s laws are independent of all other
The electrode, which shows colour change during
external factors and 𝑊 ∝ 𝑄.
redox process is called indicator electrode.
129 (1)
117 (1)
The ionic hydrides conduct electricity in molten
Cathode2H2 O + 2𝑒 − → H2 + 2OH −
1 state and liberate H2 anode.
Anode : H2 O → 2H + + O2 + 2𝑒 − CaH2 + 2H2 O ⟶ Ca(OH)2 + 2H2 ↑
2
118 (1) 130 (1)
No doubt Be is above Mg in periodic table but it is 2H + 2𝑒 ⟶ H2 ;
below Mg in electrochemical series. 2OH − ⟶ H2 O + 1/2O2 + 2𝑒
119 (3) ∴ Wt. ratio of H2 ∶ O2 = 2 ∶ 16 or 1 ∶ 8
The process of zinc-plating on iron-sheet is 131 (1)
known as galvanization. Cr/Cr 3+ (0.1 M) || Fe2+ (0.01 M) | Fe
120 (3) Oxidation half-cell; Cr → Cr 3+ + 3e− × 2
Cu can displace Ag from AgNO3 but it cannot Reduction half-cell; Fe2+ + 2𝑒 − → Fe × 3

P a g e | 58
Net cell reaction; 𝐸×𝑖×𝑡
𝑊=
2Cr + 2Fe2+ → 2Cr 3+ + 3Fe (𝑛 = 6) 96500
° ° ° 𝐸 × 3 × 50 × 60
𝐸cell = 𝐸oxidation − 𝐸reduction ∴ 1.8 = 96500
= 0.72 – 0.42 ∴ 𝐸 = 19.3
= 0.30 V 133 (1)
3+ 2
° ° 0.0591 [Cr ] Λ
𝛼 = Λ∞𝑣 =
15.8
= =0.04514
𝐸cell = 𝐸cell − log 350
𝑛 [Fe2+ ]3
134 (2)
0.0591 (0.1)2
= 0.30 − log It is the definition of conductivity.
6 (0.01)3
0.0591 10−2
= 0.30 − log −6
6 10
0.0591
= 0.30 – log 104
6
𝐸cell = 0.2606 𝑉
132 (3)
135 (4)
Fe3+ + 3𝑒 ⟶ Fe; −∆𝐺1° = −0.36 × 𝐹 × 3
Fe2+ + 2𝑒 ⟶ Fe; −∆𝐺2° = −0. 439 × 𝐹 × 2
⎯ ⎯ ⎯ +
Fe3+ + e ⟶ Fe2+ ; −∆𝐺° = [3 × (−0.36𝐹) + 2 × (0.439𝐹)]
∴ +1 × 𝐹 × 𝐸° = [3 × (−0.36𝐹) + 2 × (0.439 𝐹)]
or 𝐸° = [3 × (−0.36) + 2 × 0.439]V
136 (1) ° 0.059
0 −3 𝐸cell = log 𝐾𝑐
0
𝜆 5 × 10 −8
2
𝑢 = = = 5.18 × 10 °
𝐸cell ×2
96500 96500
log 𝐾𝑐 =
137 (1) 0.059
In voltaic cells net redox change brings in passage 0.47 × 2
log 𝐾𝑐 =
of current. In concentration cells transfer of 0.059
matter brings in passage of current. 𝐾𝑐 = 8.5 × 1015
138 (1) 141 (2)
° In case of very dilute solution of NaCl, electrolysis
Given, 𝐸Zn /Zn2+ = 0.76 V
°
brings in the following changes :
𝐸Cu /Cu2+ = 0.34 V 1
Anode : 2OH − ⟶ H2 O + 2 O2 + 2𝑒
∴ Zn is anode (∵ It has higher oxidation
Cathode : 2H + + 2𝑒 ⟶ H2
potential)
° 142 (1)
∴ 𝐸Zn 2+ /Zn = − 0.76 𝑉
MnO− +
4 + 8H + 5𝑒 → Mn
− 2+
+ 4H2 O 𝐸 ° = 1.51 V
°
and 𝐸Cu2+ /Cu = − 0.34 𝑉
° ° ° ∆𝐺1 ° = −5(1.51)F = −7.55F
𝐸cell = 𝐸cathode − 𝐸anode
= - 0.34 V – ( - 0.76 V) MnO2 + 4H + + 2𝑒 − → Mn2+ + 2H2 O𝐸 ° = 1.23 V
= 0.34 V + 0.76 V
= 0.42 V ∆𝐺2 ° = −2(1.23)F = −2.46 V
139 (3)
On substrating
2𝑒 + Cl+ ⟶ Cl− ; ∆G1° ….(i)
2Cl− ⟶ 2Cl2 + 2𝑒; ∆𝐺2° ….(ii) MnO− + −
4 + 4H + 3𝑒 → MnO2 + 2H2 O ∆𝐺3 °
2𝑒 + 2Cl+ ⟶ Cl2 ; ∆𝐺3° = −5.09 F
∴ ∆𝐺3° = 2 × ∆𝐺1° + ∆𝐺2°
∆𝐺3 ° −5.09 𝐹
−2 × 𝐸3° × F = −2 × 2 × 0.94 𝐹 − 2 × (−1.36) °
𝐸MnO − | MnO = = = 1.70 V
×𝐹
4 2 −𝑛𝐹 −3𝐹
∴ 𝐸3° = 0.52 V 143 (3)
140 (2) 2Fe3+ + Zn → Zn2+ + 2 Fe2+
2Fe3+ + 2e− → 2 Fe2+
P a g e | 59
According to Nernst’s equation, 154 (1)
° ° 𝑅𝑇 [Fe2+ ]2 In electrochemical series, iron is placed below
𝐸cell = 𝐸cell − In
𝑛𝐹 [Fe3+ ]2 sodium, so it cannot displace sodium from its salt
𝐸cell is depend on concentration of Fe2+ . solution. Hence, no reaction takes place.
Therefore on increasing the concentration of Fe + Na3 PO4 → No reaction
[Fe2+ ]𝐸cell decreases. 155 (2)
144 (2) 2MnO2 (𝑠) + Zn2+ + 2𝑒 − ⟶ ZnMn2 O4 (𝑠)
°
0.0591 2 × 87 g MnO2 required 2 × 96500 F charge
𝐸 = 𝐸RP + log[𝑀+ ] 2 × 96500 × 8
𝑛
8 g MnO2 =
Given, 2 × 87
°
𝐸RP = − 2.36 V, [𝑀+ ] = 0.1 𝑀
= 8873.56 F
𝑛 = 1 (for𝑀+ → 𝑀 )
°
0.0591 𝒬 = 𝑖𝑡
𝐸 = 𝐸𝑅𝑃 + log[𝑀+ ]
𝑛
0.0591 8873.56 = 2 × 10−3 × 𝑡
= −2.36 + log 0.1
1
= − 2.36 + 0.0591 × (−1) 8873.56 × 103
𝑡 =
= − 2.36 – 0.0591 2
= − 2.419 𝑉
= 4436781.5 s
145 (1)
1 faraday charge = 𝐸 g deposition. 4436781.5
146 (4) =
24 × 60 × 60
Λ̊ NaBr = Λ̊ NaCl + Λ̊ KBr − Λ̊ KCl
= 51.35 days
147 (1)
This is Kohlrausch law for 𝐴𝑥𝐵𝑦. 156 (4)
148 (3) All are electrolytic cells.
For the change , 2Fe3+ + 3I − ⇌ 2Fe2+ + I3− 157 (1)
°
𝐸cell = 0.77 – 0.54 = 0.23 V Λ∞ ∞
m for BaCl2 = Λ m Ba
2+
+ 2Λ∞m Cl

0.059 ∴ Λ eq for BaCl2 = 1/2Λm Ba + Λ∞
∞ ∞ 2+ −
𝐸° = log 𝐾𝑐 m Cl
2 = 127 / 2 + 76
0.0591
0.23 = log 𝐾𝑐 = 139.5 Ω−1 cm2
2
𝐾𝑐 = 6.26 × 107 158 (1)
149 (3) At cathode 2H + (𝑎𝑞) + 2𝑒 − → 2H
1 1 1 1.8
𝑘 = 𝑅 × 𝑎 = 32 × 5.4 = 0.0104 1
2H + O2 → H2 O
And 𝜆 = 𝑘 × 𝑉 = 0.0104 × 10,000 = 104 2
150 (3)
° °
________________________________
𝐸𝑂𝑃 of Mg > 𝐸𝑂𝑃 of Zn
151 (2) 1
2H + + O2 + 2𝑒 − → H2 O
Standard hydrogen electrode (SHE) is reference 2
electrode. It is used to determine the electrode
________________________________
potential of any half cell. The electrode potential
of any standard hydrogen electrode is arbitrarily 159 (1)
taken as zero. The variation of e.m.f. of cell with temperature at
152 (4) constant pressure (𝜕𝐸/𝜕𝑇)𝑃 is referred as
° ° °
𝐸cell = 𝐸𝑂𝑃 Sn
+𝐸𝑅𝑃 Fe
= 0.14 + (⎯0.44) temperature coefficient of cell.
= −0.30 V 160 (3)
153 (2) Electrolysis of water takes place as follows
𝐾𝑎 = 𝑐𝛼 2 for weak acid; H2 O ⇌ H+ + OH −
H𝐴 ⇌ H + + 𝐴− Cathode anode
At anod
P a g e | 60
oxidation
OH − →⎯⎯⎯⎯⎯⎯⎯ OH + e− Λ𝑣 Λ0
∴ 𝛼= 0= = 0.01
4OH ⟶ 2H2 O + O2 Λ 100Λ0
167 (1)
At cathode
Reduction Magnesium is more electropositive than zinc,
2H + + 2e− →⎯⎯⎯⎯⎯⎯⎯ H2 hence it can reduce Zn2+ (magnesium is placed
Given, time, 𝑡 = 1930𝑠 above zinc in electrochemical series).
Number of moles of hydrogen collected Mg + ZnO → MgO + Zn
1120 × 10−3 168 (4)
= moles
22.4 In presence of attacked electrodes (Ni electrodes
= 0.05 moles with Ni salt), the metal gets dissolved at anode
∵ 1 mole of hydrogen is deposited by = 2 moles and metal ions get discharged at cathode.
of electrons 169 (3)
∵ 0.05 moles of hydrogen will be deposited by Weak electrolytes are 100% ionized at infinite
= 2 × 0.05 dilution.
= 0.10 mole of electrons 171 (3)
Charge, 𝒬 = 𝑛𝐹 The ions which lies below H + in electrochemical
= 0.1 × 96500 series, displace H + from solution.
Charge,𝒬 = 𝑖𝑡 ∵ Cu2+ lies below H + in electrochemical series.
0.1 × 96500 = 𝑖 × 1930 ∴ Cu2+ions displace H + when H2 gas is
0.1 × 96500 bubbled in solution containing these ions.
𝑖= 172 (1)
1930
𝜅 = Λ eq ∙ 𝐶
= 5.0 A
2.54
161 (1) = (91 Ω−1 cm2 eq−1 ) ( eq ∙ cm−3 )
159/2 × 1000
At cathode2H + + 2𝑒 − → H2
1
At anode2OH − → H2 O + O2 + 2𝑒 − = 2.9 × 10−3 Ω−1 cm−1
2
162 (1) 173 (2)

Λ∞ ∞
H2 O = 𝜆H+ + Λ OH− At cathode, cations are reduced. Also discharge
163 (3) potential of H + is less than Na+.
For the cell reaction, Fe acts as cathode and Sn as 174 (1)
anode 𝑊 ∝ 𝑖 × 𝑡and 𝑊 = 𝑍 × 𝑖 × 𝑡.
175 (4)
Hence, ° °
𝐸𝑅𝑃 2− 3+
= 1.33 V and 𝐸𝑅𝑃 3+ 2+
= 0.77 V
Cr2 O7 /Cr Fe /Fe
° ° °
𝐸cell = 𝐸cathode − 𝐸anode ∴ °
𝐸𝑂𝑃 is more thus it will oxidise or
Fe2+ /Fe3+

= −0.44 − (−0.14) = −0.30 V electron will flow from Fe electrode to Cr


electrode. Also Fe electrode will be negative. Also.
The negative emf suggests that the reaction goes °
𝐸cell °
= 𝐸𝑂𝑃 °
+ 𝐸𝑅𝑃 = −0.77 + 1.33
Fe Cr
spontaneously in reversed direction 176 (1)
164 (3) Velocities of both K + and NO− 3 are nearly the

𝐸 ×𝑖×𝑡 same in KNO3 , so it is used to make salt-bridge


𝑊= 177 (1)
96500

𝐸×𝑐×𝑡
𝑃 = 96500 Given, 𝑖 = 2.5𝐴
𝑡 = 6 min 26 𝑠 = 6 × 60 + 26 = 386𝑠
165 (3)
Number of coulomb passed = 𝑖 × 𝑡
H + gets discharged at cathode and thus, [OH − ]
= 2.5 × 386
increases in solution.
= 965 C
166 (3)
Cu + 2𝑒 − ⟶ Cu
2+
Λ0
Λ𝑣 = ∴ 2 × 96500 C charge deposits Cu = 63.5 g
100

P a g e | 61
∴ 965 C charge deposits ∆𝐺° = −2.303 𝑅𝑇 log 𝐾𝑐
63.5 ∴ 𝑛𝐹𝐸° = 2.303 𝑅𝑇 log 𝐾𝑐
Cu = × 965
2 × 96500 𝑛𝐹𝐸°
= 0.3175 g log 𝐾 𝑐 =
2.303 𝑅𝑇
178 (1) 2 × 96500 × 0.295
𝑢𝑐 𝑢𝑎 =
𝑡𝑐 = ,𝑡 = 2.303 × 8.314 × 298
𝑢𝑎 + 𝑢𝑐 𝑎 𝑢𝑎 + 𝑢𝑐 log 𝐾𝑐 = 9.97
Where, 𝑢𝑎 and 𝑢𝑐 are speed of ion and 𝑡𝑐 and 𝑡𝑎 ∴ 𝐾𝑐 = 1 × 1010
are transport number of cation and anions 188 (2)
respectively of an electrolyte. Fe2+ + 2𝑒 − → Fe
Thus, 𝑡𝑐 + 𝑡𝑎 = 1
56
179 (3) 𝐸Fe = = 28
Eq. of Al deposited = 5 2
27
∴ wt.of Al = 5 × = 45 g 𝑤Fe = EFe × number of faraday
3
180 (1)
= 28 × 3 = 84 g
𝐸×𝑖×𝑡
𝑊=
96500 189 (3)
112 × 2 1 × 𝑖 ×965
∴ 22400 = 96500 Eq. of Al formed
𝑊 𝑖𝑡
= 96500
𝐸
∴ 𝑖 = 1 ampere 4 × 104 × 6 × 60 × 60 × 27 27
181 (3) ∴ 𝑊Al = 96500
(𝐸 = 3
)
4
Hg is placed below H in electrochemical series. ∴ 𝑊Al = 8.05 × 10 g
182 (1) 190 (3)
𝑄 = 2.5 × 386 = 965 C 6 × 1023 electron = 1 eq.
2F(2 × 96500 C) deposited, Cu = 63.5 g 191 (2)
Hence, 965 C will deposit, Cu = 0.3175 g Λ 𝑚 = Λ 𝑒𝑞 .× valency factor
183 (1) = 314.28 × 2 = 628.56 mho cm2 mol−1
Eq. of Al = Eq. of Ag 192 (2)
𝑊Al 𝑊Ag 𝑊 9 ° °
∴ = or 𝑊 Al = 𝐸𝑂𝑃 of Fe > 𝐸𝑂𝑃 of Cu;
9 108 Ag 108
Thus, Fe gets oxidised or Fe ⟶ Fe2+ + 2𝑒;
184 (1)
Cu2+ + 2𝑒 ⟶ Cu
Galvanic cell is Aelectrochemical cell that converts
193 (3)
the chemical energy of Aspontaneous reaction
According to Faraday law, number of ions
into electrical energy.
produced ∝ quantity of electricity passed
185 (1)
194 (1)
Anode : Fe(𝑠) + 2OH − ⟶ FeO(𝑠) + H2 O(𝑙) + 2𝑒 0.059
Cathode : Ni2 O3 + H2 O(𝑙) + 2𝑒 ⟶ 2NiO(𝑠) + 𝐸° = 𝑛 log𝐾𝑒𝑞 and ∆𝐺° = −𝑛𝐸°𝐹
2OH − ∴ ∆𝐺° = +ve, 𝐸° will be − ve and𝐾𝑒𝑞 < 1;one
[H2 O] should not write ∆𝐺° > 0.
° 0.059
𝐸cell = 𝐸𝑂𝑃Fe /FeO − log10 [OH ]
− 2 195 (3)
2
The degree of dissociation of all electrolyte
°
+ 𝐸𝑅𝑃 Ni2 O3 /NiO increases with increase in dilution
0.059 [𝐻2 𝑂] (or decrease in concentration).
+ log10
2 [𝑂𝐻 ]− 2 196 (1)
°
𝐸cell = 𝐸𝑂𝑃 /FeO + 𝐸𝑅𝑃 ° Ni− + 2𝑒 − → Ni (at cathode)
Fe Ni2 O3 /NiO
mol.wt.
186 (2) Equivalent weight of Ni = gain electron
Eq. of Cu = Eq. of Ag 58.7
𝑊 1.08 =
∴ 63.5/2
= 108
2
= 29.35
∴ 𝑊Cu = 0.3175 g
𝑖 = 12A, 𝑡 = 1h = 60 × 60𝑠.,
187 (2) eq. wt.
∆𝐺° = −𝑛𝐹𝐸° 𝑍=
96500
P a g e | 62
𝑍𝑖𝑡 × efficiency = 11.5 kJ
Weight of deposit Ni =
100 204 (1)
29.35 × 12 × 60 × 60 × 60 Fe → Fe2+ + 2e− (anode reaction)
=
96500 × 100 O2 + 2H2 O + 4𝑒 ⟶ 4OH − (cathode reaction)

= 7.883 g The overall reaction is
197 (1) 2Fe + O2 + 2H2 O → 2Fe(OH)2
Λ∞ ∞ ∞
𝑒𝑞 . = 𝜆𝑎 + 𝜆𝑐 = 315 + 35 = 350. Fe(OH)2 may be dehydrated to iron oxide FeO, or
198 (1) further oxidized to Fe(OH)3 and then dehydrated
Cell I : Fe ⟶ Fe2+ + 2e to iron rust, Fe2 O3 .
2Fe3+ + 2𝑒 ⟶ 2Fe2+ 205 (2)
∆𝐺° = −𝑛𝐸°𝐹
Fe + 2Fe3+ ⟶ n = 2 ; 1
Cell II : Fe ⟶ Fe3+ + 3e = 𝑘 × 𝑅 = 0.002765 × 400
𝑎
3Fe3+ + 3e ⟶ 3Fe2+ = 1.106 cm−1.
∆𝐺° = −𝑛𝐸°𝐹
𝐹𝑒 + 2𝐹𝑒 3+ ⟶ 𝐹𝑒 2+ ; 𝑛 = 3; 206 (1)
Cell III : 2Fe3+ ⟶ 6e + 2Fe Net redox change is zero.
3Fe ⟶ 3Fe2+ + 6𝑒 207 (3)
∆𝐺° = −𝑛𝐸°𝐹
Fe + 2Fe3+ ⟶ 3Fe2+ ; 𝑛 = 3; Given fact is :𝑋 is above 𝑍 and 𝑌 in
𝑛 and𝐸° are different for each cell. electrochemical series and thus, 𝑍 cannot displace
199 (1) 𝑋 from its salt.
The element having more 𝐸𝑂𝑃 °
is oxidised and 208 (3)
other is to be reduced. NaCl gives Na+ and Cl− ions;
200 (1) At anode : Cl− ⟶ (1/2)Cl2 + 𝑒
∆𝐺 ° = − 𝑛𝐹𝐸 ° At cathode : H + ⟶ (1/2)H2 + 𝑒
Fe2+ + 2𝑒 − ⟶ Fe 209 (4)
∆𝐺 ° = − 2 × F × (− 0.440 V) Λ 𝑣𝑠 √𝑐curves are hyperbolic for weak
= 0.880 F …(i) electrolytes because on dilution their no. of ions
3+ − as well as ionic mobility both increase.
Fe + 3𝑒 ⟶ Fe
∆𝐺 ° = − 3 × F × (− 0.036 V) 210 (4)
= 0.108 F …(ii) 𝐸cell = 𝐸OPanode + 𝐸RPcathode
On subtracting Eqs. (i) from (ii) = 𝐸OPanode − 𝐸OPcathode
Fe3+ + 𝑒 − → Fe2+ 211 (3)
∆𝐺 ° = 0.108 F – 0.880 F = − 0.772 F Cu2+
aq + 𝑒 ⟶ Cuaq ;
+
∆𝐺1° = ⎯[1 × 0.15 × 𝐹]
∆𝐺 ° − 0.772 𝐹 Cu2+
aq + 𝑒 ⟶ Cu ;
𝐸° = − = − = + 0.772 V ∆𝐺2° = ⎯[1 × 0.50 × 𝐹]
𝑛𝐹 1 ×𝐹 On adding
201 (1)
∴ Cu2+
aq + 2𝑒 ⟶ Cu; ∆𝐺3° = ⎯[2 × 𝐸3° × 𝐹]
The metals having higher negative value of
∴ 2𝐸3° = 0.65 V or 𝐸3° = 0.325 V
standard reduction potential are placed above
212 (3)
hydrogen in electrochemical series. The metals
According to Faraday’s second law of electrolysis,
places above hydrogen has a great tendency to
we have
donate electrons or oxidising power. The metals
𝑤1 𝐸1
having great oxidizing power are strongest =
𝑤2 𝐸2
reducing agent. Zn has higher negative value of 1.08 108
standard reduction potential. Therefore, it is the ∴ 𝑥
= 1
strongest reducing agent. ∴ Weight of hydrogen (𝑥) = 0.01 g
202 (2) Hence, the volume of hydrogen at
Chromium is more electropositive metal than 22400 × 0.01
STP = = 112 cm3
iron. In stainless steel, chromium forms an oxide 2
layer and thus it protects steel from corrosion. 213 (2)
203 (2) For strong electrolytes the plot of molar
Energy = charge × potential conductance (Λ 𝑚 ) 𝑣𝑠. √𝐶 is linear.
= 1 × 100 × 115 J
P a g e | 63
electrolytes. A salt used for this purpose should
have almost same speeds of its cation and anion.
223 (3)
Λ̊ CH3 COOH = Λ̊ CH3 COONa + Λ̊ HCl − Λ̊ NaCl
= 91.0 + 426.2 – 126.5
= 390.7 𝑆 cm2 mol−1
224 (2)
(Λ m )with√𝐶 for strong electrolyte. 𝐸𝑂𝑃°
of Li is high enough.
214 (2) 226 (1)
In Galvanic cell (Daniel cell) the electrical energy As ammonia is added the concentration of H +
is produced from chemical reactions. decreases, thus oxidation potential increases by
At anodeZn → Zn2+ + 2e− (oxidation) 0.65
At cathodeCu2+ + 2e− ⟶ Cu(reduction) 227 (4)
Cell reactionZn + Cu2+ ⟶ Zn2+ + Cu If 𝐸° = 0, then ∆𝐺° = −𝑛𝐸° 𝐹 = 0.
Or Zn(s) + CuSO4 (aq) → Cu(s) + ZnSO4 (aq) 228 (3)
215 (3) Molten NaCl possesses Na+ and Cl− ions.
In Galvanic cell the electrical energy is produced 229 (1)
from chemical reaction. 𝑖. 𝑒., chemical energy is Charge = 10 × 1 = 10 C
transformed into electrical energy. Also, 96500 C = 6 × 1023 electrons.
216 (1) 230 (2)
sp. conductance
Cell constant = For, Sn(𝑠) + 2Fe3+ (𝑎𝑞) → 2Fe2+ (𝑎𝑞) + Sn2+ (𝑎𝑞)
conductance
217 (4) °
𝐸cell °
= 𝐸Sn/Sn °
2+ + 𝐸Fe3+ /Fe2+
NaCl, KNO3 , HClare strong electrolytes but the size
of H + is smallest. Smaller the size of the ions, = (0.14) + (0.77) = 0.91 V
greater is the conductance and hence greater is
the conductivity 232 (4)
(κ = 𝐶 × cell constant). As AgnO3 is added to solution , KCl will be
218 (1) displaced according to following reaction
°
𝐸𝑅𝑃 for H is maximum in these. AgNO3 (𝑎𝑞) + KCl (𝑎𝑞) ⟶ AgCl(𝑠) + KNO3 (𝑎𝑞)
219 (1) For every mole of KCl displaced from solution,
2Br − (𝑎𝑞) + Cl2 ⟶ 2Cl− (𝑎𝑞) + Br2 one mole of KNO3 comes in solution resulting in
Bromine is relased by chlorine as more reactive almost constant conductivity. As the end point is
halogen displaces less reactive halogen from their reached, added AgNO3 remain in solution
salt solutions. increasing ionic concentration, hence conductivity
220 (4) increases.
Ag + + 𝑒 − ⟶ Ag 233 (4)
°
∵ 96500 C are required to deposite Ag = 108 g 𝐸𝑅𝑃 Cu2+ /Cu is more;
∴ 965 C are required to deposite Ag Thus, Cu2+ gets reduced easily to oxidise others.
108 234 (4)
= × 965 = 1.08 g 1 1 1
96500
∵ 𝑅 ∝ 𝑙 and, 𝑅 ∝ ∴ 𝑅 ∝ 𝑜𝑟 𝑅 = ρ.
221 (2) 𝑎 𝑎 𝑎
Zn acts as anode and gets oxidised. At graphite Where 𝑅 is resistance, ρ is specific resistance
cathode the reaction is , and𝑙, 𝑎 are length and area of cross-section of
MnO2 + NH4+ + 𝑒 ⟶ Mn(OH)O + NH3 wire.
1
222 (3) ∵ 𝜅 = 𝝆 where 𝜅 is conductivity
Salt bridge is used to remove or eliminate liquid 1 1 1
Then 𝜅 = 𝑅 × 𝑎 = 𝐶 × 𝑎 ;
junction potential arised due to different relative
1
speed of ions of electrolytes at the junction of two Where 𝐶 is conductance = 𝑅
electrolytes in an electrochemical cell. Thus, a salt Also, Equivalent conductivity = Conductivity
bridge such as KCl is placed in between two ×𝑉in mL where 𝑉 is solution containing 1
P a g e | 64
equivalent in it or Ag + + 𝑒 − → Ag
1000
Eq. conductivity = 𝜅 × Cu2+ + 2𝑒 − → Cu
𝑁
235 (3) Au3+ + 3𝑒 − → Au
HCl is an electrolyte. 3 Faradays liberate 1 mole of Au, 3 moles of Ag
236 (2) and 3/2 moles of Cu. Thus, molar ratio of Ag:Cu:
𝒬 = 𝑖𝑡 Au is 3:3/ 2:1 or 6:3:2.
= 1 × 60 247 (3)
° °
= 60 𝐶 𝐸𝐶𝑢 2+ /𝐶𝑢 = +0.34 V = 𝐸𝑅𝑃

°
237 (4) (Above H if arranged in decreasing 𝐸𝑅𝑃 )
HClis strong electrolyte and H has highest 248 (2)
+

conducting power due to Grothus conductance. ° 0.059


𝐸cell = 𝐸cell + log[𝑀𝑛+ ]
238 (4) 𝑛
∆ ∆ 0.059
AgNO3 → Ag 2 O → Ag + O2 𝐸cell = 0.34 + log 10−2
2
239 (3) 0.059
Cl in OCl−has oxidation number as +1. = 0.34 + × −2
2
Thus, Cl+ + 2𝑒 ⟶ Cl− (𝑖. 𝑒. , reduction of OCl−) = + 0.281 V
240 (1) 249 (1)
° ° °
𝐸cell = 𝐸red (cathode) − 𝐸oxi (anode) The required reaction (Cu2+ + Cu → 2Cu+ ) can be
= 𝐸° + − 𝐸° 2+
obtained by using the following reactions
Ag /Ag Cu+ /Cu
= 0.80 – (+ 0.34) = + 0.46 V Cu2+ + 𝑒 − → Cu+ , 𝐸°Cu2+ / Cu+ = 0.15 V …(i)
241 (1)
At anode Cu2+ + 2𝑒 − → Cu, 𝐸°Cu2+ / Cu = 0.34 V …(ii)
2H2 O(𝑙) → O2 (g) + 4 H + (𝑎𝑞) + 4𝑒 −
At cathode Multiplying Eq. (i) by 2, we get
2Cu2+ (𝑎𝑞) + 4𝑒 − → 2Cu(𝑠)
2Cu2+ + 2𝑒 − → 2Cu+ ,
Net electrolysis reaction is
2Cu2+ (𝑎𝑞) + 2H2 O(𝑙) ∆𝐺1 = −𝑛𝐹𝐸 = −2 × 𝐹 × 0.15 …(iii)
→ 2Cu(𝑠) + 4H + (𝑎𝑞) + O2 (g)
So, H2 O is obtained. Cu2+ + 2𝑒 − → Cu
243 (4)
∆𝐺2 = −𝑛𝐹𝐸 = −2 × 𝐹 × 0.34 …(iv)
Al3+ + 3𝑒 − → Al
𝑤 = 𝑍𝑄 Subtract the Eq. (iv) from Eq. (iii)
Where, 𝑤 = amount of metal
𝑤 = 5.12 kg Cu2+ + Cu → 2Cu+
= 5.12 × 103 g
Z = electrochemical equivalent ∆𝐺3 = −𝑛𝐹𝐸 = −1 × 𝐹 × 𝐸°
equivalent weight atomic mass
Z = = Also ∆𝐺3 = ∆𝐺1 − ∆𝐺2
96500 electrons × 96500
27 ∴ −1 𝐹𝐸° = (−2𝐹 × 0.15) − (−2𝐹 × 0.34)
𝑍 =
3 × 96500
27 𝐸° = −0.38
5.12 × 103 = × 𝑄
3 × 96500
5.12 × 103 × 3 × 96500 This is the value for the reaction
𝑄 = 𝐶
27
= 5.49 × 107 𝐶 Cu2+ + Cu → 2Cu+
245 (4) But the given reaction is just reverse of it
The increasing order of deposition of cations at
the cathode is ∴ 𝐸cell for given reaction = + 0.38 V
Cu2+ < Ag + < Au3+
𝐸 ∝ 𝑍 250 (1)

P a g e | 65
Cell representation is done as follows 260 (1)
Anode | Anodic electrolyte || cathodic electrolyte | ° 0.059 [Zn2+ ]
𝐸cell = 𝐸cell − log
cathode 𝑛 [Cu2+ ]
(i) Oxidation is loss of electron and it takes place
at anode. Reduction is gain of electron and it takes 0.059 0.1
= 1.10 − log
place at cathode. 2 0.1

= 1.10 V
∴ For cell reaction,
Zn + Cu2+ ⟶ Zn2+ + Cu 262 (4)
Zn is anode and Cu is cathode. °
𝐸cell °
= 𝐸cathode °
− 𝐸anode
°
∴ Cell representation is ∴ 2.46 = (+0.80) − 𝐸Al3+ /Al

°
Or 𝐸Al3+ /Al = 0.80 – 2.46 = − 1.66 V
2+ 2+
Zn | Zn || Cu | Cu
263 (3)
251 (2) During electrolysis, volumes of O2 and H2
In CuSO4 ,change is Cu2+ + 2𝑒 ⟶ Cu; liberated are in the ratio of 1 : 2 Hence, volume of
In CuCN, change is Cu+ + 𝑒 ⟶ Cu; H2 liberated will be 4.48dm3 .
Thus, 𝑊 ∝ 𝐸Cu , which is more in CuCN. 264 (1)
252 (1) 2Al + dil . H2 SO4 → Al2 SO4 + H2 ↑
°
More is 𝐸𝑂𝑃 , more is the tendency to get itself
265 (1)
oxidised and more is reducing power. Given °
°
High value for 𝐸𝑟𝑒𝑑 . Shows more electronegativity
values are of𝐸𝑂𝑃 .
𝑖. 𝑒., Zn is more electropositive than Fe.
254 (1) ° °
(𝐸Zn 2+ /Zn < 𝐸Fe2+ /Fe )
Higher the negative value of𝐸 ° , more is the
reducing power. 266 (1)
The order of 𝐸 ° values (negative value) is During rusting, oxidation of iron takes place, 𝑖. 𝑒. ,
−2.37 > −0.76 > −0.44 it acts as anode. Hence, coating/connecting iron
(Mg) (Zn) (Fe) with metal of lower reduction potential (in
∴ Mg can reduce both Zn2+ andFe2+ . Zn can comparison to iron) is the best way to prevent its
reduce Fe2+ , but not Mg 2+ . Fe cannot reduce Mg rusting. In this process, the metal with low
and Zn but can oxidize them. reduction potential undergoes oxidation (𝑖. 𝑒. acts
as anode) while iron acts as cathode. (Cathodic
255 (1)
This, is Hückel-Onsager equation. protection)
Use of saline water accelerates the process of
256 (1)
Kohlraush gave the standard value of conductivity rusting thereby increasing the electric conduction
for different concentration of KCl solution. Thus, of electrolyte solution formed on the metal
since, surface.
𝑙 267 (4)
𝜅=𝐶 × ; The cell reaction is
𝑎
By finding conductance of same concentration KCl Cd (s) + 2AgCl (s) → 2 Ag (s) + Cd2+ (aq) + 2Cl−
solution, one finds 𝑙/𝑎. (aq)
257 (2) 𝐸1 = 0.6915 V at 0℃
MnO2 in Lechlanche cell. 𝐸2 = 0.6753 V at 25℃
𝜕 𝐸𝑐𝑒𝑙𝑙 𝐸2 − 𝐸1
258 (3) Now, = 𝑇 −𝑇
𝜕𝑇 2 1
Reduction is always carried out at cathode. 0.6753−0.6915
= 298−273
259 (4)
°
Given, 𝐸cell = 0.34 𝑉 = - 6.48 × 10−4
𝜕𝐸
°
𝐸cell = 𝐸 ° 2+ − 𝐸° +
△S =nF [ 𝜕 𝑐𝑒𝑙𝑙𝑇
]
(Cu /Cu) (H2 /H )
0.34 = °
𝐸(Cu 2+ /Cu) − 0.00 Now, we put the value
° △S = 2 × 96500 (- 6.48 × 10−4 ) = - 125.064
∴ 𝐸(Cu 2+ /Cu) = + 0.34

P a g e | 66
We know that, weight of silver deposited
△G = −𝑛𝐹𝐸𝑐𝑒𝑙𝑙 weight of hydrogen liberated
= - 2 × 96500 × 0.6753 eq. wt. of silver
=
= - 1.303 × 105 eq. wt. of hydrogen
As, △G = △H - T△S 𝑤 108
−2
=
For calculating △H = △G + T△S 5.04 × 10 1.008
= - 1.303 × 105 + 298 (- 125.064 kJ) 108 × 5.04 × 10−2
𝑤 = = 5.4 g
△H = - 1.6726 × 105 J 1.008
276 (4)
= - 167.26 kJ
Large negative RP or more positive oxidation
268 (2)
potential and thus, more is the tendency to get
Rusting of iron is catalyzed by moist air.
oxidized.
269 (4)
0.059 277 (2)
°
𝐸cell = log𝐾𝑐 In presence of Hg electrode preferential discharge
𝑛
0.059 of Na+ (in comparison to H + ) occurs.
0.295 = 2 log 𝐾𝑐 ; ∵ 𝐾𝑐 = 1010
278 (4)
270 (2) 𝐸 = 𝑄. 𝑉 = 1C × 1V = 1J. It is unit of energy.
It is either ohm−1 m−1 or Siemens Also 1J = 107erg.
−1 −1
m , 𝑖. 𝑒. , 𝑆 m . 279 (1)
271 (1) From Asolution of CuSO4 , Cu can be recovered by
Wt. of Ag deposited = eq. wt. of Ag = 108 g Fe metal. Because Fe is more reactive than Cu, it
Wt. of Ni deposited = eQ. wt. of Ni = 29.5 g replace Cu easily.
Wt. of Cr deposited = eq. wt. of Cr = 17.3 g 280 (4)
272 (3) 0.059 [Ni2+ ]
°
When the solution of a weak electrolyte is diluted, 𝐸cell = 𝐸cell + 2
log [Zn2+ ]
the volume of the solution increases, hence ° 0.059 1 °
∴ 0.5105 = 𝐸cell + 2 log 1 or𝐸cell = 0.5105 V
equivalent conductivity (𝜆𝑐 ) increases. However,
281 (2)
during this process, the number of current
The charging of lead storage battery involves the
carrying particles per cm2 decreases, hence
reverse reactions shown in answer 5.
specific conductivity (𝐾𝑐 ) decreases.
282 (1)
273 (1)
𝐸°does not depend on stoichiometry of change.
𝑊 𝑖×𝑡
= 283 (1)
𝐸 96500
𝑊 More negative is the standard reduction potential,
∴ 𝐸 = 10−2 (Agis monovalent)
greater is the tendency to lose electrons and
∴ 𝑄 = 𝑖 × 𝑡 = 96500 × 10−2 = 965 C hence, greater is the reactivity
274 (3)
Sn (𝑎𝑞) + 2Fe3+ (𝑎𝑞) → 2Fe2+ (𝑎𝑞) + Sn2+ (𝑎𝑞) 284 (1)
°
𝐸cell °
= 𝐸oxi °
+ 𝐸red Eq. of Ag = Eq. of H2 ;
° ° 𝑊 5600 × 2
= 𝐸Sn /Sn2+ + 𝐸Fe3 / Fe2+ =
108 22400 × 1
Given , 𝐸°Sn2+/ Sn = − 0.14 𝑉 ∴ 𝑊Ag = 54 g
°
∴ 𝐸Sn /Sn2+ = + 0.14 V 285 (1)
∞ ∞
°
𝐸 3+ 2+ = 0.77 V Λ∞ ∞
NH4 OH = Λ NH4 Cl + Λ NaOH − 𝜆NaCl
Fe / Fe
°
𝐸𝑐𝑒𝑙𝑙 = 0.14 + 0.77 = 0.91 V 286 (1)
1
275 (3) 𝜅 = 𝑅 ×cell constant
Given, weight of hydrogen liberated
= 5.04 × 10−2 g ∴ Cell constant= κ × 𝑅 = 0.012 × 55 = 0.66 cm−1
Eq. wt. of hydrogen = 1.008
288 (2)
Eq. wt. of silver = 108 Weight of Cu Eq. wt. of Cu
Weight of silver deposited, 𝑤 =? =
Weight of H2 Eq. wt. of H2
According to Faraday’s second law of electrolysis,
P a g e | 67
Weight of Cu 63.6/2 299 (4)
=
0.504 1 At infinite dilution an electrolyte is 100% ionised
∴ Weight of Cu= 15.9 g or 𝛼 = 1 for weak electrolytes. At this point all
289 (3) interionic effect disappears.
Number of g-equivalent = number of faraday pass 300 (3)
Cuvoltameter or Cu or Ag coulometer are used to
4g=4F
detect the amount deposited on an electrode
290 (3) during passage of known charge through solution.
Electrolytes on dissolution in water furnish ions 301 (2)
and these ions carry charge towards opposite Specific conductance = conductance × cell
electrodes. constant
291 (4) 1
1.3 Sm−1 = S × cell constant
During electrolysis of CuSO4 , Cu2+ gets discharged 50
∴ Cell constant
at cathode and OH − at anode. Thus, solution
becomes acidic due to excess of H + and = 1.3 × 50 m−1 = 65 m−1 = (65/100)cm−1
SO2−
4 orH2 SO4 1000 ×conductance ×cell constant
292 (1) Molar conductivity =
molarity
Fluoro group causes negative inductive effect 1000 1 65
increasing ionization, thus 0.1 M difluoroacetic = × × = 6.25 Scm2 mol−1
0.4 260 100
acid has highest electrical conductivity. = 6.25 × 10−4 Sm2 mol−1

302 (4)
pH = 3, [H + ] = 10−3
°
𝐸 = 𝐸red + 0.059 log (ion)
𝐸 = 0 + 0.059 log (10−3 )
𝐸 = + 0.059 (−3) = − 0.177 𝑉
293 (3) 303 (2)
Hydrolysis of water can be represented by the Ag + + 𝑒 − → Ag
following equations 9650 C = 0.1 F = 0.1 equivalent Ag
2H2 O ⇌ O2 + 4H + + 4𝑒 − = 0.1 mol Ag
∵ 4 Faraday of charge liberate O2 = 32 g = 10.8 g Ag
32
∴ 1 Faraday of charge liberate O2 = 4 g 304 (4)
=8 g Electrochemical cell are based upon the reaction
294 (3) between various electrolytes. The reaction given
AgNO3 + KCl ⟶ AgCl ↓ + KNO3 . in option (d) does not involve electrolytes, so it
295 (4) cannot be Abase for electrochemical cell.
Zn + MgCl2 →no reaction 305 (2)
96500 C = 6 × 1023 electrons.
This type of reaction does not occur because 306 (1)
In the internal circuit of Agalvanic cell ions flow
Mg 2+ 𝐸° = −2.37 V, whileZn2+ 𝐸 ° = −0.76 V while in the external circuit, there is Aflow of
296 (1) electrons from zinc rod to copper rod.
The process is called cathode protection where 307 (2)
iron acts as cathode and thus, not oxidised. On dilution, ionic mobility increases but number
297 (4) of ions present in 1 mL decreases; Thus, only
These are characteristic of conductivity water. conductivity decreases and rest all increases.
308 (2)
298 (1) C5 H12 + 8O2 ⟶ 5CO2 + 6H2 O
2+
𝐸° = 𝐸, when [Zn ] = 1𝑀; ∆𝐺° = 5 × 𝐺°CO2 + 6 × 𝐺°H2 O ⎯ 𝐺C° 5 H12 − 8 × 𝐺𝑂° 2
Also process is Zn2+ (𝑎𝑞) + 2𝑒 ⟶ Zn(𝑠). = 5 × (−394.4) + 6 × (−237.2) + 8.2

P a g e | 68
= ⎯3387 kJ °
𝐸cell = 0.00𝑉
∆𝐺° = 𝑛𝐸°𝐹 ∴ 𝐸cell = −
0.05915
log 𝒬
𝑛
3387 × 103 = 32 × 𝐸° × 96500 [(C −2.4 )5 ⟶
Given, pH = 1.0
5(C 4+ ) + 32𝑒]
∴ [H + ] = 1 × 10−1
∴ 𝐸° = 1.0968 V𝑛 = 32
0.05915 1
309 (2) 𝐸cell = − log +
1 length
𝑛 [H ]
Cell constant = = [∵ The reaction occurring is 2H + + 2𝑒 − → H2 ]
𝑎 area
∴ unit iscm−1 . 0.05915
= + log(H + )
310 (3) 1
From given data (from ∆𝐺 ° = − 𝑛𝐸 ° 𝐹 ) = 0.05915 log(10−1 )
4. Cu (𝑠) → Cu2+ (𝑎𝑞) + 2𝑒 − = - 0.05915 V
= - 59.15 mV
∆𝐺1° = − 2 × (− 0.34) × 𝐹 313 (2)
Laws of electrolysis were proposed by Michael
5. Cu2+ (𝑎𝑞) + 𝑒 − → Cu2+ (𝑎𝑞)
Faraday in 1833.
∆𝐺2° = − 1 × ( 0.15) 𝐹 6. Faraday’s first law “The mass of any
substance deposited or liberated at any
On addition, electrode is directly proportional to the
quantity of electricity passed”.
Cu (𝑠) → Cu2+ (𝑎𝑞) + 𝑒 − . ∆𝐺1° = − 1 × 𝐸 ° × 𝐹
𝑖. 𝑒. , 𝑤 ∝ 𝑄
∆𝐺3° = ∆𝐺1° + ∆𝐺2°
where, 𝑤 = mass of ions liberated in gram
= (− 2 × − 0.34 × 𝐹) + ( − 1 × 0.15 × 𝐹)
𝑄= quantity of electricity passed in coulombs.
=+ 0.68 F – 0.15 F = 0.53 F
7. Faraday’s second law “When the same
0r 𝐸 ° = − 0.53 𝑉 quantity of electricity is passed through
different electrolytes, the masses of
Reaction,
different ions liberated at the electrodes
2Cu+ (𝑎𝑞) ⇌ Cu2+ (𝑎𝑞) + Cu (𝑠), 𝐸 ° = ? are directly proportional to their chemical
equivalents.”
So, Cu+ (𝑎𝑞) + 𝑒 − ⇌ Cu (𝑠) , 𝐸 ° = + 0.53 𝑉
𝑤1 𝐸1 𝑍1 𝑖𝑡 𝐸1
+ (𝑎𝑞) 2+ (𝑎𝑞) − ° 𝑖. 𝑒., = or = .
𝐶𝑢 ⇌ 𝐶𝑢 + 𝑒 ; 𝐸 = − 0.15 V 𝑤2 𝐸2 𝑍2 𝑖𝑡 𝐸2

2Cu+ (aq) ⇌ Cu2+ (aq) + Cu (s); E 0 = +0.38V 314 (3)


311 (2) AgorCu voltameters or coulometers are used to
250 measure the total charge passed through solution
250mL of 1 M AgNO3 contain = 1000
in terms of weight of Ag or Cu deposited there on
= 0.25 mole AgNO3
voltameter or coulometer cathode.
∵ Electricity required to liberate 1 g equivalent of
315 (4)
metal
Specific conductance is defined as the
= 96500 C
conductance of one centimeter cube (cc) of the
∴ Electricity required to liberate 0.25 g equivalent
solution of an electrolyte. Upon dilution, the
of metal
96500 × 0.25 concentration of ions per cc decreases, so the
= specific conductance falls. Thus, specific
1
= 24125 C conductance ∝ concentration of electrolytes,
312 (4) hence specific conductance is least for the
0.05915 solution for 0.002 N concentration.
°
𝐸cell = 𝐸cell − log 𝒬 316 (1)
𝑛

For standard hydrogen electrode, Λ∞ ∞
AgCl = 𝜆𝐴𝑔+ + Λ Cl−

P a g e | 69

= 𝜆∞ ∞
AgNO3 + 𝜆NaCl − 𝜆NaNO3 All are same.
317 (2) 328 (2)
H2 SO4is strong electrolyte. Eq. of 𝐴 = Eq. of 𝐵 = Eq. of 𝐶
2.1 2.7 7.2
318 (2) or = =
7/𝑛1 27/𝑛2 48/𝑛3
The electrochemical cell stops working after 0.3 𝑛1 = 0.1𝑛2 = 0.15 𝑛3
sometime because electrode potential of both the 𝑛2 𝑛3
∴ 𝑛1 = =
electrodes become equal. 3 2

320 (1) If 𝑛1 = 1 then𝑛2 = 3, 𝑛3 = 2


Eq.of Mg = Eq. of Al 329 (2)
𝑊Mg 𝑊Al Cu2+ + 2KI ⟶ CuI2 + 2K + ;
or =
12 9 2CuI2 ⟶ Cu2 I2 + I2
321 (3) 330 (1)
° ° °
𝐸cell = 𝐸𝑂𝑃ZN
+ 𝐸𝑅𝑃Cu
𝐸°RPCu Equal equivalent of each are liberated.
= 0.76 + 0.34 = +1.10 V. Eq. of 𝑋 = Eq. of 𝑌
322 (3) 𝑊1
= 2
𝑊
∴ 𝑊1 = 𝑊2
2𝑀/2 𝑀/1
1 mole of monovalent metal ion means charge of
332 (3)
𝑁 electrons
In this reaction,
𝑖. 𝑒., 96500 C or 1 faraday.
323 (1)
1
Cell reaction : 2 H2 (g) + Ag + (𝑥) ⇌ Ag(𝑥) +
H + (1 M)

0.0591 [H + ]
𝐸 = 𝐸° = − log EMF = 𝐸cathod − 𝐸anode
𝑛 [Ag + ]
= −0.41 − (−0.76)
0.0591 1
0.503 = 0.80 − log
1 𝑥 = + 0.35 V
𝑥 = 9.43 × 10−6 M 333 (3)
Number of moles of Ag + in 350 mL 𝑄 = 𝑖 × 𝑡 = 100 × 10−3 × 30 × 60 = 180 C
334 (2)
−6 50 % H2 SO4aqueous solution can be electrolysed
9.43 × 10 × 350
= = 3.3 × 10−6 by using Pt electrodes as
1000
2H2 SO4 → 2HSO− 4 + 2H
+
−6 −4
Mass of Ag = 3.3 × 10 × 108 = 3.56 × 10 g 2HSO− −
4 → H2 S2 O8 + 2e (at anode)
3.56 ×10−4 335 (3)
% of Ag in the ore = 1.05 × 100
Λ̊ CH3 COOH = Λ̊ HCl + Λ̊ CH3 COONa − Λ̊ NaCl
= 0.0339% 336 (4)
The metal should be capable of adsorbing H (𝑒.g.,
324 (1) Pt).
Current 𝑖 =? 337 (2)
27 We know that 1 Faraday charge liberates one
Equivalent weight of Al = 3 = 9
𝑤 0.09 × 96500 gram- equivalent of a metal, hence 0.5 F charge
𝑤 = 𝑍. 𝑖. 𝑡or𝑖 = = will liberate
𝑍. 𝑡 9 × 96.5
= 10 A = 0.5 × 23
325 (2) = 11.50 g of sodium (E = 23)
Fe being above Cu in electrochemical series and 338 (2)
thus, liberates Cu from CuSO4. Ionic mobility depends upon the charge to size
326 (3) ratio of ion. The ionic size in case of hydrated
1 faraday deposits 𝐸 g of species. cation is
327 (4)
P a g e | 70
K + (𝑎𝑞) < Na+ (𝑎𝑞) < Li+ (𝑎𝑞) °
𝐸cell = 𝐸cathode − 𝐸anode
= - 0.44 – (0.76)
339 (2)
= - 0.44 + 0.76
Reduction always occurs at cathode during
=0.32 V
electrolysis.
342 (1)
340 (1) 𝑄 faraday
2H − ⟶ H2 + 2𝑒; Hydrogen in CaH2 is ⎯ ve. 𝑄 = 𝑁. 𝑒 𝑜𝑟 𝑒 = =
𝑁 Av. no.
341 (4)
Zn2+ + 2e− ⟶ Zn; 𝐸 ° = −0.76 𝑉
Fe2+ + 2e− ⟶ Fe; 𝐸 ° = −0.44 𝑉
Cell reaction is
Fe2+ + Zn ⟶ Zn2+ + Fe
343 (4)
Fe2+ + 2𝑒 ⟶ Fe; ⎯ ∆G1 = 2 × (−0.44) × 𝐹
3+
Fe + 3𝑒 ⟶ Fe ; − ∆G2 = 3 × (−0.036) × 𝐹
− − − + −
Fe2+ ⟶ Fe3+ + 𝑒; ∆G3 = −0.88 + 0.108 = 0.772 or 0.772 = 1 × 𝐸° × 𝐹
∴ Fe3+ + 𝑒 ⟶ Fe2+ ; 𝐸° = +0.772 V
Above procedure should be used only when two half reactions on algebraic sum give a third half reaction.
344 (4) 𝑤 = 𝑍𝑖𝑡
Na2 S4 O6 is Where, 𝑤 = amount of substance
𝑍 = Faraday constant or electrochemical
equivalent
𝑖 = current in ampere
𝑡 = time in second
The two S atoms which are linked to each other
If 𝑖 = 1, 𝑡 = 1, 𝑤 = 𝑍
have 0 oxidation number. The oxidation number
350 (4)
of other two S-atoms can be calculated as
Mercury cannot displace hydrogen from acid. The
2𝑥 + 2 × 0 + 6 × −2 = −2
reason can be explained on the basis of
2𝑥 = 12 − 2 = 10
electrochemical series. The metal, which is placed
𝑥 = +5
above hydrogen in this series, can displace
345 (4)
0.059 𝑀
hydrogen from an acid. Mercury is placed below
E = 𝐸 ° - 𝑛 log [𝑀𝑛+] hydrogen in ECS, hence, it cannot displace
° 0.059 products hydrogen from an acid.
= 𝐸 - 𝑛 log [reactants]
351 (4)
E = 0 at equilibrium
0.059 Eq. of Fe2+ = Eq. of Fe3+
0 = 𝐸 ° - 𝑛 log 𝐾𝑒𝑞 𝑊1 𝑊2
or 𝐴/2
= 𝐴/3
𝑛𝐸 °
Or log 𝐾𝑒𝑞 = 0.059 𝑊 2+ 3
1 ×0.36
or 𝑊Fe = 2
Or log 𝐾𝑒𝑞 = = 6.09 Fe3+
0.059 352 (3)
𝐾𝑒𝑞 = 1.2 × 106
Oxidation of Cl− at anode and reduction of Na+ at
346 (4) cathode.
Aluminium is more electropositive than Zn, hence 353 (3)
Al replace it from its salt solution. In case of attacked electrodes, metal dissolves at
347 (4) anode and deposits at cathode.
Cu is below Fe in electrochemical series. [Ag ⟶ Ag + + 𝑒 (anode); Ag + + 𝑒
348 (1) ⟶ Ag(cathode)]
Because, barring Ag, other come after the Fe in Thus, concentration of salt does not change.
electrochemical series 354 (1)
349 (3) 𝑡K+ + 𝑡Cl− = 1
According to Faraday Ist law ∴ 𝑡K+ = 1 − 0.505 = 0.495

P a g e | 71
355 (3) as well as drinking water to astronauts.
0.0591 365 (2)
𝐸red = 0.78 + log(10−7 )2
1 Λ̊ 𝑚 (NH4 Cl) = Λ̊ 𝑚 (NH4+ ) + Λ̊ 𝑚 (Cl− )
= 0.78 − 0.059 × 7 = 0.367 V …..(i)
356 (3) Λ̊ 𝑚 (NaOH) = Λ̊ 𝑚 (Na+ ) + Λ̊ 𝑚 (OH − ) …..(ii)
This is representation of standard hydrogen
Λ̊ 𝑚 (NaCl) = Λ̊ 𝑚 (NH + ) + Λ̊ 𝑚 (Cl− )
electrode.
…..(iii)
357 (4)
°
By (i) + (ii) + (iii)
𝐸cell = 0.87 + 0.40 = 1.27 V
4Λ̊ 𝑚 (NH4 OH) = Λ̊ 𝑚 (NH4+ ) + Λ̊ 𝑚 (OH − )
Cell reaction ∶ Fe + Ni2 O3 → FeO + 2NiO = Λ̊ 𝑚 (NH4 Cl) + Λ̊ 𝑚 (NaOH)
− Λ̊ 𝑚 (NaCl)
358 (3)
366 (1)
Only Zn and Fe are above H;
Faraday’s laws are independent of external
Also Fe3+ can be reduced to Fe2+ by H.
factors.
359 (2)
367 (3)
BeCl2 is predominantly more covalent among 𝐸×𝑖×𝑡
halides of alkaline earth metals. 𝑊=
96500
360 (3) 36.5 × 𝑖 × 360 × 60
∴ 20 = 96500
△𝐺 =△𝐻 − 𝑇△𝑆
= 2.45 ampere
For a spontaneous cell reaction, △ 𝐻 should be
negative and △ 𝑆 should be positive. Hence, △ 𝐺 368 (1)
∞ ∞
should be negative. Λ∞ ∞
NH4 OH = Λ NH4 Cl + 𝜆OH− − 𝜆Cl−

361 (1) 369 (2)


°
Charge = 2 × charge of electron = 2 × 1.602 × The metal with more 𝐸𝑂𝑃 is oxidised.
10−19 C. 370 (2)
362 (4) It is secondary reference electrode.
° 0.059 [Fe2+ ] 371 (4)
𝐸Cell = 𝐸cell + log [Zn2+ ]
2 1
° 0.059 0.01 2H + + 2𝑒 ⟶ H2 ; 2OH − ⟶ H2 O + O2 + 2𝑒
0.2905 = 𝐸cell + log 2
2 0.10
° 373 (2)
∴ 𝐸cell = 0.32
0.059
The cathode and anode reactions respectively are
°
No, 𝐸cell = 2
log10 𝐾
0.059 Cu2+ + 2𝑒 − → Cu
∴ 0.32 = log10 𝐾
2
2Cl− → Cl2 + 2𝑒 −
𝐾 = 100.32/0.0295
363 (4) The two moles of electrons have been transferred
Equivalent conductance (⋀) = specific from anode to cathode to produce Cu andCl2 in a
conductance (𝜅) × ϕ mole ratio of 1:1. Thus, 2F electricity is required
where, ϕ = volume in cm3 containing 1 g
equivalent of electrolyte 374 (2)
1. g equivalent is dissolve in = Sum of molar conductivity of reactants = sum of
1000 cm3 molar conductivity of products
Therefore, for the reaction
1000
1 g equivalent is dissolve in = 0.1 cm3 CH3 COOH + NaOH ⟶ CH3 COONa + HCl
⋀0𝑚 CH3 COOH = ⋀0𝑚 CH3 COONa + ⋀0𝑚 HCl
3
= 10000cm − ⋀0𝑚 NaCl
= 91 + 425.9 – 126.4
So, ⋀ = 10000 κ
= 390.5Ω−1 cm2 mol−1
364 (1) 376 (4)
An important application of fuel cell used in space The half reactions are
programmes to provide power for heat and light Fe(s) → Fe2+ (𝑎𝑞) + 2e− × 2

P a g e | 72
O2 (g) + 4H + + 4𝑒 − ⟶ 2H2 O value of reduction electrode potential.
+
2Fe(s) + O2 (g) + 4H Thus , order of reducing character is
2+ (𝑎𝑞) 𝐵 > 𝐶 > 𝐴
→ 2Fe + 2H2 O (𝑙)
0.059 −3 2
(10 ) 390 (2)
𝐸 = 𝐸° − log −3 4
= 1.57 V 11.2
4 (10 ) (0.1) No. of moles of H2 = 22400
377 (2) No .of equivalence of hydrogen
Use of electrolysis is not done in the production of 1.12 × 2
water. = = 10−4
22400
379 (4) No. of Faradays required = 10−4
These are characteristics of fuel cells. ∴ Current to be passed in one second
380 (2) = 96500 × 10−4
Cu2+ + 2𝑒 ⟶ Cu; ∆𝐺1° = −2 × 0.337 × 𝐹 = 9.65 A
2+ + °
Cu + 𝑒 ⟶ Cu ; ∆𝐺2 = 1× 0.513 × 𝐹 391 (4)
− − − − +
Mg will not deposit on cathode during
+ ° ° °
Cu + 𝑒 ⟶ Cu; ∆𝐺3 = ∆𝐺1 − ∆𝐺2 = −0.52 𝐹 electrolysis.
∴ −1 × 𝐸° × 𝐹 = 0.52 𝐹 ⟹ 𝐸° = 0.52 V 392 (1)
381 (2) For strong electrolytes Λ 𝑣𝑠 √𝑐 plots are straight
Anode at which oxidation occurs is represented at line.
left hand side. 393 (3)
382 (2) 0.059
𝐸 = 𝐸° − log[H + ] 2
Zn is above iron in electrochemical series. 2
383 (2) 0.059
= 1.30 − log(10−2 )2
° 0.0592 [Cu2+ ] 2
𝐸cell = 𝐸cell + log + 2 0.236
𝑛 [T1 ]
= 1.30 + = 1.418 V
𝑛 = 2 2
∴ According to above equation 𝐸cell can be 394 (3)
increased by increasing [ Cu ]. 2+ Among given elements, 𝐷 has the minimum
384 (1) reduction potential (- 2.37 V) hence, it can
Anode is electrode at which oxidation occurs. displace all other from their salts.
385 (4) 395 (3)
Hg 2 Cl2 (𝑠) + 2𝑒 ⟶ 2Hg(𝑙) + 2Cl 2(𝑎𝑞). − Λ0CH3 COOH = Λ0CH3 COONa + Λ0HCl − Λ0NaCl
386 (1) 396 (2)
Λ m = Λ 𝑒𝑞 .× valency factor; Ag + + 𝑒 − → Ag
For NaCl, valency factor = 1;
Molecular conductivity Λ m is defined as the 96500 C charge liberates silver = 108 g
conductance of all the ions present in a solution ∴ 96500 C will liberate silver = 10.8 g
containing 1g molecule in it; Λ 𝑒𝑞 .is defined as the
conductance of all the ions present in a solution 397 (3)
containing 1g equivalent in it. The oxidizing power of MnO− 4 /Mn
2+
couple
387 (3) decreases by 0.38 V.
Zn is coated to protect iron from rusting. 398 (1)
388 (3) For strong electrolytes, 𝛼 = 1 at normal dilution,
The equivalent conductance of strong electrolyte but Λ 𝑣 ≠ Λ ∞ .
is increased by dilution, because its value is equal 399 (3)
to the multiple of 𝐾𝑣 and the volume of solution. ° °
0.059 [Cu2+ ]
𝐸cell = 𝐸𝑂𝑃Zn + 𝐸𝑅𝑃Cu + log
By making dilution the volume of solution 2 [Zn2+ ]
0.059
increases which also increase the value of ∴ 1.1 = 0.78 + 𝐸𝑅𝑃 °
+ 1
Cu 2
equivalent conductivity. °
∴ 𝐸𝑅𝑃 2+ = 0.32
389 (4) Cu /Cu
°
Reducing character is based upon higher negative ∴ 𝐸 𝑅𝑃Cu2+/Cu = −0.32V

P a g e | 73
400 (3) 9. Oxidizing power of halogen decreases
Cl is placed below Br in electrochemical series; from F2 to I2 because their reduction
the non-metal placed below in series, replaces potentials decreases from fluorine to
other from its solution. iodine.
401 (2)
The laws of electrolysis were given by Faraday. 10. The reducing power of hydrogen halides
402 (1) increases from hydrogen chloride to
In a galvanic cell, the electrons flow from anode to hydrogen iodide since, the stability of the
cathode through the external circuit. At anode 𝐻 – 𝑋 bond decreases in the same order.
(−ve pole) oxidation and at cathode (+ pole) Hence, all statements are correct.
reduction takes place 409 (1)
403 (1) (i) The electrode with higher oxidation potential
In Danial cell, oxidation occurs at anode and acts as anode and electrode with lower oxidation
reduction occurs at cathode. Thus, the element, potential acts as cathode.
which has higher negative value of reduction (ii) Cell reaction is spontaneous when 𝐸cell is
potential, is used as anode and that with lower positive. In case of AandB; oxidation potential of
negative value of reduction potential is used as 𝐴 = −0.03 oxidation potential of 𝐵 = +0.108
cathode.
∵ Reduction potential of Zn is more negative Because oxidation potential of 𝐵 is more than 𝐴 so
than Cu. oxidation of 𝐴 is not possible. The cell having 𝐴 as
∴ Zn | Zn2+ is anode and Cu2+ | Cu is cathode. anode and 𝐵 as cathode is not possible.
404 (3)
The Gibb’s free energy change △ 𝐺 and emf (𝐸 ° ) of Therefore, non-spontaneous cell reaction takes
a reversible electrochemical cell are related by the place between 𝐴and 𝐵.
following expression. 410 (1)
°
△ 𝐺 = − 𝑛𝐹𝐸cell 0.059
°
or = − 𝑛𝐹𝐸 𝐸Cu/Cu2+ = 𝐸Cu/Cu 2+ − log[Cu2+ ]
2
406 (1) if log[Cu2+ ] = 0, 𝑖. 𝑒. , [Cu2+ ] = 1, then𝐸Cu/Cu2+ =
°
For a reaction to be feasible, the value of 𝐸cell °
𝐸Cu/Cu 2+
must be positive. ° °
or 𝑂𝐴 = 𝐸Cu/Cu 2+ = −𝐸Cu/Cu2+ = −0.34
Cu + 2HCl → CuCl2 + H2 (g)
0.059
°
𝐸cell = 𝐸H° +/H2 − 𝐸Cu
°
2+ /Cu
Now, 𝐸Cu/Cu2+ = −0.34 − 2 log 0.1
= 0.00 – (+0.34) = -0.34 V 0.059
= − 0.34 + V
Zn + 2HCl → ZnCl2 + H2 (g) 2
411 (4)
°
𝐸cell = 𝐸H° +/H2 − 𝐸Zn
°
2+ /Zn
Zn is placed above Sn in electrochemical series.
= 0.00 – (-0.76) = +0.76 V 412 (1)
1
Ag + 2HCl → AgCl + H2 (g) 𝐸cell = 𝐸𝐶 − 𝐸𝐴
2 °
° ° ° Given, 𝐸Ag + /Ag = 0.80 V
𝐸cell = 𝐸H+/H2 − 𝐸Ag2+/Ag
+ +
H2 | H || Ag | Ag
= 0.00 – (0.80) = - 0.80 V
∴ Hydrogen is anode and silver is cathode.
Hence, only reaction (ii) is feasible.
𝐸cell = 𝐸𝐶 − 𝐸𝐴
407 (2)
= 0.80 – 0 (∵𝐸H° + /H = 0)
Eq. of Ni = Eq. of Cr
0.3 𝑊 = 0.80 V
∴ 59/2 = 52/3
413 (3)
∴ 𝑊𝐶𝑟 = 0.176 g The number of ions present in solution as well as
408 (1) ionic mobility increase on dilution in case of weak
1
8. Reducing character ∝ reduction potentials electrolytes and thus, equivalent conductivity
increases. Also in case of strong electrolytes only

P a g e | 74
ionic mobility increases with dilution and thus, NH4 OHis weak electrolyte its Λ 𝑣𝑠 √𝑐 curves are
equivalent conductivity increases. For weak shown in fig. (d) of problem 41.
electrolytes, dilution causes increase in number of 422 (1)
ions as well as increase in ionic mobility. Ions move towards opposite electrodes due to
414 (4) coulombic forces of attraction.
1 423 (3)
Resistivity =
𝑘 At LHS (oxidation) 2 × (Ag → Ag + + 𝑒 − ),
1 −1 −1
Or 𝑘 = 15.8 ohm cm °
𝐸oxi = −𝑥
𝑘 × 1000 1 × 49 × 1000 At RHS (reduction)
Also, Λ 𝑣 = =
𝑀 18.5 × 15 °
Λ 1 × 49 × 1000 Cu2+ + 2𝑒 − → Cu, 𝐸red = +𝑦
Now, 𝛼 = Λ 𝑣 = 18.5 × 15 × 348 = 0.507 °
𝑀 2Ag + Cu → Cu + 2Ag + , 𝐸cell
2+
= (𝑦 − 𝑥)
Or = 50.7% °
Note𝐸 values remain constant when half – cell
415 (3) equation is multiplied / divided.
° 2.303𝑅𝑇 424 (3)
𝐸cell = log 𝐾𝑒𝑞
𝑛𝐹 °
More is 𝐸𝑅𝑃 , more is the tendency to get reduced.
° 0.0591
𝐸cell = 𝑛 log 𝐾𝑒𝑞 [At 298 K] °
𝐸𝑅𝑃 for Ag is maximum.
0.0591 425 (2)
0.591 = log 𝐾𝑒𝑞
1 𝑊 =𝑍×𝑖×𝑡
∴ log 𝐾𝑒𝑞 = 10 ∴ 𝑊 = 0.0011180 × 0.5 × 200 = 0.11180 g
∴ 𝐾𝑒𝑞 = 1 × 1010 426 (2)
416 (2) According to Nernst equation.
In other cells, two liquid are not present.
0.0591 [Cu2+ ]
°
417 (1) 𝐸cell = 𝐸Cell + log [Zn2+ ]
2
During the electrolysis of fused NaCl, chloride ions
are oxidized at anode and this process is called 0.0591 [Zn2+ ]
°
oxidation 𝐸cell = 𝐸Cell − log [Cu2+ ]
2
419 (2) Or 𝑦 = 𝑐 + (−𝑚)𝑥
LHS half cell Thus, the slope is negative.
H2 (g) → 2H + (1 M) + 2𝑒 − 427 (3)
𝑃1
From Kohlrausch’s law
RHS half cell
Λ∞ ∞
𝑚 = v+ 𝜆+ + v− 𝜆−

2H + (1 M) + 2𝑒 − → H2 (g)
𝑃2
For CaCl2
∞ ∞
H2 (g) → H2 (g) Λ∞
m (CaCl2 ) = Λ Ca2+ + 2Λ Cl−
P1 p2 = 118.88 × 10−4 + 2 × 77.33 × 10−4
𝑝2
°
𝐸cell = 0.00 V, 𝐾 = ,𝑛 = 2 = 118.88 × 10−4 + 154.66 × 10−4
𝑝1
= 273.54 × 10−4 m2 mho mol−1
𝑅𝑇
°
𝐸cell = 𝐸cell − log e 𝐾 428 (2)
𝑛𝐹
𝑅𝑇 𝑃2 H2 SO4is strong electrolyte and thus, ionises
= 0 − log e completely.
2𝐹 𝑃1
𝑅𝑇 𝑝1 429 (1)
𝐸𝑐ell = log 𝑒 °
𝐸Fe 2+ /Fe = −0.441 V
2𝐹 𝑝2
420 (3) °
𝐸Fe 3+ /Fe = −0.771 V
Eq. of Al = Eq. of Cu = Eq. of Na °
𝐸cell °
= 𝐸𝑂𝑃 °
+ 𝐸𝑅𝑃 (See redox
1 1 Fe/Fe2+ Fe3+ /Fe2+
or mole Al =2 mole Cu = 1 mole Na
3 change)
or 2 : 3 : 6 or 1 : 1.5 : 3 mole ratio. = + 0.441 + 0.771 = 1.212 V
421 (2) 430 (3)
Λ 𝑣𝑠√ccurves on extrapolating to zero Only oxidation occurs at anode. Also discharge
concentration gives Λ∞ for strong electrolytes. potential of H2 O is less than discharge potential of

P a g e | 75
SO2−
4 . = 𝑁/2 atoms of Cu.
431 (2) 439 (3)
° ° 0.059 [Cl− ]2LHE 𝑃2 𝐸cell = 𝐸OP + 𝐸RP
𝐸cell = 𝐸𝑂𝑃Cl
+ 𝐸𝑅𝑃Cl
+ log [Cl− ]2RHE 𝑃1
Ni/Ni2+
2 Au3+ /Au
2
° 0.059 °
0.059 𝑃2 = 𝐸𝑂𝑃 Ni
− log[Ni2+ ] + 𝐸𝑅𝑃Au
+
= log 2
2 𝑃1 0.059
3
log[Au3+ ]
𝐸cell is +ve when 𝑃2 > 𝑃1 . 0.059 0.059
432 (2) = 0.25 ⎯ 2
log(1.0) + 1.50 + 3 log 1.0 =
Given 125mL of 1 M AgNO3 solution. It means that 1.75 V
∵ 1000mL of AgNO3 solution contains 440 (1)
°
= 108 g Ag 𝐸Zn/Zn 2+ > 𝐸°H/H+

∴ 125mL of AgNO3 solution contains 441 (4)


108 × 125 More the reduction potential, more is the power
= gAg
1000 to get itself reduced or lesser is reducing power or
= 13.5 g Ag
greater is oxidizing power
∵ 108 g of Ag is deposited by 96500 C 442 (4)
∴ 13.5 g of Ag is deposited by For the given cell,
96500
= × 13.5 ° 0.0591 [𝑍𝑛2+ ]
108 𝐸cell = 𝐸cell − log
2 [𝐶𝑢2+ ]
= 12062.5 C
° 0.0591 1
𝒬 = 𝑖𝑡 1. 𝐸1 = 𝐸cell − log
2 0.1
𝒬 12062.5
Or 𝑡 = = = 50
𝑖 241.25
° 0.0591
433 (1) = 𝐸cell −
2
By Kohlrausch’s law
° 0.0591 1
∧°NaBr = ∧°NaCl + ∧°KBr − ∧°KCl 2. 𝐸2 = 𝐸cell − 2
log 1
= 126 + 152 – 150
= 128 S cm2 mol−1 ° 0.0591
= 𝐸cell − ×0
434 (1) 2
CaF2 = 𝑘solution − 𝑘water °
= 𝐸cell
435 (1)

(126 Scm2 mol−1 )Λ∞ ∞
NaCl = Λ Na+ + Λ Cl− …(i) 3. °
𝐸2 = 𝐸cell −
0.0591 0.1
log 1
2

(152 Scm2 mol−1 )Λ∞ ∞
KBr = Λ K+ + Λ Br− …(ii) 0.0591
°
= 𝐸cell +
∞ 2
(150 Scm2 mol−1 )Λ∞ ∞
KCl = Λ K+ + Λ Cl− …(iii)
∴ 𝐸3 > 𝐸2 > 𝐸1
By Eqs. (i) + (ii) – (iii) 443 (4)
∞ Follow Kohlrausch’s law.
∵ Λ∞ ∞
NaBr = Λ Na+ + Λ Br−
444 (3)
= 126 + 152 − 150 CuSO4 ⇌ Cu2+ + SO2− 4
At cathode Cu2+ (𝑎𝑞) + 2𝑒 − ⟶ Cu(𝑠)
= 128 Scm2 mol−1 At anode 2H2 O(𝑙) ⟶ O2 (g) + 4H + (𝑎𝑞) + 4𝑒 −
Thus, for the production of one mole of copper
436 (4)
from copper sulphate 2F of electricity is required.
Strong electropositive metals (I group, II group
445 (2)
and Al) cannot be obtained at cathode by
If 𝐹 is Faraday and 𝑁 is Avogadro number, charge
electrolysing their aqueous salt solutions.
of
437 (1) 𝐹
speed of ions m sec−1 electron = 𝑁
Ionic mobility = potential gradient = volt m−1
446 (1)
438 (2) A cation having highest reduction potential will be
1 faraday = 1 eq. of Cu = 1/2 mole Cu reduced first and so on. However, Mg 2+ in
P a g e | 76
aqueous solution will not be reduced of O2 in presence of H + , galvanization of iron
°
(EMg object is made.
2+ / Mg < E 1 ). Instead water would
H2 O/ H2 +OH−
2 458 (2)
be reduced in preference °
𝐸Cell °
= 𝐸𝑂𝑃 °
+ 𝐸𝑅𝑃
Sn Pb
447 (1) ° °
or𝐸𝑅𝑝 Sn
< 𝐸𝑅𝑃Pb , then Sn will oxidise to reduce
Because fluorine is most powerful oxidizing agent 2+
Pb .
than other halogens
459 (1)
𝑙
448 (2) Cell constant = 𝑎
Passage of current in electrolytic solution is due Where, 𝑙 = distance between the electrode
to migration of ions towards opposite electrodes. 𝑎 = area of the electrode
449 (1) 460 (2)
𝑡K∞+ = 𝑡KCl

× 𝑡K+ Standard electrode potential is measured by
130 × 0.495 = 64.35 voltmeter.
450 (1) 461 (1)
H2 is anode because oxidation takes place. Cu is The metals, present below hydrogen in the
cathode because reduction takes place electrochemical series, cannot liberate hydrogen
from the dilute acids.
451 (1)
Among the given metal only Ag is present below
∆𝐺° = −𝑛𝐸°𝐹 (for Cu + 2Ag + ⟶ Cu2+ + 2Ag) 𝑛
hydrogen in electrochemical series, so it does not
=2
evolve hydrogen withdil HCl.
= − 2 × 0.46 × 96500
Ag + dil. HCl → No reaction
= −88788 J = −89.0 kJ
462 (1)
452 (2)
N In MnO−4 the oxidation number of Mn is + 7.
Resistance of 10 solution = 2.5 × 103 Ω +7 +2
1 ∴ Mn + 5𝑒 − ⟶ Mn
κ = × cell constant
resistance In the reaction, 5 electrons are involved hence 5
1 Faraday will be needed for the reduction of 1
= × 1.25
2.5 × 103 mole of MnO− 4.
1.25 × 10−3 Therefore, for 0.5 mole of MnO−
= = 5 × 10−4 Ω−1 cm−1 4 , number of
2.5 Faradays required = 2.5 F
Equivalent conductance
463 (4)
𝜅 × 100
= Given, mass of nitrobenzene = 12.3 g
𝑀
Molecular mass of nitrobenzene = 123 g
5 × 10−4 × 1000 12.3
= ∴ Moles = = 0.1
1/10 123
= 5Ω−1 cm2 equiv −1
453 (4)
Follow theory of corrosion.
454 (3)
𝐸cell = 𝐸OP + 𝐸RP
455 (2)
° ° ° °
𝐸𝑂𝑃 Zn
> 𝐸𝑂𝑃 Cu
𝑜𝑟 𝐸𝑅𝑃Zn
< 𝐸𝑅𝑃 Cu
456 (3) ∴ H2 required for reduction of 0.1 mole
Mg → Mg 2+ + 2e− (at anode) nitrobenzene
Cu2+ + 2e− → Cu (at cathode) = 0.1 × 3 = 0.3 mol of hydrogen
°
𝐸cell °
= 𝐸cathode °
− 𝐸anode ∴ Amount of charge required to liberate 0.3 mol
= + 0.34 – (- 2.37) of hydrogen
= + 2.71 V = 2 × 96500 × 0.3 (∵ H2 ⟶ 2H + )
457 (4) = 57900 C
To prevent rusting or oxidation of Fe by the action 464 (3)

P a g e | 77
Rust is Fe2 O3 . 𝑥H2 O or mixture of Fe2 O3 + ° ° °
𝐸cell = 𝐸cathode - 𝐸anode
Fe(OH)3 . = (-0.14) – (-0.44)
465 (1) = - 0.14 + 0.44
−4
𝐾 × 1000 4.23 x10 = 0.30 V
Λ𝑣 = = × 1000
𝑐 0.04 474 (4)
Λ
Also, 𝛼 = Λ∞𝑣 By Nernst equation,
Λ 4.23 × 10−4 × 1000 ° 2.303 𝑅𝑇
∴ Λ∞ = 𝑣 = 𝐸cell = 𝐸cell − log10 𝐾
𝛼 0.04 × 0.0612 𝑛𝐹
= 172.8 At equilibrium 𝐸cell =0
466 (1) Given that,
Ionic mobility ∴ 𝑅 = 8.315 JK −1 mol−1
ionic conductance 𝑇 = 25 ℃ + 273 = 298𝐾
µAg+ =
96500 𝐹 = 96500 𝐶 and 𝑛 = 2
5 × 10−4 2.303 × 8.314 × 298
= °
96500 ∴ 𝐸cell = log10 𝐾
2 × 96500
= 5.2 × 10−9 cm/s 0.0591
= log10 𝐾
467 (2) 2
°
∆𝐺 ° = −𝑛𝐹𝐸 ° ∵ Given that 𝐸𝑐𝑒𝑙𝑙 = 0.295 V
0.0591
∴ 0.295 = log10 𝐾
−21.2 = −1 × 96500 × 𝐸 2
0.295 × 2
log10 𝐾 = = 10
21.2 0.0591
𝐸= = 0.220 V
96500 log10 𝐾 = antilog 10
𝐾 = 1 × 1010
468 (1)
475 (1)
1 1
𝜆= 𝑘 × 𝑉= × × 𝑉 𝑤 = 𝑍𝑖𝑡
𝑅 𝑎
1 32.69 × 5 × 60 × 40
= × 0.66 × 100000 ∴𝑤= = 4.065 g
210 96500
= 314. 28 mho cm2 eq.−1
469 (1) 476 (4)
A characteristic of electrode. 0.059
𝐸° = log 𝐾;
470 (2) 𝑛
1000 477 (3)
Λ= 𝑘 × 𝐸𝑂𝑃° °
= +Ve thus, E𝑅𝑃 = −ve.
𝑀
1 1000 478 (4)
=𝐶 × ×
𝑎 𝑀 Λ̊ 𝑚 (CH3 COOH) = Λ̊ 𝑚 (CH3 COONa) + Λ̊ 𝑚 (HCl)
Λ× 𝑎 × 𝑀
∴ 𝐶= − Λ̊ 𝑚 (NaCl)
𝑙 × 1000
= 91.0 + 425.9 – 126.4
𝑖. 𝑒. , 𝐶 ∝ conc.
= 516.9 – 126.4
∝𝑎
1 = 390.5 Scm2 mol−1
∝ 479 (4)
𝑙
𝐶 × 𝑙 × 1000 It is fact.
∴ unit of constant(Λ) = 480 (1)
𝑎 × 𝑀
𝑆 ×m At cathode : Cu2+ + 2𝑒 ⟶ Cu;
= 2
m × mole m−3 At anode : Cu ⟶ Cu2+ + 2𝑒
= 𝑠 m2 mol−1 481 (1)
472 (2) The PH of 0.1 M HCl and 0.1 M acetic acid is
The electrode potential of glass electrode depends
only on [H + ]. not the same, because HCl is a strong acid,
473 (1)
so its pH is more and CH3 COOH is a weak acid,
Here Fe acts as anode while Sn act as cathode. We
know that,
P a g e | 78
so its pH is less atomic mass
Ag + + 𝑒 − → Ag, 𝐸Ag = = 108
1
482 (2)
𝑘 𝑤Ag 108
Cell constant = = 0.0212 × 55 Number of faraday = = =1
C 𝐸Ag 108
= 1.166 cm−1
483 (2) 492 (3)
1 Equivalent conductance =
H | H + || Ag + | Ag 1000 ×conductance ×cell constant
2 2 normality
°
𝐸cell °
= 𝐸cathode °
− 𝐸anode So, units are, Ω cm equiv or S cm2 equiv −1 .
−1 2 −1

493 (1)
° ° °
= 𝐸Ag+ /Ag − 𝐸H+/1H Follow 𝐸𝑂𝑃 values in electrochemical series.
2 2
494 (1)
= (0.80) − (0.0) = 0.80 V 0.059
𝐸1 = 𝐸° − log[H + ]1
1
484 (3) 0.059
𝐸2 = 𝐸° − log[H + ]2
This is Kohlrausch law for 𝑎2 𝑏 type of 1
electrolytes. on adding (also 𝐸H° = 0)
485 (2) 0.059
𝐸1 + 𝐸2 = − [log(H + )1 + log(H + )2 ]
° °
𝐸cell = 𝐸𝑂𝑃Al + 𝐸𝑅𝑃H 1
°
Now for CH3 COOH = CH3COO− + H +
∴ 1.66 = 𝐸𝑂𝑃 Al
+ 0 𝐾𝑎 [CH3 COOH]
486 (2) [H + ] =
[CH3 COO− ]
𝐸Ag × Q 108 × 9.65 𝑦
𝑤Ag = = = 1.08 × 10−2 g ∴ [H + ]1 = 𝐾𝑎 .
96500 96500 𝑥
+
𝑥
= 10.8 mg [H ]2 = 𝐾𝑎 .
𝑦
0.059 𝐾 .𝑦 𝐾 .𝑥
487 (2) ∴ 𝐸1 + 𝐸2 = − 1 [log 𝑎𝑥 + log 𝑎𝑦 ]
2H2 O + 2𝑒 − → H2 + 2OH −
= −0.059[2 log 𝐾𝑎 ]
For 0.01 mole H2 , 0.02 mole of electrons are 𝐸1 +𝐸2
log 𝐾𝑎 =
consumed charge required 2 ×(−0.059)
𝐸 +𝐸
= 0.02 × 96500 𝐶 = 𝑖 × 𝑡 log 𝐾𝑎 = 1 2
0.02 ×96500 0.118
Time required = 10 × 10−3
= 19.3 × 104 𝑠 or p𝐾𝑎 =
𝐸1 +𝐸2
0.118
495 (3)
488 (1) 3 eq. of Ag and 3 eq. of Cu will be deposited. Na
The substances which have lower reduction and Al will not deposit during electrolysis of their
potentials are stronger reducing agent. The aqueous salt solution. Because both have higher
reduction potential of zinc is lowest among these ° °
𝐸𝑂𝑃 than 𝐸𝑂𝑃 of H. These metals can be extracted
hence, it is the strongest reducing agent.
by electrolysis of their fused salts.
489 (3)
496 (1)
H2 gas is evolved, when Zn reacts with NH4 Cl in
Reactions
dry cell battery.
(i) Fe(𝑠) → Fe2+ + 2𝑒 − , 𝐸 ° = + 0.44 V
Zn + 2NH4 Cl → Zn (NH3 )2 Cl2 + H2 ↑
Colourless
gas
and ∆𝐺1° = − 𝑛𝐸 ° 𝐹 = − 2 × 0.44 × 𝐹
490 (1) 1
° (ii)2H + + 2𝑒 − + 2 O2 ⟶ H2 O(𝑙); 𝐸 ° = +1.23 𝑉
The correct relation of 𝐸cell with free energy of
cell reaction and equilibrium constant are as
and ∆𝐺2° = − 2 × (+ 1.23) × 𝐹
follows
°
Δ𝐺° = 𝑛𝐹𝐸cell Net reaction,
and Δ𝐺° = −2.303 𝑅𝑇 log𝐾eq
1
491 (1) Fe(𝑠) + 2H + + O2 → Fe2+ + H2 O(𝑙)
2
P a g e | 79
∆𝐺3° = ∆𝐺1° + ∆𝐺2° ∵ 1 eq. H2 = 11.2 L
∴ 0.5 eq. H2 = 5.6 L
= −2 × (+ 0.44) F + (− 2 × 1.23 × F) 507 (1)
Eq. of H2 = 8/1 = 8
= − 0.88 F × − 2.46 F = − 3.34 F
∴ 8 faraday are needed.
= − 3.34 × 96500 J 508 (4)
At equilibrium 𝐸cell = 0.
= −322.31 kJ = − 322 kJ 509 (3)
2 × 96500 C electricity is used to liberate
497 (2)
∞ ∞
Λ∞ ∞
AcOH = Λ AcONa + Λ HCl − Λ NaCl = 22400 mL O2 at STP
= 91.0 + 426.2 – 126.5
= 390.7 ∴ 9.65 × 1000 C electricity will liberate
498 (2)
22400 × 9.65 × 1000
0.0591 0.01 0.0591 =
𝐸1 = 𝐸0 − log = 𝐸0 + ×2 2 × 96500
2 1 2
0.0591 1 0.0591 = 1120mL
𝐸2 = 𝐸0 − log = 𝐸0 − ×2
2 0.01 2
∴ 𝐸1 > 𝐸2 510 (4)
499 (2) During electrolysis of NaCl(𝑎𝑞), H + ions are
𝐸cell = +ve thus redox changes are discharged at cathode and the pH of solution
𝐴 ⟶ 𝐴+ + 𝑒 LHS shows oxidation. increases due to decrease in[H + ].
𝐵 + + 𝑒 ⟶ 𝐵RHS shows reduction. 511 (2)
𝐴 + 𝐵+ ⟶ 𝐴+ + 𝐵 °
Ecell °
= E(RHS) °
− E(LHS) = 1.61 − 1.51 = 0.10 V
500 (1)
Follow Debye Hückel theory of strong ∆𝐺 ° = −𝑛𝐹𝐸 °
electrolytes.
501 (2) = −5 × 96500 × 0.10 J
Fe3+ and Fe (CN)3− 6 are oxidants. Higher is 𝐸𝑅𝑃
°
∆𝐺 ° = −48.25 kJ
stronger is oxidant.
Fe3+ + 𝑒 ⟶ Fe2+ ; 𝐸𝑅𝑃 °
= 0.77 V 512 (2)
[Fe(CN)6 ] + 𝑒 ⟶ [Fe(CN)6 ]4− ; 𝐸𝑅𝑃
3− ° H2 O ⟶ H2 + 1/2O2
= 0.35 V
503 (2) ∴ Mole ratio of H2 O: O2 ∷ 1: 1/2
Mg lie above Cu in electrochemical series and 513 (4)
°
hence, Cu electrode acts as cathode 𝐸𝑂𝑃 forLi/Li+ is maximum in these.
°
𝐸cell °
= 𝐸Cu °
2+ / Cu = 𝐸Mg2+ / Mg 514 (3)
° 0.059
∴ 2.70 V = 0.34 − 𝐸Mg2+ /Mg 𝐸° = log𝐾𝑐
𝑛
°
∴ 𝐸Mg2+ / Mg = −2.36 V
∴ 0.295 =
0.059
log𝐾𝑐
2
504 (4) ∴ 𝐾𝑐 = 1010
When Alead storage battery is discharged, the 515 (3)
following cell reactions take place. °
𝐸cell forZn|Zn2+ ||Ni2+ |Ni is positive
At anode ° °
𝐸° = 𝐸𝑂𝑃 + 𝐸𝑅𝑃 = 0.76 − 0.23 = 0.53 V
Pb + H2 SO4 ⟶ PbSO4 + 2H + + 2e− 𝑍𝑛 Ni

At cathode 516 (1)


Given, that
PbO2 + 4H + + SO2− −
4 + 2e ⟶ PbSO4 + 2H2 O
Zn / Zn2+ || Cu2+ / Cu
505 (2)
∴ Zn is anode and Cu is cathode.
𝑄 = 10−6 × 1C ;
Given,
Also, 96500 C = 6.023 × 1023 electrons.
Zn2+ / Zn = − 0.76 V
506 (3)
4.5 Cu2+ / Cu = + 0.34 V
Eq. of H2 = Eq. of Al = = 0.5
27/3 𝐸cell = 𝐸cathode − 𝐸anode
P a g e | 80
= 0.34 – (- 0.76) 𝐸cell increases.
= 0.34 + 0.76
= 1.10 V 528 (4)
517 (1) For the cell, Ni | Ni2+ || Au3+ | Au
°
Sulphuric acid (H2 SO4 ) used in lead storage Given, 𝐸Ni 2+ /Ni = −0.25V

battery. °
𝐸Au 3+ /Au = +1.5 𝑉

518 (1) Here, Ni is anode and Au is cathode.


Electrical energy is provided during charging of ∴ 𝐸cell = 𝐸𝐶 − 𝐸𝐴
battery. = 1.5 – (-0.25)
519 (4) = 1.5 + 0.25
Ionic mobilities increase on dilution. = 1.75 V
520 (3) 529 (2)
These are the facts about use of Mg in protecting °
𝐸cell = 𝐸anode(op) °
− 𝐸cathode(op)
iron against corrosion.
= 0.76 – 0.41
521 (4)
= +0.35 V
Complex formation enhances the electrolytic
530 (1)
deposition of Au.
Standard electrode potential of hydrogen
522 (3)
electrode is zero.
pH < 7; Aqueous solution of CuSO4 is acidic in
531 (2)
nature. Furthermore some drops of H2 SO4 is also
1 g atom of Mg = 2g eq. Mg = 2 faraday.
added during electrolysis.
532 (3)
523 (3)
Sn2+ → Sn4+ + 2𝑒 −
Cryolite is used to lower the m. p. of alumina as 0.059 [Sn4+ ]
well as to make it good conductor of current. 𝐸cell = 𝐸 ° - 𝑛
log [Sn2+]
524 (1) = 𝐸° -
0.059 0.01
log [ ]
2 0.1
Faraday discovered electrolysis. 0.059
°
525 (4) = 𝐸 +
2
H2 SO4 will furnish maximum H + . 533 (4)
526 (4) More is 𝐸𝑂𝑃°
, more is reducing power of metal.
°
𝐸cell °
= 𝐸𝑂𝑃 I
°
+ 𝐸𝑅𝑃II
534 (4)
; I is oxidised, II is reduced.
527 (3) The elements which are below H2 in
+ 2+
Zn (𝑠) + 2H (𝑎𝑞) → Zn (𝑎𝑞) + H2 (g) electrochemical series, cannot displaceH2 .
[Zn2+ ] ∵Out of Li+ , Sr 2+ , Al3+ and Ag + , Ag + is below H2 in
Reaction quotient 𝑄 = [H+ ]2
electrochemical series,
Corresponding cell is so Ag + cannot displaceH2 .
2+ +
Zn | Zn (C1 ) || H (aq) | Pt (H2 ) 535 (4)
-anode + cathode
Nature of ion also includes size, charge on ion.
° 0.0591 536 (2)
and 𝐸cell = 𝐸cell − log 𝐾 °
2
𝐸cell for the reaction 𝑀+ + 𝑋 − → 𝑀 + 𝑋 is given
0.0591 [Zn2+ ] as follows
°
= 𝐸cell − log + 2 °
= 𝐸cathode °
+ 𝐸anode
2 [H ]
= 0.44 V + 0.33 V
+ 2
° 0.0591 [H ] = + 0.11 V
= 𝐸cell + log 2+ °
2 [Zn ] Since, 𝐸cell is positive for the above reaction .
Hence, this reaction is spontaneous.
If H2 SO4 is added to cathodic compartment,
537 (2)
(towards reactant side), then 𝒬 decreases (due to
When 1 F electricity is passed through the
increase in H + ).
solution. 1 g-equivalent of Cu is liberated.
Hence, equilibrium is displaced towards right and 538 (2)
° 0.059
𝐸𝑅𝑃 = 𝐸RP + log[H + ]
1

P a g e | 81
= 0 + 0.059× (−3) = −0.177 V. [Zn2+ ] 1
𝑄 = = = 10
539 (1) [Cu2+ ] 0.1
° ° ° 0.0591
𝐸cell = 𝐸𝑂𝑃 + 𝐸𝑅𝑃
Mg Cu 𝐸cell = 1.10 – log 10
= 2.37 + 0.34 = 2.71 V. 2
= 1.10 – 0.0295
540 (2)
= 1.0705 V
Cr2 O2− −
7 + 6𝑒 ⟶ 2Cr
3+
550 (1)
Reduction of 1 mol of Cr2 O2−
7 to Cr
3+
required 6
For the given cell, reaction is
moles of electrons. Hence, charge required = 2 ×
96500 C Zn + Fe2+ → Zn2+ + Fe
541 (3)
Sn4+ + 2𝑒 ⟶ Sn2+ ; 𝐸𝑅𝑝 °
= 0.15 V 0.0591 𝐶1
𝐸 = 𝐸° − log
Cr 3+ + 3𝑒 ⟶ Cr; °
𝐸𝑅𝑃 = −0.74 (Higher 𝑛 𝐶2
𝐸𝑂𝑃 ) 0.0591 𝐶
or, 𝐸° = 𝐸 + log 𝐶1
Redox change 2Cr + 2Sn4+ ⟶ 2Cr 3+ + 3Sn2+ 𝑛 2
° °
∴ 𝐸cell = 𝐸𝑂𝑃 + 𝐸𝑅𝑃 = 0.74 + 0.15 = +0.89 V
Cr Sn 0.0591 10−2
542 (3) = 0.2905 + log −3 = 0.32 V
2 10
2 3+ 2
(Al )2 + 4𝑒 − ⟶ Al
3 3 0.0591
2 2− −
𝐸° = log 𝐾c
or 3 (O )3 ⟶ 4𝑒 + O2 2
−∆𝐺 = 𝑛𝐹𝐸 (𝐸 is potential required) 0.32 × 2 0.32
960 × 1000 ∴ log 𝐾𝑐 = =
𝐸= − = −2.5 V 0.0591 0.0295
4 × 96500
0.32
543 (2) 𝐾𝑐 = 100.295
2H2 O ⇌ 4H + + 2O2−
At cathode, 4H + + 4𝑒 − → 2H2 551 (2)
At anode, 2O2− − 4𝑒 − → O2 𝑘
Cell constant = = 0.0212 × 55
∴ 4 Faraday of charge liberates = 1 mol C
= 22.4 dm3 O2 = 1.166 cm−1
∴ 1 Faraday of charge liberates 552 (1)
22.4 Conductance of a solution = conductance of ions
= × 1 present in solution = ∑ C𝑖 + Cwater .
4
= 5.6 dm3 O2 553 (1)
544 (1) Λ0M = Λ0𝑎 + Λ0𝑐
Half cell reaction occurs only when it is coupled 554 (1)
with other electrode. Fe is more electropositive than copper
545 (3)
Hence, Cu2+ can oxidize Fe
Eq. of Ag = Eq. of Hg 2+ = Eq. ofHg 2+
2
𝑊Hg2+ 𝑊Hg2+
or
0.216
= = 2 555 (4)
; (and 𝑎 = 200.6)
108 𝑎/2 𝑎/1
More is reduction potential, more is the power to
546 (2)
get itself reduced or greater is oxidising power.
Cations carrying positive charge move towards
556 (3)
cathode where they get discharged.
The ratio of amount deposited during the same
547 (1)
charge takes place in the ratio of their equivalent
We know from Kohlrausch’s law
weights and is independent of solution
⋀°CH3 COOH = ⋀°CH3 COONa + ⋀°HCl − ⋀°NaCl
concentration.
548 (3) 557 (1)
° ° 0.0591 𝐸. 𝑖. 𝑡 1 × 0.4 × 30 × 60
𝐸cell = 𝐸cell − 𝑙𝑜𝑔 𝒬 𝑊= =
𝑛 96500 96500
Cu2+ + Zn → Zn2+ + Cu −3 7.46 ×103 ×22.4
= 7.46 × 10 g and volume =
0.1 M 1M 2

P a g e | 82
= 0.0836 litre 1.15
= S cm−1
558 (4) 250
1 faraday deposits 1 g equivalent of any
1.15 × 1000
substance. ∴ Λ eq = 250 ×1
559 (2) Λ eq = 4.6 Ω−1 cm2 equiv −1
wt. of Al deposited eq. wt. of Al
=
wt. of Cu deposited eq. wt. of Cu 567 (1)
2.7 27/3
= 𝑋 coulomb deposits 1 mole Al or 3 eq. of Al and
wt. of Cu 63.5/2
thus, it will deposit 3 mole or 3 eq. of Ag is
wt. of Cu = 9.525 g
monovalent,
560 (4) 𝑊 𝑊
° 2.303𝑅𝑇
𝐸𝑐𝑒𝑙𝑙 = 𝑛𝐹 log 𝐾𝑒𝑞 [∴ (for Ag) = (for Al)]
𝐸 𝐸
0.0591 568 (3)
0.295 = log 𝐾𝑒𝑞
2 0.059
∴ log 𝐾𝑒𝑞 = 10 𝐸° = log𝐾𝑐
2
10
∴ 𝐾𝑒𝑞 = 10 0.059
0.46 = log𝐾𝑐
561 (3) 2
Metal placed above in electrochemical series log 𝐾𝑐 = 15.59
replaces the other from its salt solutions. ∴ 𝐾𝑐 = 3.9 × 1015
562 (4) 569 (1)
More or +ve is 𝐸𝑂𝑝 °
for an electrode more is its Quantity of current is charge, 𝑖. 𝑒., coulomb or
reducing power and 𝑣𝑖𝑐𝑒 − 𝑣𝑒𝑟𝑠𝑎. ampere sec.
563 (1) 570 (4)
𝑙 𝐹 =𝑁 ×𝑒
Given, 𝑎 = 0.5 cm−1
571 (3)
𝑅 = 50Ω It is better to write unit as kg coulomb−1 in place
𝑁 = 1.0 of g coulomb−1 .
1 𝑙 0.5
Specific conductance (𝜅) = 𝜌 = 𝑅.𝑎 = 50 572 (2)
1000 [Cu2+ ]
Λ = 𝜅 × −∆𝐺 = 𝑛𝐸𝐹 = 𝑛𝐸°𝐹 + 𝑅𝑇 ln
𝑁 [Zn2+ ]
0.5 1000
= × ∴ ∆𝐺 is function of ln(𝑐2 /𝑐1 ).
50 0.1
= 10 Ω−1 cm2 g eq−1 574 (1)
° °
564 (1) Metal with − ve. 𝐸𝑅𝑝 or + ve 𝐸𝑂𝑃 possesses the
1 1 1 tendency to get itself oxidised.
𝑘= × = × 0.66 575 (3)
𝑅 𝑎 210
= 3.14 × 10−3 mho cm−1 According to Faraday law’s
565 (4) 𝐸 ×𝑄
𝑚 = 𝑍 × 𝑄 =
° °
𝐸cell = 𝐸cathode − 𝐸anode ° 96500
2+ 3+ 108
Ni / Ni [1.0 M] || Au [1.0 M] | Au 𝑚 = × 2 × 96500 = 216 𝑔
3+ 96500
𝐸cell (Au / Au) = 0.150 V
576 (1)
𝐸cell (Ni2+ / Ni) = − 0.25 V °
° ° ° 𝐸Cr 3+ /Cr2+ = − 0.41 V
𝐸cell = 𝐸cathode − 𝐸anode
°
= 0.150 – (- 0.25) 𝐸Mn 3+ /Mn2+ = + 1.57 V

°
= 0.15 + 0.25 𝐸Fe 3+ /Fe2+ = + 0.77 V

= + 0.4 V °
𝐸Co 3+ /Co2+ = + 1.97 V
566 (2) °
𝜅 ×1000 More negative value of 𝐸red indicates better
Equivalent conductivity (Λ eq ) = 𝐶
reducing agent thus easily oxidized. Thus,
oxidation of Cr 2+ to Cr 3+ is the easiest.
cell costant
Conductivity (𝜅) = resistance 577 (1)
Cu is placed above Ag in electrochemical series,
P a g e | 83
hence it can replace Ag from its salts solution. 1 mol = 123 g nitrobenzene requires 6 mol
Therefore, the reaction occur as follows electrons
O +1
Oxidation = 6 × 96500 C charge
Cu + AgNO3 →⎯⎯⎯⎯⎯⎯⎯⎯⎯ CuNO3 + Ag
578 (2) 6×96500×12.3
∴12.3 g nitrobenzene will require= 123
Specific conductivity (κ)
1
= × cell constant = 6 × 9650 = 57900 C
R
Cell constant = 𝜅 × 𝑅 586 (3)
= 0.0129 × 100 = 1.29 It is definition of Kohlrausch’s law.
580 (3) 589 (2)
° 0.059 [Cu2+ ] Reaction for electrolysis of water is
𝐸cell = 𝐸cell + log ;
2 [Zn2+ ] 2H2 O ⇌ 4H + + 2O2−
Thus, on doubling concentration of both Cu2+ and 2O2− → O2 + 4𝑒 −
Zn2+ , there will be no effect no 𝐸cell . 4𝑒 − + 4H + → 2H2
581 (1) ∴ 𝑛 = 4,
∆𝐺° = −𝑛𝐹𝐸° So, 4 F charge liberates = 1 mol = 22.4 dm3
Fe2+ + 2𝑒 − → Fe …(i) oxygen
∆𝐺° = −2 × 𝐹 × (−0.440 V) = 0.880 F ∴ 1 F charge will liberate=
22.4
= 5.6 dm3 oxygen
4
Fe3+ + 3𝑒 − → Fe …(ii)
590 (2)
∆𝐺° = −3 × 𝐹 × (−0.036) = 0.108 F
The charge required to liberate one gram
On subtracting Eqs. (i) from (ii), we get
equivalent of an element is always equal to 1
Fe3+ + 𝑒 − → Fe2+
faraday (𝑖. 𝑒. , = 96500 𝐶).
∆𝐺° = 0.108 F − 0.880 F = −0.772 F
591 (1)
∆𝐺° (−0.772 F)
𝐸° for the reaction = − =− 1
𝑛𝐹 1×F Oxidizing tensdncy ∝
electrode potential
= +0.772 V
𝑇𝑋 → No reaction
582 (2)
𝑇𝑌 → 𝑋, 𝑍
NaCl ⟶ Na+ + Cl−
𝑇𝑍 → 𝑋
molten
⇒ Order of electrode potential is
At cathodeNa+ + 𝑒 − ⟶ Na
𝑇𝑌 < 𝑇𝑍 < 𝑇𝑋
At anode2Cl− − 2𝑒 − ⟶ Cl2
⇒ Order of oxidation of the anion is
When one mole of NaCl is electrolysed, 1 mole Na
1 𝑌− > 𝑍− > 𝑋−
and 2 mole Cl2 is obtained.
592 (3)
230
Thus, when 230 g( 23 = 10mol) Na is formed, the Given, current = 241.25 C
moles of We know that 1 C electricity will deposit 1.118 ×
moles of Na 10−3 g of silver.
Cl2 obtained will be = 2 ∴ 241.25 C electricity will deposit
10
= 2 = (1.118 × 10−3 ) × 241.25
= 5.0 mol = 0.27 g of silver.
583 (1) 593 (3)
1000 1 1 1000 In Agalvanic cell, oxidation (𝑖. 𝑒., removal of𝑒 − )
𝛬= 𝑘 × = × ×
𝑀 𝑅 𝑎 𝑀 occurs at anode. These electrons flow through
1 2.2 1000 external circuit from anode to cathode. Therefore,
= × × = 25.73
45 3.8 0.5 the direction of current in external circuit is from
584 (2) cathode (- ve) to anode (+ ve).
Electrons flow from Zn to Cu in outside circuit and 594 (1)
current from Cu to Zn. Anode is positive electrode and cathode is – ve
585 (4) electrode in electrolytic cell whereas, anode is
C6 H5 NO2 + 6H + + 6e− → C6 H5 NH2 + 2H2 O – ve electrode and cathode is +ve electrode in

P a g e | 84
electrochemical cells. is written as
595 (4) Cu (𝑠) + Hg 2+ (𝑎𝑞) → Cu2+ (𝑎𝑞) + Hg (𝑙)
Since, reduction occurs at Ag electrode hence, 604 (2)
+
increase in the concentration of Ag or decrease Specific conductivity of a solution decreases with
in concentration of Cu2+ will increase the voltage. dilution.
596 (1) 605 (1)
1 1 1
Conductance = 𝑅 = 210 At anode; H
2 2
+ OH − → H2 O + 𝑒 −
1
= 4.76 × 10−3 mho. At cathode; H+ + 𝑒 − ⇌ H
2 2
597 (3) Redox change; H + + OH − ⇌ H2 O
Blocks of magnesium metal provide cathodic [H2 O]
protection and protect oxidation of steel. 𝐾 =
[H + ][OH − ]
598 (4) 𝑅𝑇
Cobalt is anode, 𝑖𝑒, oxidation takes place on cobalt 𝐸° = In𝐾𝑤
𝐹
electrode 𝑖𝑒, cell reaction is 606 (2)
Smaller is the size of ion, more is hydration, lesser
Co + 2Ag + → Co2+ + 2Ag
is mobility, i.e., mobility of ion in aqueous solution
𝑅𝑇 [Co2+ ] decreases with decreases with decrease in size of
°
𝐸cell = 𝐸cell − ln hydrated ion.
𝑛𝐹 [Ag + ]2
607 (4)
[Co2+ ] ° In concentration cell, two electrolytic solutions of
Thus, less is the factor [Ag+ ]
, greater is the 𝐸cell
same electrolyte but having different
599 (2) concentrations (𝑒. g., Pt H2 |HCl| |HCl| Pt H2 ) are
𝑐1 𝑐2
As the reduction potential of Zn is less than that of
used. Moreover electrolyte of one strength but
Ag, hence Zn will act as anode when Acell is made
electrodes of two different concentrations are
using them.
used.
Hence , the correct reaction will be
608 (4)
Zn(s) → Zn2+ (aq) + 2e− (oxidation)
The degree of ionisation of an electrolyte
2Ag + (𝑎𝑞) + 2𝑒 − → 2Ag (𝑠)
increases with dilution and temperature as well
Zn(𝑠) + 2Ag + (𝑎𝑞) → Zn2+ (𝑎𝑞) + 2Ag (𝑠)
as in presence of polar solvent.
600 (1)
609 (4)
In the process of electro decomposition for 𝜅 × 1000
purification of metal, impure metal acts as anode. Equivalent conductivity, Λ eq = normality
601 (4) = ohm−1 cm2 (g − equiv)−1
Wt. of Cu deposited eq. wt. of Cu 610 (3)
=
Wt. of H2 produced eq. wt. of H According to Kohlrausch’s law

0.16 64/2 32 Λ∞ ∞
ClCH2 COOH = Λ ClCH2 COO− + Λ H+
= =
wt. of H2 1 1 Given from Kohlrausch law,
0.16
Wt. of H2 = 32 = 5 × 10−3 g Λ∞ −1 2
ClCH2 COONa = 224Ω cm g eq
−1

Volume of H2 liberated at STP Λ∞ ∞
ClCH2 COONa = Λ ClCH2 COO− + Λ Na+ …(i)
22400 Λ∞ −1 2 −1
= × 5 × 10−3 𝑐𝑐 HCl = 203Ω cm g eq
2 ∞ ∞ ∞
Λ HCl = Λ H + Λ Cl …(ii)
= 56 cc
Λ∞NaCl = 38.5 Ω −1
cm 2
g eq−1
602 (3) ∞
Λ∞NaCl = Λ Na+ + Λ Cl ….(iii)
1 g atom of Al = 3 eq. of Al = 3 faraday charge
Adding Eqs. (i) and (ii) and subtracting Eq. (iii)
3 mole electrons = 3𝑁 electron. ∞
Λ∞ClCH2 COO− + Λ H+
603 (4)
= Λ∞ ∞ ∞
ClCH2 COONa + Λ HCl − Λ NaCl
Galvanic cell is
Cu (𝑠) | Cu2+ (𝑎𝑞) || Hg 2+ (𝑎𝑞) | Hg (𝑙) = 224 + 203 – 38.5
In the above cell, oxidation of copper and = 427 – 38.5
reduction of mercury takes place. Its cell reaction = 388.5Ω−1 cm2 g eq−1

P a g e | 85
611 (3) 𝐸 × 𝑖 × 𝑡 23 × 5 × 10 × 69
𝑊Na = =
The molar conductivity of potassium 96500 96500
hexacyanoferrate (II) = 0.715 g
𝑖. 𝑒. , K 4 [Fe(CN)6 ] is highest because it gives 620 (2)
° °
maximum number of ions on ionization. 𝐸𝑂𝑃 = −𝐸𝑅𝑃 for any element.
K 4 [Fe(CN)6 ] ⟶ 4K + + [Fe(CN)6 ]4− 621 (3)
612 (4) Molar conductivity or molar conductance
1 (Λ 𝑚 ) = 𝜅 × 𝑉
H2 O ⟶ H2 + O2 . 1000
2 Λ𝑚 = 𝜅 ×
2H + + 2𝑒 − ⟶ H2 𝐶𝑚
∴ 1 mole of H2 is liberated from 2 moles of Where, 𝐶𝑚 is molar concentration (mol L−1 )
electrons. 1
∴ Molar conductance (𝛬𝑚 ) ∝ (𝐶 )
2 moles electrons = 2 F charge
622 (2)
= 2 × 96500 C charge °
Metal having higher 𝐸𝑂𝑃 replaces the other from
Given , 𝑖 = 4 𝐴, 𝑡 = 30 min
its solution.
∴ 2 × 96500 C charge liberates
623 (1)
= 22400mL H2 at NTP
Cell reaction is Mg + Sn2+ → Mg 2+ + Sn
∴ 4 × 30 × 60 C charge liberates
0.0591 [Mg 2+ ]
22400 °
𝐸cell = 𝐸cell − log
= × 4 × 30 × 60 mL H2 2 [Sn2+ ]
2 × 96500
= 835.6mL 0.0591 10−2
= (2.34 − 0.14) − log −1 = 2.23 V
= 0.836 L of H2 2 10
613 (2) 624 (1)
The unit of electrochemical equivalent (Z) is g/C. ° °
0.059 [H + ]2 . 𝑃1
𝐸cell = 𝐸𝑂𝑃 + 𝐸 𝑅𝑃H + log
𝑤 = 𝑍. 𝑖. 𝑡
H 2 [H + ]2 . 𝑃2
𝑤 0.059 𝑃1
∴ 𝑍 = g/𝐶 = log
𝑖. 𝑡 +2 𝑃2
614 (2) 𝐸cell is +ve when 𝑃1 > 𝑃2 .
2 faraday will deposit 2 eq. or 1 mole of Cu. 625 (4)
615 (2) −∆𝐺 = 𝑛𝐸𝐹
Mn7+ + 5𝑒 ⟶ Mn2+ ; − 966 × 103 = 4 × 𝐸 × 96500
Thus, 5 mole electron = 5 faraday. ∴ 𝐸 = −2.5 V
616 (4) Thus, an e.m.f. of + 2.5 is needed to carryout the
AgI (𝑠) + 𝑒 − ⇌ Ag (𝑠) + I − ; 𝐸 ° = 0.152 V electrolytic reduction of Al2 O3 .
Ag (s) ⟶ Ag + + 𝑒 − 𝐸° = - 0.8 V 626 (3)
AgI (𝑠) ⟶ Ag + + I − 𝐸 ° = - 0.952 Reducing power, 𝑖𝑒, the tendency to lose electrons
° 0.059 increases as the reduction potential decreases
𝐸cell = log 𝐾𝑠𝑝
𝑛
0.059 627 (2)
−0.952 = log 𝐾𝑠𝑝
1 Standard oxidation potentials of Zn, Cu, Ag and Ni
electrode are + 0.76, - 0.34, - 0.80 and + 0.25 V
−0.952
log 𝐾𝑠𝑝 = = − 16.135 respectively. (Given)
0.059
Zn2+ + 2e− → Zn ; 𝐸𝑐𝑒𝑙𝑙 = - 0.76 V
2+ −
617 (1) Cu + 2e → Cu ; 𝐸 cell = + 0.34 V
+ −
Cl2 is placed above F2 in electrochemical series, Ag + e → Ag ; 𝐸cell = + 0.80 V
halogen placed below replaces the other from its Ni2+ + 2e− → Ni ; 𝐸cell = - 0.25 V
solution. 4. Cu + 2Ag (aq) → Cu2+ (aq) + 2 Ag
618 (4)
° ° °
Λ𝑣 5.2 𝐸cell = 𝐸cathode − 𝐸anode
𝛼= ∞= = 0.0133 𝐸cell = 0.80 – 0.34 = 0.46 V
Λ 390.7
Or 1.33 %.
619 (1)
P a g e | 86
5. Zn + 2 Ag + (aq) → Zn2+ (aq) + 2 Ag When H2 SO4 is added then [H + ] will increase
therefore 𝐸cell will also increase and equilibrium
𝐸cell = 0.80 – (− 0.76) = 1.56 𝑉 will shift towards right
(maximum voltage) 631 (2)
6. H2 + Ni2+ (aq) → 2H + (aq) + Ni Electrode potential of cell must be positive for
spontaneous reaction.
𝐸cell = − 0.25 𝑉 Zn2+ ⟶ Zn ; 𝐸 ° = −0.76 𝑉
Cu2+ ⟶ Cu ; 𝐸 ° = −0.34 𝑉
2+ 2+
7. Zn + Cu (aq) → Zn (aq) + Cu Redox reaction is
𝐸cell = 0.34 – ( − 0.76) = 1.00 𝑉 Zn → Zn2+ + 2e− (oxidation)
Cu2+ + 2e− → Cu (reduction)
+ 2+ 2+ 2+
8. Zn + 2 H (aq) → Zn (aq) + H2 Zn + Cu → Zn + Cu
° °
𝐸cell = 𝐸cathode − 𝐸anode
𝐸cell = 0 – ( 0.76 𝑉) = 0.76 𝑉
= - 0.34 – (- 0.76)
628 (4) = + 0.42 V
More is solute-solute interaction, lesser is 𝐸cell positive , so above reaction is feasible.
is
conductance. More is solute-solvent interaction,
more is conductance. An increase in temperature
also increases conductance due to increase in
ionic mobility.
629 (4)
The cell reaction is,
1
H (g) +
2 2
AgCl(s)⟶ H + (aq) + Cl− (aq) + Ag (s)
Obviously, here hydrogen is being oxidized and
AgCl is getting reduced. Hence, the correct cell
representation will be
Pt | H2 (g) , HCl solution || AgCl(s) | 𝐴𝑔
630 (3)
Zn(𝑠) + 2H + (𝑎𝑞) ⇌ Zn2+ (𝑎𝑞) + H2 (g)

° 0.059 [Zn2+ ]
𝐸cell = 𝐸cell − log + 2
2 [H ]

P a g e | 87

You might also like